pharm rough draft

¡Supera tus tareas y exámenes ahora con Quizwiz!

A client is diagnosed with Helicobacter pylori infection. The physician will order amoxicillin and what other type of medication?

Proton pump inhibitors Rationale: The treatment of choice for Helicobacter pylori infection is a PPI and clarithromycin plus either amoxicillin or metronidazole.

The nurse is caring for a 73-year-old client receiving warfarin. When the nurse performs the initial shift assessment, the nurse observes blood in the client's urinary drainage bag. After reporting the observation to the physician, which substance will the nurse likely administer?

Vitamin K

What happens if excretion does not take place?

Waste (excess drugs) will build up and we start to see adverse effects.

Which factor would prohibit the administration of glipizide? -increase in alkaline phosphatase -the ingestion of carbohydrates -allergy to sulfonamides -a diagnosis of hypertension

allergy to sulfonamides

A client who received a tissue plasminogen activator (tPA) begins to bleed. Which medication will the nurse provide as prescribed to control the bleeding?

aminocaproic acid

Because of the risk for lactic acidosis, metformin is contraindicated in which client? -a 50-year-old who has undergone surgery 2 weeks ago -a 37-year-old woman who takes oral contraceptives -an 82-year-old diagnosed with type 2 diabetes -a 16-year-old with a diagnosis of anorexia nervosa

an 82-year-old diagnosed with type 2 diabetes

The nursing instructor is teaching students about proper administration of sucralfate. According to the instructor, sucralfate should be administered

an hour before meals. Rationale: Sucralfate, a proton pump inhibitor, must be given time to act before significant gastric activity, and it requires an acidic environment to be effective. For these reasons, sucralfate should be given an hour before meals and antacids.

Antiinfective agents

are drugs designed to target foreign organisms that have invaded and infected the body of a human host. For centuries, people have used various naturally occurring chemicals in an effort to treat disease. Often this was a random act that proved useful. For instance, the ancient Chinese found that applying moldy soybean curds to boils and infected wounds helped prevent infection or hastened cure. Their finding was, perhaps, a forerunner to the penicillins used today.

Misoprostol is a synthetic form of prostaglandin E prescribed to protect the gastric mucosa from erosion and ulceration. The drug is contraindicated in patients who

are pregnant. Rationale: Misoprostol is contraindicated during pregnancy because it may cause abortion, premature birth, or birth defects. It is also contraindicated in women of childbearing potential who are not using effective contraception.

A 51-year-old man is being discharged from the hospital following treatment with anticoagulants for a deep vein thrombosis. The nurse will instruct the client to:

consider safety measures to prevent bleeding and be alert for signs of bleeding.

A client has been diagnosed with hypothyroidism and admits to the nurse that she has heard of her thyroid gland but does not know the function of thyroid hormone. The nurse should explain the fact that thyroid hormone is responsible for which action? -regulating the levels of most other hormones in the body -controlling the rate of cell metabolism throughout the body -stimulating the brain and sex organs -regulating levels of glucose in the blood and body tissues

controlling the rate of cell metabolism throughout the body

A 10 year-old client has been diagnosed with a cestode (tapeworm) infection that is believed to be longstanding. The nurse who is providing the client's care and administering medications should:

ensure that the client receives frequent, nutritious meals.

The nurse cautions a woman who is breastfeeding about avoiding the use of magnesium laxatives based on the understanding that the:

newborn may experience diarrhea

Prior to administering an antihyperlipidemic to a client, what information does the nurse need to gather? Select all that apply:

• Dietary history • Vital signs • Weight The pre-administration assessment for antihyperlipidemic drugs includes a lipid profile, liver function tests, dietary history, vital signs, weight, and an inspection for xanthomas.

Administration of a cardiotonic drug is contraindicated in which of the following conditions? Select all that apply:

• Digitalis toxicity • Cardiac tamponade • Ventricular tachycardia

What question is important for the nurse to ask a client who is scheduled to receive a first dose of radioactive iodine for hyperthyroidism? -"Do you have any trouble swallowing?" -"Have you discussed your cancer treatment plan with your provider?" -"Are you afraid of needles?" -"Does anyone ever have trouble finding your veins?"

"Do you have any trouble swallowing?"

A client who is prescribed digoxin asks the nurse how he should take the drug. Which instruction would be most appropriate?

"Take the drug on an empty stomach at the same time each day."

An 86-year-old male client who was admitted earlier in the week with thrombophlebitis is being sent home on enoxaparin. Which statement by the client suggests that he understands proper management of his condition and proper use of the drug?

"The medication will increase my risk of bleeding."

The nurse is aware that critically ill clients are more at risk for constipation for what reasons? (Select all that apply.)

-Decreased activity -Use of opioid medications -Change in bowel routines

A nurse should monitor a client taking glimepiride (Amaryl) for which of adverse effects? (Select all that apply.) -Nausea -Edema -Heartburn -Hypoglycemia -Lactic acidosis

-Hypoglycemia -Heartburn -Nausea

Levothyroxine (Synthroid) 88 mcg is prescribed for a client. How many mg of Synthroid will the nurse administer? Do NOT round the answer.

0.088 mg -1 mg=1000 mcg

The nurse identifies the therapeutic range for digoxin as:

0.8 to 2 ng/mL

The client is scheduled to get a breakfast tray at 07:00. At what time should the client receive a prescribed dose of insulin lispro? -07:00 -06:45 -06:00 -06:20

06:45

A missionary who works overseas has been diagnosed with threadworms and has been prescribed ivermectin 200 micrograms(mcg)/kg PO as a one-time dose. The client weighs 132 lbs so the nurse should administer how many milligrams (mg) of ivermectin?

12

Recovering from laxative abuse takes time and patience. After using laxatives to stimulate defecation, approximately how long will it take for the fecal column to re-establish with normal food intake?

2 to 3 days

A nurse must immediately report serum digoxin levels greater than? (Choose one)

2.0 nanograms/mL

The nurse is reviewing the coagulation studies of a client who is receiving a heparin infusion. The client's baseline partial thromboplastin time is 32 seconds. Which result would indicate therapeutic effectiveness?

64 seconds

A pediatric nurse is caring for a 4 year-old child who has been diagnosed with the most common childhood helminthic disease in the United States. The nurse should anticipate what intervention?

A three-day course of mebendazole

A client whose daily commute includes travel on a ferry boat has been prescribed diphenhydramine. During client teaching, the nurse should instruct the client to take the medication at what time? A) 30 to 60 minutes before the ferry trip B) the night before each trip C) as soon as the ferry leaves the dock D) when the client boards the ferry

A) 30 to 60 minutes before the ferry trip Explanation: To prevent motion sickness, it is necessary to take diphenhydramine 30 to 60 minutes before the trip begins.

A patient who is prescribed ipratropium administers the drug at 9:15 AM. The patient should begin to notice the drug beginning to act at which time? A) 9:30 AM B) 9:45 AM C) 10:00 AM D) 10:15 AM

A) 9:30 AM Explanation: Inhaled ipratropium has an onset of action of 15 minutes, so the patient should begin to feel the effects of the drug at 9:30 AM.

A client has been admitted to a health care facility with asthma. The nurse is to administer theophylline to the client. To which clients can the nurse safely administer theophylline? A) A 65-year-old male with asthma B) A 65-year-old female with hepatic disease C) An 83-year-old female with cardiac disease D) A 43-year-old male with hypertension

A) A 65-year-old male with asthma Explanation: The nurse can safely administer theophylline to the client who is 65 years of age. It needs to be administered cautiously in clients older 69 years or those with hepatic disease, cardiac disease, or hypertension.

The nurse is providing care for a diverse group of clients. Which client should the nurse monitor most closely for signs and symptoms of a fungal infection? A) A client who is receiving combination therapy for acquired immunodeficiency syndrome (AIDS) B) A client with breast cancer who is post-operative day two following a mastectomy C) A client with chronic renal failure who has peritoneal dialysis three times per week D) An older adult client with osteoporosis who is taking a bisphosphonate

A) A client who is receiving combination therapy for acquired immunodeficiency syndrome (AIDS) Explanation: Clients who are immunosuppressed, such as those who have AIDS, are at particular risk for fungal infections. Many older adults also have high risks, but the risk among clients with AIDS is known to be particularly high. Post-operative clients and those with renal failure are not particularly high-risk.

What medications are classified as echinocandin antifungals? (Select all that apply.) A) Anidulafungin B) Voriconazole C) Caspofungin D) Micafungin E) Terbinafine

A) Anidulafungin C) Caspofungin D) Micafungin Explanation: Antifungals in the echinocandin group include anidulafungin, caspofungin, and micafungin. The other two are azoles.

Common, potentially serious, adverse effects of antibiotic drugs include: A) Skin rash B) Pain C) Constipation D) Hypopnea

A) Skin rash Explanation: Examine skin for any rash or lesions, examine injection sites for abscess formation, and note respiratory status—including rate, depth, and adventitious sounds to provide a baseline for determining adverse reactions. Report nausea, vomiting, diarrhea, skin rash, recurrence of symptoms for which the antibiotic drug was prescribed, or signs of new infection (e.g., fever, cough, sore mouth, drainage). These problems may indicate adverse effects of the drug, lack of therapeutic response to the drug, or another infection. Pain, constipation, and hypopnea are not common adverse effects of antibiotic drugs.

A 42-year-old male client has a history of smoking for 20 years. The client states he is feeling fine and asks why stopping smoking is necessary. What information might the nurse share with the client about smoking? A) Smoking decreases the size of the tubes leading to the lungs B) The chest becomes more elastic with increased time of smoking C) Carbon dioxide levels may decrease in the lungs D) Smoking causes the respiratory system to increase oxygen delivery to the lungs

A) Smoking decreases the size of the tubes leading to the lungs Explanation: Smoking can decrease the efficiency of the respiratory system. Nicotine causes a decrease in bronchial diameter, constriction of blood vessels, a decrease in ciliary function (which assists in moving foreign particles out of the respiratory tract), and can destroy lung tissue over time. These factors can all result in a decrease in gas exchange. In addition, many tobacco products contain substances (e.g., tars) that can build up in the lungs. The chest does not become more elastic with increased time of smoking; instead, it loses elasticity and becomes stiffer. Thus, the lungs cannot expand, and the change in the ratio of pressure of oxygen and carbon dioxide in the lungs is the effect of aging on the respiratory system.

First-generation antihistamines may be effective against what symptoms? (Select all that apply.) A) Sneezing B) Rhinorrhea C) Cough D) Congestion E) Fever

A) Sneezing B) Rhinorrhea C) Cough Explanation: First-generation antihistamines (e.g., chlorpheniramine, diphenhydramine) have anticholinergic effects that may reduce sneezing, rhinorrhea, and cough. They do not have antipyretic effects.

The nurse is describing how aminophylline achieves its effect. Which would the nurse incorporate into the description? A) Stimulation of the central nervous system B) Stimulation of beta-adrenergic receptors C) Reduction of airway hyperresponsiveness D) Stabilization of mast cell membranes

A) Stimulation of the central nervous system Explanation: Stimulation of the central nervous system is the mechanism of action for xanthine derivatives. Stimulation of beta-adrenergic receptors is the mechanism of action for beta-2 agonists. Reduction of airway hyperresponsiveness is the mechanism of action for inhaled corticosteroids. Stabilization of mast cell membranes is the mechanism of action for mast cell stabilizers.

A client with a complex medical history is showing signs and symptoms of sepsis. What aspect of this client's health history would rule out the safe and effective use of an aminoglycoside antibiotic? A) The client has chronic renal failure B) The client has a history of not adhering to treatment C) The client has type 2 diabetes, controlled with oral antihyperglycemics D) The client has a known latex allergy

A) The client has chronic renal failure Explanation: Renal failure would preclude the use of an aminoglycoside. Nonadherence must always be addressed, but this client variable is not specific to aminoglycosides. Neither latex allergies nor type 2 diabetes would necessarily rule out the use of an aminoglycoside.

The client is receiving ketoconazole. The nurse should be concerned about a drug-drug interaction on the basis of what other comorbidity? A) Type 2 diabetes B) Chronic pain C) Venous ulcers D) Penicillin allergy

A) Type 2 diabetes Explanation: Ketoconazole interacts with hypoglycemics, which are given for the treatment of type 2 diabetes. There is no obvious reason why a client's pain, penicillin allergy or venous ulcer would preclude the safe and effective use of ketoconazole.

A nurse is teaching a colleague about the factors that are known to contribute to antibiotic resistance. When phenomenon should the nurse describe? A) Use of antibiotics for viral illnesses B) Use of excessive doses during anti-infective therapy C) Over-the-counter availability of some antibiotics in the United States D) Increasing incidence of adverse effects of anti-infectives in recent years

A) Use of antibiotics for viral illnesses Explanation: The use of antibiotics for viral illnesses or infections is a contributing factor to the development of resistance. Antibiotics are not available over the counter in the United States and there has not been a trend toward excessive dosing or increases in adverse effects.

Anidulafungin is prescribed to treat what form of fungal infection? A) candidiasis B) histoplasmosis C) tinea infection D) aspergillosis

A) candidiasis Explanation: Anidulafungin is a semisynthetic antifungal medication that inhibits glucan synthase, an essential component in the fungal cell wall. It is used to treat esophageal candidiasis and other Candida infections. Anidulafungin is not prescribed to effectively treat any of the other suggested fungal infections.

What medication should be prescribed to minimize the risk associated with a blood transfusion for a client with a history of a previous transfusion reaction? A) diphenhydramine B) epinephrine C) clemastine D) cetirizine

A) diphenhydramine Explanation: A client who is having a blood transfusion or a diagnostic test may receive diphenhydramine, often by injection and usually as a single dose, to prevent allergic reactions. Epinephrine is the drug of choice for treating severe anaphylaxis once it has occurred. Clemastine is prescribed for allergic rhinitis and urticaria. Cetirizine is prescribed for the management of seasonal allergies.

A client is receiving an aminoglycoside antibiotic for an infection. The nurse would monitor the client closely for: A) hearing loss. B) lethargy. C) visual changes. D) hallucinations.

A) hearing loss. Explanation: Aminoglycosides collect in the eighth cranial nerve and can cause hearing loss, dizziness, and vertigo. Lethargy and hallucinations may be associated with other anti-infective agents. Visual changes such as blindness are associated with chloroquine use.

The primary health care provider prescribes flucytosine to treat a systemic fungal infection. The nurse would administer this drug by which route? A) oral B) IV C) IM D) topical

A) oral Explanation: Flucytosine is given orally. The options of IM, IV, and topical are not methods of administration for this drug.

The nurse is caring for a client receiving amphotericin B. Which condition should the nurse assess for in this client? A) phlebitis B) bruising at the intramuscular injection site C) nausea 30 minutes after administered orally D) swelling at the subcutaneous injection site

A) phlebitis Explanation: Amphotericin B and flucytosine are available in IV form. During intravenous administration, there could be pain at the injection site with the possibility of phlebitis or thrombophlebitis.

A client is receiving amphotericin B. The nurse would assess the client closely for which adverse reaction? A) renal impairment B) intensive weight loss C) skin discoloration D) CNS disorders

A) renal impairment Explanation: Renal damage is the most serious adverse reaction to the use of amphotericin B and requires close monitoring. Conditions such as CNS disorders, skin discoloration, and weight loss are not known to be common adverse effects of amphotericin B.

When speaking of pharmacokinetics, water soluble drugs are considered to be?

Active because they need a carrier. Ex: BP medication

The nurse is caring for a 25-year-old female client who is receiving chorionic gonadotropin alpha. What would be the most appropriate nursing diagnosis for this client? a.Risk for imbalanced fluid volume related to diuresis b.Acute pain related to administration of injections c.Imbalanced nutrition: more than body requirements related to increased appetite d.Risk for disproportionate growth related to adverse effects

Acute pain related to administration of injections Feedback: Nursing diagnoses related to drug therapy might include acute pain related to need for injections. Not all patients are anxious at the thought of an injection so more information would be needed. The patient is more likely to have reduced nutritional intake because of gastrointestinal (GI) adverse effects. Evaluating the effectiveness of the teaching plan is not a nursing diagnosis.

A nurse is caring for a patient with intestinal stenosis who has been prescribed psyllium. During the course of the treatment, the patient shows the signs of colon obstruction. What intervention should the nurse perform to avoid the occurrence of colon obstruction?

Administer the drug with adequate fluid intake

What would a nurse identify as a systemic hemostatic agent?

Aminocaproic acid

A common drug regimen for eradication of H. pylori includes a proton pump inhibitor (PPI) and two antibiotics. Which of the following is one of the preferred antibiotics?

Amoxicillin Rationale: The treatment of choice for H. pylori infection is a PPI and clarithromycin plus either amoxicillin or metronidazole.

Levofloxacin 750 mg IV is ordered for a client with a urinary tract infection. The medication is to mixed yielding 250 mg/15 mL. How many mL should be drawn up in the syringe?

Answer: 45mL Explanation: 750/250 = 3; 3 x 15 mL = 45 mL

The nurse's assessment of a client who has presented to the emergency department reveals hyperglycemia. Which type of insulin will have the most rapid effect on the client's blood sugar levels? -30/70 -Regular -NPH -Aspart

Aspart

A young adult, diagnosed with hookworms, has begun taking mebendazole. When monitoring this client's health status during treatment, what diagnostic value should the nurse prioritize?

Aspartate aminotransferase (AST) and alanine aminotransferase (ALT)

A client with acute coronary syndrome is prescribed clopidogrel. Which additional medication will the nurse expect to be prescribed for this client?

Aspirin

The nurse is evaluating the education of a client that uses albuterol for an acute asthma attack. The nurse knows that the lesson has been effective when the client states that albuterol is which of the following types of medication? A) LABA B) SABA C) Antiasthma D) Leukotriene modifier

B) SABA Explanation: Albuterol is a short-acting beta-2 agonist (SABA). It is used to treat and prevent bronchospasm

A client receives a dose of insulin lispro at 8 AM. The nurse would be alert for signs and symptoms of hypoglycemia at which time? -Between 12 noon and 8 PM -Between 10 AM and 12 noon -Between 2 PM and 4 PM -Between 8:30 AM and 9:30 AM

Between 8:30 AM and 9:30 AM

A physician prescribes a chemical stimulant laxative. Which would be appropriate?

Bisacodyl

Which of the following would the nurse expect to administer to a client with traveler's diarrhea?

Bismuth subsalicylate

What would the nurse expect to assess in a client with a whipworm infection?

Bloody diarrhea

The nurse is administering cefazolin to a client. What adverse reactions should the nurse recognize as being the most problematic for this client? (Select all that apply.) A) Diarrhea B) Vomiting C) Vaginal yeast infection D) Gastritis E) Facial edema

C) Vaginal yeast infection E) Facial edema Explanation: The most important adverse effects for the nurse to observe for in a client receiving cefazolin or any other cephalosporin are superinfections and hypersensitivity reactions demonstrated by the yeast infection and the facial edema.

The nurse knows that what physiologic changes will effect a client's stroke volume?

Changes in heart contraction Rationale: Stroke volume is the volume of blood ejected from the heart at each beat. If a heart contracts harder, more blood is ejected. If a heart cannot contract very well, then less blood is ejected and stroke volume falls. Heparin dose, walking patterns, and respiratory rate do not directly affect stroke volume.

A client is suspected of having a helminthic infection. When reviewing the client's history, what information may lead the nurse to suspect that the client has a tapeworm infection?

Consumption of sushi

A client has required multiple doses of albendazole to eradicate a helminth infection. What assessment finding should the nurse attribute to the adverse effects of treatment?

Creatinine clearance 61 mL/min/1.73m2 (1.02 mL/s/m2) (low)

What should you never do to a drug with an enteric coating?

Crush it up or open it

An adult client who experiences angina pectoris with exertion is informed by the nurse that the leading cause of angina is what? a. Smoking b. Inadequate cardiac output C. Infarction of the myocardium d. Coronary atherosclerosis

D

What client is most likely to benefit from the administration of nitroglycerin? a. A pediatric client with muscular dystrophy b. A client with peripheral vascular disease and a chronic venous ulcer C. A client with type 1 diabetes and diabetic nephropathy d. A client with chronic constipation who has developed anal fissures

D

Mr. Ashum is prescribed an albuterol inhaler as part of his treatment regimen for asthma. What is the mechanism of action for this medication? A) Albuterol causes stimulation of the bronchial tissue. B) Albuterol blocks the stimulation of the beta-2 receptors. C) Albuterol decreases vital capacity. D) Albuterol causes relaxation of the bronchial smooth muscles.

D) Albuterol causes relaxation of the bronchial smooth muscles. Explanation: The main result of albuterol binding to beta-2 receptors in the lungs is relaxation of bronchial smooth muscles. This relaxation of bronchial smooth muscle relieves bronchospasm, reduces airway resistance, facilitates mucous drainage, and increases vital capacity.

The nurse is providing education to a client diagnosed with the common cold. What measure should the nurse recommend implementing to help liquefy cold-related secretions? A) Reducing salt intake B) Deep breathing C) Gargling with normal saline D) Increasing fluid intake

D) Increasing fluid intake Explanation: Drinking plenty of water while taking guaifenesin may help loosen mucus in the lungs. Deep breathing, gargling with normal saline, and reducing salt intake do not necessarily help liquefy and mobilize secretions.

The nurse is reviewing a client's morning blood work and notes a theophylline level of 22.2 mcg/mL (123.21 µmol/L). What action should the nurse take? A) Withhold the scheduled dose of theophylline pending the next day's blood work results. B) Administer the scheduled dose of theophylline with as needed (PRN) dose of a bronchodilator. C) Inform the health care provider that an increase in the client's dose of theophylline may be necessary. D) Inform the health care provider that the client has toxic theophylline levels.

D) Inform the health care provider that the client has toxic theophylline levels. Explanation: To determine theophylline dosage, prescribers should measure serum theophylline levels. Therapeutic range is 5 to 15 mcg/mL (27.75 to 83.25 µmol/L); toxic levels are 20 mcg/mL (111 µmol/L) or above. The health care provider must be informed of this elevated serum level.

Which of the following medications is contraindicated in children? A) Amikacin B) Cefazolin C) Streptomycin D) Levofloxacin

D) Levofloxacin Explanation: Fluoroquinolones are contraindicated in clients who are younger than 18 years of age.

A patient is being treated for heart failure. Which of the following is indicative of improved heart failure status?

Decreased pedal edema

A 50-year-old client has undergone a bunionectomy and has been admitted to the postsurgical unit. What aspect of the client's medical history would contraindicate the use of heparin for deep vein thrombosis (DVT) prophylaxis?

Diagnosis of ulcerative colitis

A nurse is caring for a 26-year-old who has been diagnosed with roundworms. The client is prescribed pyrantel. What adverse effect would the nurse inform the client about?

Diarrhea

Which of the following are examples of modifiable risk factors for hyperlipidemia? (Select all that apply)

Diet Weight Modifiable risks factors for hyperlipidemia are weight, diet, and physical inactivity. Non-modifiable risk factors for hyperlipidemia are age, gender, and family history.

Which would the nurse identify as a cardiac glycoside?

Digoxin

A 35-year-old female client controls the symptoms of her cardiovascular disease with ACE inhibitors. She discovers that she is pregnant and contacts her primary physician regarding her medication regimen. What would the nurse expect the physician to do?

Discontinue the drug Rationale: The FDA has issued a black box warning for ACE inhibitors and ARBs during pregnancy, because their use can cause injury and even death to a developing fetus.

A patient has been prescribed albendazole on an outpatient basis for an anthelmintic infection. Which of the following instructions should the nurse give to the patient?

Disinfect the bathtub or shower stall immediately after bathing.

The nurse should assess for what as a symptom in a client with left-sided heart failure?

Dyspnea Symptoms of left-sided failure include dyspnea and moist cough. Edema, hepatic engorgement, and distended neck veins are symptoms of right-sided failure.

Efficient respiration depends on which factors? A) The ability of the lungs to expand and contract B) The ability of respiratory gases to cross the C) alveolar-capillary membrane D) The patency of the airway E) All the above

E) All the above Explanation: The efficiency of the respiratory system depends on the quality and quantity of air inhaled, the patency of air passages, the ability of the lungs to expand and contract, and the ability of O2 and CO2 to diffuse across the alveolar-capillary membrane.

What is the most common cause of subclinical hyperthyroidism? -excess thyroid hormone therapy -central nervous system depressant therapy -untreated osteoporosis -history of neck radiation

Excess thyroid hormone therapy

What distinguishing characteristic is associated with type 1 diabetes? -The disease always starts in childhood. -Oral agents can control blood sugar. -Blood glucose levels can be controlled by diet. -Exogenous insulin is required for life.

Exogenous insulin is required for life.

A client was diagnosed with a helminthic infection and prescribed mebendazole. The day after starting the medication, the client contacts the nurse to report two episodes of diarrhea over the past 24 hours. What is the nurse's best action?

Explain that this is likely due to the medication and instruct the client to seek care if the diarrhea worsens

There are two pathways that can be activated by the coagulation process. One pathway begins when factor XII is activated. The other pathway begins when there is trauma to a blood vessel. What are these pathways?

Extrinsic and intrinsic pathways

Which agent would a nurse expect to administer to a patient with Christmas disease?

Factor IX complex

Bone marrow depression can occur with thiabendazole.

False

A patient with hyperlipidemia is prescribed ezetimibe. Which of the following ongoing assessments should the nurse perform during treatment?

Frequently monitor blood cholesterol. The nurse should frequently monitor blood cholesterol as part of the ongoing assessment for a patient receiving ezetimibe. Taking a dietary history of the patient and inspecting the skin and eyelids for evidence of xanthomas are the pre-administration assessments that a nurse should perform for a patient receiving ezetimibe. The nurse obtains the reports of fasting blood sugar for a diabetic patient.

The nurse should instruct the client to take what action if nitroglycerin tablets taken sublingually are not effective in eliminating chest pain? a. Go to the emergency department b. Call the health care provider C. Lie down after taking an aspirin d. Take more tablets until pain subsides, to a maximum of six tablets

Go to the Emergency Department

Which of the following adverse effects is more likely to be caused by cimetidine than by other H2RAs?

Gynecomastia Rationale: Cimetidine is more likely to cause gynecomastia and mental confusion than other H2RAs.

Increased levels of low-density lipoprotein (LDL) combined with certain risk factors can lead to the development of which medical condition? (Choose one)

Heart Disease Increased levels of LDL in combination with other risk factors can lead to the development of atherosclerotic heart disease.

Define pharmacodynamics

How the drug affects the body. It studies the dose response and maximal efficacy

A client presents at the clinic reporting weight loss despite an increased appetite. For which condition should this client be assessed? -Chronic thyroiditis -Hyperglycemia -Hypothyroidism -Hyperthyroidism

Hyperthyroidism

Which adverse effect might occur in a client receiving milrinone?

Hypotension

Which would be considered a therapeutic effect of digoxin?

Increased force of contraction

A 75-year-old client is diagnosed with type 2 diabetes mellitus, hypertension, osteoporosis, and gastric ulcer disease. She is prescribed PPIs. The nurse is aware that long-term (greater than 1 year) administration of PPIs may lead to what problem for this client?

Increased risk for hip fractures Rationale: Sucralfate is well tolerated by older adults. PPIs are also well tolerated, but long-term use (greater than 1 year) is associated with increased risk of hip fractures in adults older than 50 years of age. The risk of fractures increases the longer the medications are taken. The risk of hip fractures is also greater in those taking higher dosages of PPIs.

A client is to receive abciximab. The nurse would expect to administer the drug by which route?

Intravenous

A client with constipation has been prescribed mineral oil. Which reason should the nurse provide for instructing the client to take the medication between meals or at bedtime?

It interferes with vitamin absorption

A client is prescribed pyrantel for the treatment of a roundworm infection. The nurse understands that this drug may be preferred for which reason?

It requires a single dose.

When speaking of pharmacokinetics, how could a full stomach affect absorption?

It will take longer for the medication to be absorbed

Your client appears to be a candidate for statin therapy. What laboratory value should be assessed prior to initiating therapy?

LFT Liver function tests (LFT) and serum cholesterol levels should be assessed prior to initiating statin therapy.

Once a client develops primary hypertension, therapy should last for how long? (Choose one)

Lifelong Rationale: Once primary hypertension develops, management of the disorder becomes a lifetime task.

The client's digoxin level is 0.125. How does the nurse interpret this level?

Low

The nurse is preparing to administer insulin lispro (Humalog) to a client at 7 AM. What is the nurse's priority intervention related to this medication? -Cleanse the administration site with soap and water before administration. -Have the client lie quietly for 45 minutes. -Monitor the client for a hypoglycemic reaction at noon. -Make sure the client's breakfast is available in the next 5-10 minutes.

Make sure the client's breakfast is available in the next 5-10 minutes.

A nurse is caring for a patient with diabetes mellitus who is receiving an oral antidiabetic drug. Which of following ongoing assessments should the nurse perform when caring for this patient? -Monitor the patient for lipodystrophy. -Observe the patient for hypoglycemic episodes. -Document family medical history. -Assess the skin for ulcers, cuts, and sores.

Observe the patient for hypoglycemic episodes.

A nurse is reviewing information about proton pump inhibitors. The nurse recognizes that which of the following is available as an over-the-counter agent?

Omeprazole Rationale: Omeprazole is available over the counter; lansoprazole, rabeprazole, and esomeprazole are prescription medications.

What organ provides the control over the amount of calcium in the blood? -pituitary -thyroid gland -parathyroid glands -kidneys

Parathyroid glands

After reviewing the drugs that may interfere with warfarin, the students indicate that they understand the material when they identify what as requiring a dosage increase in the warfarin?

Phenytoin

A client is receiving inamrinone. Which of the following would be most important for the nurse to monitor?

Platelet count

The pharmacology instructor is describing medications that increase the contractile force of the heart. Which term describes this effect?

Positive inotropic

A nurse is caring for a client who is taking digoxin and a loop diuretic. Which would be most important for the nurse to monitor?

Potassium levels

Which of the following is an important assessment for the nurse to make before administering a laxative?

Question the client regarding the type and intensity of symptoms to provide a baseline evaluation.

The nurse is working with an 8 year-old client who has been diagnosed with ascariasis infection following several weeks of symptoms that care providers attributed to a virus. When planning the client's care, the nurse should consider referral to what member of the care team?

Registered dietitian

The patient has Maalox ordered for administration before meals. Which of the following conditions is contraindicated with the administration of Maalox?

Renal dysfunction Rationale: Magnesium-based antacids are contraindicated in patients with renal failure. Magnesium-based antacids are not contraindicated with hypertension, regular heart rate, or stress ulcer.

After teaching a group of students about conditions that can lead to heart failure, the instructor determines that additional teaching is needed when the students identify which of the following?

Renal failure

The patient has been prescribed diphenoxylate with atropine (Lomotil) for his diarrhea. The nurse recognizes that this drug is effective against diarrhea because of which of the following actions?

Slows peristalsis by acting on the smooth muscles of the intestine

A 75-year-old client is treating his ulcer with antacids. Based on the client's age, the nurse expects the physician to prescribe a dose of antacid that compares with the average prescribed dose in what way?

Smaller than the average prescribed dose Rationale: Smaller doses of antacids may be effective in older adults, because they usually secrete less gastric acid than younger adults do.

A client with a blood pressure of 165/95 mmHg would be in classified as which stage of hypertension? (Choose one)

Stage 2 Rationale: A client is diagnosed with Stage 2 hypertension when their systolic blood pressure is greater than or equal to 160 OR their diastolic blood pressure is greater than or equal to 100.

The client has a blood pressure of 165/100 mmHg. The nurse knows that this client would be in classified as which stage of hypertension?

Stage 2 Rationale: A client is diagnosed with Stage 2 hypertension when his or her systolic blood pressure is greater than or equal to 160 mm Hg OR the diastolic blood pressure is greater than or equal to 100 mm Hg.

A client is to receive enoxaparin. The nurse would administer this drug by which route?

Subcutaneous injection

After teaching a group of students about the various methods for the delivery of insulin, the instructor determines that the teaching was successful when the students identify which method as most commonly used for administration? -Implantable infusion pump -Jet injector -Subcutaneous injection -Insulin pen

Subcutaneous injection

An older adult client has been diagnosed with osteoporosis and has begun taking alendronate. The nurse should be aware that this drug can increase bone density by which means? -decreasing renal excretion of calcium -suppressing the function of osteoclasts -increasing the binding of vitamin D to calcium ions -enhancing the function of osteoblasts

Suppressing the function of osteoclasts

A patient has had multiple blood pressure readings that indicate he is hypertensive. How is hypertension defined?

Systolic pressure above 140 mm Hg or diastolic pressure above 90 mm Hg Rationale: Hypertension is defined as a systolic pressure above 140 mm Hg or a diastolic pressure above 90 mm Hg on multiple blood pressure measurements. The blood pressure of 160/110 is considered hypertensive but does not define hypertension. The blood pressure of 130/95 is hypertensive but does not define hypertension. The blood pressure of 150/100 is hypertensive but does not define hypertension.

During a general health assessment, a client indicates taking thyroid medication to increase weight loss. What statement should be the basis of the nurse's response? -Taking thyroid medication will not contribute to weight loss but only result in a redistribution of fat deposits. -While taking thyroid medication, the client needs to take additional dietary supplements of iodine as well. -Taking excessive or unnecessary thyroid medication may produce serious or life-threatening manifestations of toxicity. -While taking thyroid medication, the client needs to take a calcium supplement as well.

Taking excessive or unnecessary thyroid medication may produce serious or life-threatening manifestations of toxicity.

Which would be most important to include when teaching a patient about using psyllium?

Taking the agent with a large amount of water

When describing the various helminthic infections, which would an instructor include as being due to a cestode?

Tapeworm

Which would be associated with an infection caused by a flatworm?

Tapeworm

A client is being sent home with subcutaneous heparin after a total hip replacement. The nurse understands what symptom would indicate a serious drug reaction?

Tarry stools

When speaking of pharmacokinetics, what happens during the first-pass effect?

The 1st pass effect goes through the liver but there isn't much absorption.

A stable daily dose of warfarin is reached when which parameter is achieved?

The PT and INR are within their therapeutic ranges, and the dose does not cause bleeding.

A client is taking cholestyramine (Questran) and ezetimibe (Zetia). What administration guideline is most important to teach this client?

The administration of ezetimibe (Zetia) is four hours after cholestyramine. Ezetimibe (Zetia) should be taken one hour before or four hours after a bile acid sequestrant. The two medications should not be taken together. Ezetimibe acts in the small intestine to inhibit absorption of cholesterol and decrease the delivery of intestinal cholesterol in the liver. Cholestyramine should not be administered one hour after ezetimibe.

The nurse has administered a subcutaneous injection of 250 mcg chorionic gonadotropin alpha. What outcome best indicates therapeutic effects? a.The client's urine output decreases and becomes more concentrated b.The client become pregnant c.The client's height increases d.The client's sexual development ceases

The client become pregnant

A nurse has administered 250 mcg ganirelix acetate subcutaneously to a client. What result indicates successful treatment? a.The client's cancer pain is relieved b.The client becomes pregnant c.The client's breast cancer goes into remission d.The client experiences relief from perimenopausal symptoms

The client becomes pregnant

The nurse is collaborating closely with the provider of a client whose helminthic infection is being treated with albendazole. What aspect of the client's pre-illness health status creates a heightened risk for adverse effects of albendazole?

The client often takes cimetidine 200 mg PO to treat dyspepsia

An adult client who recently immigrated has been diagnosed with a tapeworm after several months of weight loss. The client has been prescribed albendazole and has presented for a follow-up assessment. What finding should the nurse prioritize for communicating to the provider?

The client reports pruritis and his thorax has a yellowish tint.

A client returns for evaluation after completing a course of mebendazole therapy. There is evidence that the helminthic infection has not completely cleared. What principle will guide the client's subsequent treatment?

The client should wait three weeks before starting another course of mebendazole

The nurse monitors which client for an increased risk of digoxin toxicity?

The client with renal dysfunction

A client was been diagnosed with whipworms two weeks ago and prescribed mebendazole 100 mg PO b.i.d. for three days. What assessment finding should the nurse attribute to successful eradication of the helminth?

The client's hemoglobin and hematocrit levels have increased to within reference ranges

A 9-year-old boy was diagnosed with hookworm infection and will be sent home with a prescription for mebendazole. When providing education, the nurse should teach the mother which measures to avoid reinfection following treatment?

The importance of vigilant hygiene for the client and the other members of the family

(Pharmacokinetics): What is distribution?

The movement of a drug to the body's tissues

A 37-year-old woman with moderately elevated lipid levels requests immediate pharmacotherapy for her dyslipidemia. The nurse explains that a period of intensive diet therapy and lifestyle modification will be utilized before drug therapy is considered. The nurse explains the rationale for this regimen as:

Therapeutic lifestyle changes are the preferred method for lowering blood lipids. Nonpharmacologic interventions are always preferred to pharmacologic ones if there is potential for success. Medications are to be used only when nonpharmacologic efforts have proven unsuccessful.

Which type of infestation could result in pneumonia or liver abscesses in severe cases?

Threadworm

What is a tissue-invading worm infection caused by eating undercooked pork?

Trichinosis

Cells of a failing heart due to heart failure often lack the ability to produce the energy needed for effective contraction.

True

A client has been prescribed intravenous heparin. What laboratory value will the nurse prioritize when providing care for this client?

aPTT

Resistance

ability of pathogens over time to adapt to an antiinfective to produce cells that are no longer affected by a particular drug

What is the first step of pharmacokinetics?

absorption

A client is receiving digoxin and experiences severe bradycardia. Which medication would the nurse anticipate administering if prescribed?

atropine

What assessment should the nurse perform on a daily basis when a client is prescribed furosemide?

body weights

A nurse is caring for a client with chronic lymphocytic thyroiditis. The health care provider has prescribed liothyronine. For which condition of the client should the nurse be cautious before administering the drug? -upper respiratory tract infection -diabetes -elevated body temperature -cardiac disease

cardiac disease

A client is being treated for heart failure. Which is most indicative of improved health status?

decreased pitting edema

The nurse is preparing to administer a stimulant cathartic to the client. The nurse should inform the client of the possible adverse effect of:

diarrhea.

The provider orders heparin for a 35-year-old female client. The nurse administers the drug only after confirming that the client:

does not have peptic ulcer disease.

sensitivity testing

evaluation of pathogens obtained in a culture to determine the antiinfectives to which the organisms are sensitive and which agent would be appropriate for treatment of a particular infection

Which would a nurse identify as an example of a sulfonylurea? -Glyburide -Miglitol -Acarbose -Metformin

glyburide

Your patient has been prescribed inamrinone. You know that this drug works by

increasing cyclic adenosine monophosphate (cAMP).

Your 32-year-old patient tells you that he takes antacid tablets several times each day. Knowing that the brand of antacid he uses contains calcium carbonate, you caution the patient that overuse could place him at risk for

metabolic alkalosis. Rationale: Overuse of antacids containing calcium carbonate can cause alkalosis and raise urine pH.

A clinic nurse has been assigned a 49-year-old female patient who has a history of diabetes. A recent diagnosis of hypertension has been made, and the patient has been prescribed a thiazide diuretic and labetalol. The patient will be scheduled to return to the clinic once a month for the next 6 months. A priority action by the nurse will be to

monitor the patient's blood pressure. Rationale: Monitoring of blood pressure would be the priority assessment in the care of this patient. Questioning the patient about her dietary intake and weighing her would be appropriate, but secondary in importance. It is unnecessary to monitor the patient's respiratory rate.

The nurse should teach clients who are taking thyroid hormones to take the medication: -on an empty stomach. -at bedtime. -in the evening after dinner. -in divided doses in the morning and evening.

on an empty stomach

To maximize absorption of digoxin, the nurse would instruct the patient to take the drug:

on an empty stomach

Which medication reduces aldosterone-induced retention of sodium and water when prescribed for the treatment of heart failure?

spironolactone

selective toxicity

the ability to affect certain proteins or enzyme systems that are used by the infecting organism but not by human cells

Which of the following antacids may produce constipation and should be used cautiously in clients that have chronic constipation? Select all that apply:

• Calcium carbonate (Mylanta) • Aluminum hydroxide (ALternaGEL) Rationale: The aluminum and calcium containing antacids may produce constipation.

Which dosage forms are used for digoxin (Lanoxin) maintenance therapy? (Select all that apply.)

• Capsule • Injection • Tablet

Administration of a cardiotonic drug is contraindicated in which conditions? (Select all that apply.)

• Digitalis toxicity • Cardiac tamponade • Ventricular tachycardia

A female client is treated with antibiotics for bacterial pneumonia. When obtaining a drug history, the nurse finds that the client ceased her medication regimen when she no longer experienced symptoms during the first round of antibiotics. The nurse is responsible for the client's education plan. In order to effectively treat the pneumonia, what must the client do? A) Complete a full course of antibiotics as prescribed. B) Continue the antibiotics for 1 week after cessation of symptoms. C) Take the antibiotics at breakfast, lunch, and supper. D) Take the antibiotics every 3 hours during the day.

A) Complete a full course of antibiotics as prescribed. Explanation: Client education related to antibiotic therapy should stress the importance of completing a full course of antibiotics as prescribed.

The pulmonologist sees many patients daily who suffer from a variety of respiratory disorders. What are some of the common signs and symptoms many of these patients present? (Select all that apply.) A) Cough B) Increased temperature C) Increased secretions D) Mucosal congestion

A) Cough C) Increased secretions D) Mucosal congestion Explanation: Common signs and symptoms of respiratory disorders include cough, increased secretions, mucosal congestion, and bronchospasm. Increased temperature is not common among respiratory disorders unless infection is involved.

Amphotericin B is given by which route? A) IV B) IM C) PO D) SQ

A) IV Explanation: Amphotericin B and flucytosine are available in IV form.

A client is prescribed salmeterol. The nurse would expect this drug to be administered by which route? A) Inhalation B) Oral C) Subcutaneous D) Intravenous

A) Inhalation Explanation: Salmeterol is administered via inhalation.

What are classified as anti-infectives? (Select all that apply.) A) anthelmintics B) antifungals C) antiprotozoals D) anticonvulsants E) antimetabolites

A) anthelmintics B) antifungals C) antiprotozoals Explanation: Anti-infectives include antibiotics, antivirals, antifungals, antiprotozoals, and anthelmintic agents.

What route of administration is most commonly used for the administration of ciprofloxacin when prescribed to an older adult client? A) oral (PO) B) intravenous (IV) C) subcutaneous (SQ) D) intramuscular (IM)

A) oral (PO) Explanation: While ciprofloxacin is typically administered by the PO or IV route, in most cases, oral administration is used. The medication is not administered by either the SQ or IM routes.

Students learning about antifungal medication are taught that what medications will increase the effects of fluconazole? A) warfarin B) diphenhydramine C) acetaminophen D) hydrochlorothiazide

A) warfarin Explanation: Ketoconazole and fluconazole strongly inhibit the cytochrome P450 (CYP450) enzyme system in the liver and are associated with many drug-drug interactions such as increased serum levels of the following agents: cyclosporine, digoxin, oral hypoglycemics, warfarin, oral anticoagulants, and phenytoin

The nurse teaches the client how to use topical nitroglycerin and includes what teaching points in the teaching plan? a. "Rotate application sites." b. "Assess the skin for signs of breakdown." c. "Make sure no öne touches the side with the medication." d. "Do not shower with the patch in place." e. "Increase fluid intake to avoid low blood pressure."

A, B, C

A nurse is providing client education to a 13-year-old girl who was just diagnosed with type 1 diabetes mellitus. Which statement by the client will alert the nurse that special instructions regarding insulin are necessary? -"We live in a two-story house." -"I walk two blocks to school every day." -"My mother is going to give me my insulin." -"I am on the middle school track team."

"I am on the middle school track team."

The nurse is teaching a patient who is prescribed calcitriol about the drug. Which patient statement indicates that the teaching was successful? -"I must take the drug on an empty stomach." -"I need to limit the amount of dairy products that I eat." -"I need to have my calcium levels checked periodically." -"I need to use an antacid that contains magnesium to prevent problems."

"I need to have my calcium levels checked periodically."

The nurse is teaching a patient who is prescribed calcitriol about the drug. Which patient statement indicates that the teaching was successful? -"I need to limit the amount of dairy products that I eat." -"I need to use an antacid that contains magnesium to prevent problems." -"I must take the drug on an empty stomach." -"I need to have my calcium levels checked periodically."

"I need to have my calcium levels checked periodically."

The nurse is discussing diabetes with a group of individuals who are at risk for the disease. Which statement by a participant indicates an understanding of the role of insulin in the disease? -"Insulin assists glucose molecules to enter the cells of muscle and fat tissues." -"Insulin causes fat to be broken down to provide energy for the body." -"Insulin is stimulated by the liver to break down proteins and provide the body with nutrients." -"Insulin is used to move carbohydrate particles from the gastrointestinal system to the liver."

"Insulin assists glucose molecules to enter the cells of muscle and fat tissues."

A college student presented to the medical center believing that she may have a sexually transmitted infection. The nurse is referring the client to the provider and suspects that the client may be diagnosed with a helminthic infection instead. What assessment question should the nurse prioritize?

"Is there any chance that you might be pregnant?"

A client's gastrointestinal symptoms have been attributed to a helminth infection and the provider has prescribed praziquantel. When reviewing the prescription with the client, what instructions should the nurse most likely provide?

"Make sure to take all three doses that you've been prescribed."

Digoxin has been prescribed for an outclient with symptomatic heart failure. What is the priority teaching point to convey to this client?

"Make sure to take your pulse for a minute before taking your digoxin." Clients should measure their heart rate for a full minute before taking a dose of digoxin. The drug can be taken with food, and potassium intake should be increased, not decreased. The client's daily dose of digoxin should never be increased in response to short-term changes in symptoms.

A patient has GERD and is taking ranitidine (Zantac). She continues to have gastric discomfort and asks whether she can take an antacid. Which of the following is an appropriate response by the nurse?

"Yes, but be sure to wait at least 2 hours to take the antacid after you take the ranitidine." Rationale: If both ranitidine and antacids are prescribed, give them at least 2 hours apart to prevent decreased absorption of ranitidine. The nurse should advise the patient to wait at least 2 hours between doses and to take the antacid after the ranitidine. The other three statements are not true.

A nurse is providing education to a client who will soon begin taking levothyroxine for the first time. Which teaching point should the nurse include in this education session? -"You'll need to adhere to a strict diet with high protein and low carbohydrates." -"I'll help you arrange to come to an outpatient clinic for IV administration of levothyroxine." -"You'll most likely take this drug for the rest of your life." -"I'll teach you how to check your blood sugar levels for the first few weeks that you take the drug."

"You'll most likely take this drug for the rest of your life."

A female client is a newly diagnosed diabetic. She is a stay-at-home mother and responsible for meal planning and management of the home. What will the home care nurse teach this client? (Select all that apply.) -Reinforce instructions on dealing with hypoglycemia. -Instruct the client to go to the emergency department immediately if she develops a cold or upper respiratory infection. -Assist the client in making menus that will meet the needs of both the client and the family. -Watch the client draw up and administer her insulin. -Encourage the client to check her blood glucose every hour.

-Assist the client in making menus that will meet the needs of both the client and the family. -Watch the client draw up and administer her insulin. -Reinforce instructions on dealing with hypoglycemia.

The nurse is providing discharge instructions to a client who has just been diagnosed with type 1 diabetes. What instructions are most important for the client to follow related to diet? (Select all that apply.) -Lose 10-15 pounds. -If you skip a meal, increase your next insulin dosage. -Use artificial sweeteners instead of sugar in tea and coffee. -Read food labels carefully to look for hidden sources of sugar. -Avoid drinking beer, wine, or liquor.

-Avoid drinking beer, wine, or liquor. -Use artificial sweeteners instead of sugar in tea and coffee. -Read food labels carefully to look for hidden sources of sugar.

Which should be included in client teaching as a means of avoiding constipation? (Select all that apply.)

-Be active and exercise daily. -Eat foods high in bulk or roughage. -Drink plenty of fluids.

The nurse is providing teaching to a client with type 1 diabetes. The client requires additional teaching when he learns that his insulin needs may increase because of what condition? (Select all that apply.) -Fever -Anxiety -Stress -Decreased food intake -Exercise

-Decreased food intake -Exercise

Thyroid hormones are principally concerned with the increase in metabolic rate of tissues that can result in certain effects? What are some of these effects? (Select all that apply.) -Decreased oxygen consumption -Decreased respiratory rate -Increased heart rate -Increased cardiac output -Increased body temperature

-Increase Heart Rate -Increase Temperature -Increase Cardiac Output

What happens during the pharamaceutic phase? What would happen when a drug in is it's enteric coating?

-Solid drug is taken orally and is broken down into liquid state. -only certain pieces will be released at a time.

Clients on digoxin need to have their digoxin level monitored. The nurse identifies the therapeutic range for digoxin as which of the following?

0.8 to 2 ng/mL

A patient is in the intensive care unit and being administered ranitidine (Zantac) parenterally. How long will it take for ranitidine (Zantac) to reach peak blood levels?

15 minutes Rationale: Parenteral ranitidine reaches peak blood level in 15 minutes. Parenteral ranitidine reaches its peak prior to 30 minutes, 1 hour, and 2 hours.

The nurse receives an order to administer leuprolide 5 mcg/kg subcutaneously to a child with precocious puberty. The child weighs 66 lbs. What is the correct dosage for this child? Provide your answer using numbers only - no units.

150 Feedback: To begin, convert the child's weight to kilograms by dividing by 2.2: 66 ÷ 2.2 = 30. Multiply the child's weight in kilograms times the mg/kg: 30 × 5 = 150 mcg/dose.

A nurse is administering digoxin intravenously as ordered. The nurse would administer the drug over which time frame?

5 minutes

A client is started on enoxaparin immediately after hip surgery. The nurse would explain to the client that this drug will be continued for how long?

7 to 10 days

A client is to receive diltiazem 360 mg/dose orally in four divided doses. How many mg should the nurse administer per dose? Provide your answer using numbers only - no units.

90 Feedback: The patient will take 90 mg/dose (360 divided by 4 doses equals 90 mg/dose).

A client receiving heparin has a baseline aPTT of 28 seconds. Which result would the nurse interpret as suggesting too much heparin?

98 seconds

A client has been diagnosed with angina and prescribed nitroglycerin. What aspect of the client's health history should prompt the nurse to monitor the client particularly closely after administration of the drug? a. Orthostatic hypotension b. Depression and use of a selective serotonin reuptake inhibitor C. Reynaud's syndrome d. Gastroesophageal reflux disease (GERD)

A

In which client would the use of a beta-blocker most likely be contraindicated? a. A client with angina pectoris who is trying to become pregnant b. A client who takes a bisphosphonate for osteoporosis and who has angina C. A client with angina who recently completed a course of moxifloxacin for pneumonia d. A client who has angina and diabetic retinopathy

A

The nurse is caring for a client diagnosed with human immunodeficiency virus (HIV) and newly diagnosed angina. What drug would the nurse question if ordered? a. Ranolazine b. Nitroglycerin C. Propranolol d. Diltiazem

A

The nurse is caring for a client who is taking a sustained-release (SR) oral nitrate. What should the nurse instruct this client to how to take the medication? a. With water b. Sublingually until absorbed c. With milk or milk products d. 1 hour after eating

A

The nurse is providing emergency health care at a refugee camp and there are several clients who are being treated for helminthic infections. What client should the nurse prioritize for referral to one of the camp care providers?

A client who is taking albendazole and whose urine output has decreased sharply over the past 24 hours.

A patient with hypertension has been started on losartan, an angiotensin II receptor blocker (ARB). After 6 weeks of therapy, the physician concludes that losartan alone is not adequately controlling the hypertension. What would the physician likely add to the regimen to improve control?

A diuretic Rationale: When losartan therapy is started, maximal effects on blood pressure usually occur within 3 to 6 weeks. If losartan alone does not control blood pressure, a low dose of a diuretic may be added. A combination product of losartan and hydrochlorothiazide is available. (less)

Explain drug-to-food interactions

A patient should avoid certain foods because it can interact with medication

After administering somatropin to an 11-year-old client with growth failure, what outcome would indicate that the drug should be stopped? a.Early sexual development b.Thyroid overactivity c.Closure of the epiphyses in long bones d.Gynecomastia

Ans: C Feedback: Closure of the epiphyses is a sign that the drug should be stopped. Early sexual development, thyroid overactivity, and gynecomastia would not be associated with this drug.

Which would be tested to determine parasitic sources of infection? A) Blood B) Stool C) Sputum D) Sweat

B) Stool Explanation: When investigators search for parasitic sources of infection, they may examine stool for ova and parasites.

Which cells are responsible for producing surfactant? A) Basal cells B) Type 2 cells C) T cells D) Squamous cells

B) Type 2 cells Explanation: The type 2 cells of the alveoli produce surfactant, which reduces surface tension to keep the alveoli open for gas exchange. Basal cells, T cells, and squamous cells relate to the integumentary system and are potential origins of skin cancer.

Which cranial nerve is responsible for stimulating the diaphragm to contract and relax? A) Accessory B) Vagus C) Trigeminal D) Hypoglossal

B) Vagus Explanation: The vagus nerve plays a key role in stimulating diaphragm contraction and inspiration.

What helps regulate the rate and depth of respiration? A) Cerebrum B) Cerebellum C) Medulla oblongata D) Medulla ossea

C) Medulla oblongata Explanation: The rate and depth of respiration are regulated by the respiratory center in the medulla oblongata, the pneumotaxic center in the pons, and the apneustic center in the reticular formation.

A female client calls the pediatrician's office because her child reports a sore throat. The nurse anticipates the pediatrician will take what action? A) Order an antibiotic. B) Order an antitussive. C) Request a list of drug allergies. D) Request a throat culture.

D) Request a throat culture. Explanation: For sore throat, a throat culture for streptococcal organisms should be performed and the results obtained before an antibiotic is prescribed.

What allows the alveoli sac to stay open and allows for exchange of gases? A) Adenosine triphosphate (ATP) B) Histamine C) Serotonin D) Surfactant

D) Surfactant Explanation: Surfactant is a lipoprotein that decreases the surface tension in the sac and prevents alveolar collapse. ATP, histamine, and serotonin are released from mast cells throughout the airway to ensure a quick and intense inflammatory reaction to any cell injury, which may lead to a respiratory infection.

Define halftime?

Halftime is the time it takes for the amount of a drug in the body to decrease to one-half the peak level.

The nurse is caring for a patient who is on thrombolytic therapy. The patient has a co-morbidity of renal insufficiency. What does the nurse know about the amount of heparin required to treat this patient?

Lower doses are required.

The effects of warfarin (Coumadin) are monitored by what laboratory test(s)?

PT and INR

A student has presented to the college campus medical clinic and has been diagnosed with a helminthic infection. When planning this client's care, the nurse should include interventions to address what psychosocial nursing diagnosis?

Situational low self-esteem

A patient with type 1 diabetes has come to the clinic for a routine follow-up appointment. While assessing the patient's skin, the nurse observes brown spots on his lower legs. What might these spots indicate? -Impaired glucose tolerance -Infection -Poor insulin injection technique -Widespread changes in the blood vessels

Widespread changes in the blood vessels

A nurse assessing a client on digoxin suspects toxicity. Which visual disturbances would the nurse expect to assess?

Yellow or green vision

To compensate for clopidogrel's slow onset of action, the nurse should anticipate implementing which intervention?

administering a one-time loading dose that is higher than the maintenance dose

Prophylaxis

treatment to prevent an infection before it occurs, as in the use of antibiotics to prevent bacterial endocarditis in high-risk patients or antiprotozoals to prevent malaria

You are teaching your 47-year-old client about omeprazole, which the physician has prescribed as part of a regimen to treat an H. pylori infection. What statement, made by the client, suggests that he understands proper use of the drug?

"I need to swallow the drug whole." Rationale: Omeprazole, a proton pump inhibitor, must be swallowed whole; it should not be cut, crushed or chewed. The drug should be taken approximately an hour before a meal, not after a meal. Concomitant use of antacids is common, though the physician may instruct the patient to take the two medications at different times during the day.

A client is receiving low-molecular-weight heparin (LMWH) to prevent thromboembolic complications. The nursing student asks the instructor the reason why this treatment is given instead of heparin. What is the instructor's best explanation of the rationale for LMWH over heparin?

"LMWH is associated with less thrombocytopenia than standard heparin."

The nurse is teaching the parents of a child with pinworms about the prescribed mebendazole therapy. Which statement by the parents indicates successful teaching?

"Our son should chew the tablet thoroughly each morning and evening for 3 days in a row."

What statement would be appropriately included in the teaching concerning type 2 diabetes? -"If you drink alcohol, it may be necessary for you to increase your oral antidiabetic medication." -"Until you need to start insulin injections, you do not have to check your blood sugar." -"Regular exercise makes your body better able to use the insulin it produces." -"Clients with type 2 diabetes always progress to insulin injections if they do not follow dietary guidelines."

"Regular exercise makes your body better able to use the insulin it produces."

The nurse is preparing to administer mebendazole for a client with a helminthic infection. A colleague asks the nurse, "Is that safe? I know that this client has diabetic nephropathy." What is the nurse's most appropriate response?

"The drug's excreted in the feces, so impaired kidney function won't affect excretion."

Which of the following questions would be most important for a nurse to ask a female patient who is starting on an angiotensin II receptor blocker for hypertension?

"When was your last menstrual period?" Rationale: It would be important to know when the patient's LMP occurred and that the patient is not pregnant. These drugs are category C for the first trimester and category D for the second and third trimesters of pregnancy and should not be used in pregnancy unless the benefits outweigh the risks. The other questions provide good assessment information but would provide no specific insights regarding the patient who is beginning angiotensin II receptor blocker therapy.

A young child has been brought to the clinic with signs and symptoms that are consistent with otitis externa. What assessment question should the nurse ask to address the etiology of this health problem? A) "Has your child been swimming a lot lately?" B) "How would you describe your child's overall level of hygiene?" C) "Are all of your child's immunizations up-to-date?" D) "Has your child been spending quite a bit of time around animals lately?"

A) "Has your child been swimming a lot lately?" Explanation: Otitis externa in children is often attributable to moisture in the ear, often because of swimming. Poor hygiene, interaction with animals, and lack of immunizations are not likely to be relevant factors.

A primary health care provider has prescribed levothyroxine to a client with hypothyroidism. Which information would the nurse include in the teaching plan to promote an optimal response to the drug therapy? -Administer the drug early in the morning before breakfast. -Bring your recorded pulse rates to the primary health care provider. -Offer suggestions about the dosage schedule. -Apply soothing creams or lubricants to prevent rashes.

Administer he drug early in the morning before breakfast

A client taking methimazole develops a rash. What is the nurse's best recommendation to treat the rash? -Use an antibacterial soap when bathing. -Avoid topical lubricants. -Wear gloves to prevent scratching and subsequent skin damage. -Apply a soothing cream until the rash subsides.

Apply a soothing cream until the rash subsides

The nurse is caring for a client who receives monthly injections of goserelin. What assessment should the nurse perform when monitoring the client for therapeutic effects? a.Assess the character and severity of the client's cancer pain b.Assess the client's height and plot it against age-based percentiles c.Monitor the client's quantity and distribution of body fat d.Monitor the client's level of consciousness and orientation

Assess the character and severity of the client's cancer pain

An antacid has been added to the treatment regimen for a client on multiple medications. When is the best time for the nurse to administer the antacid?

At bedtime Rationale: Antacids are best administered one to two hours after meals and at bedtime. No oral drugs should be administered within one to two hours of an antacid.

Some recent anti-infective agents have been developed that are completely free of adverse effects. A) True B) False

B) False Explanation: No anti-infective agent has yet to be developed that is completely free of adverse effects.

A teenager is admitted to a health care facility for a fungal infection. It has been determined that the infection was present for a long time, but there was no treatment undertaken. The teenager now has a systemic fungal infection for which flucytosine is prescribed. Which would be most important for the nurse to assess before beginning therapy? A) oral mucosa B) weight C) vaginal discharge D) hydration status

B) weight Explanation: With flucytosine, it is important for the nurse to weigh the teenager because the dosage of the drug is determined by the weight. The teenager has a systemic fungal infection, so inspection of the mouth or assessment of vaginal discharge would not be important. Although it would be important to assess the teenager's hydration status, this is not as important as obtaining the weight.

The nurse should prioritize what assessment when caring for a client receiving albendazole therapy?

Blood urea nitrogen and creatinine levels

When an anti-infective's function is to destroy the infective pathogen, it is classified as: A) resistant. B) bacteriostatic. C) bactericidal. D) sensitive.

C) bactericidal. Explanation: Anti-infectives can act to destroy an infective pathogen (bactericidal) or to prevent the pathogen from reproducing (bacteriostatic). Resistance is the ability over time to adapt to an anti-infective drug and produce cells that are no longer affected by a particular drug. Culture and sensitivity testing identify the causal pathogen and the most appropriate drug for treating the infection.

The nurse is assessing a pediatric client who has been diagnosed with pinworms. The client's parent states, "The health care provider prescribed some pills that I have to give my child for three weeks. It will be hard to remember to give them every day." What is the nurse's best action?

Confirm the length of the course of treatment with the provider.

The home care nurse sees a patient for the first time. The patient has crackles in the lower lobes of the lungs, an audible S3, and pitting edema in the feet and ankles. What condition is the patient experiencing?

Congestive heart failure

A 29-year-old woman has been prescribed alosetron (Lotronex) for irritable bowel syndrome. Before starting the drug therapy, the nurse will advise the patient about which of the following adverse effect(s)?

Constipation

Recommended treatments for patients with dyslipidemia are made according to their blood levels of total and LDL cholesterol and risk factors for cardiovascular disease. What does the impact of existing cardiovascular disease have on recommendations?

Decreases target serum LDL level Existing cardiovascular disease lowers the target LDL serum level in these patients.

The nurse is caring for a client with neurogenic diabetes insipidus and administers what drug to treat the condition? a.Dexamethasone b.Desmopressin c.Methylprednisolone d.Physostigmine

Desmopressin (DDAVP, Stimate) Feedback: Synthetic preparations of antidiuretic hormone (ADH), which are purer and have fewer adverse effects, are used to treat diabetes insipidus. Only one ADH preparation is currently available, desmopressin. Dexamethasone, methylprednisolone, and physostigmine would not be indicated for treatment of this disorder.

Your client is being treated for hypokalemia. His current medications include Inderal, digoxin, and Coumadin. He is complaining about nausea, abdominal discomfort, and visual changes. Which of the following problems would you suspect?

Digitalis toxicity

The nurse is educating a client who will be adding an injection of pramlintide to his insulin regimen. What information is most important for the nurse to share with this client to ensure safe medication administration? -Do not give pramlintide in the same site where insulin is administered. -Pramlintide should only be injected in the hip. -Inject pramlintide in the same site where insulin is administered. -Mix pramlintide in the same syringe with insulin.

Do not give pramlintide in the same site where insulin is administered

The nurse may advise a patient to take what after a myocardial infarction to prevent straining during defecation?

Docusate (Colace)

When describing the drugs used to treat traveler's diarrhea, which would the nurse identify as the most common cause?

E. coli

True or False: More patients experience hyperthyroidism than hypothyroidism.

False

True or False: Pramlintide is administered orally.

False

True or False: Stress typically results in hypoglycemia.

False

True or false: oral liquid is absorbed quicker than IV liquid

False

Heavy intake of which herb should be avoided by a client who is prescribed an antidiabetic medication? -oregano -basil -garlic -anise

Garlic

What medication would the nurse expect the provider to prescribe as palliative treatment for advanced prostate cancer? a.Histrelin b.Ganirelix c.Nafarelin d.Somatropin

Histrelin (Vantas) Feedback: Histrelin (Vantas) is used to provide palliative treatment for advanced prostate cancer. Nafarelin (Synarel) is used for treatment of endometriosis and precocious puberty; Ganirelix (Antagon) is used for inhibition of premature luteinizing hormone surge in women undergoing controlled ovarian stimulation as part of a fertility program; Somatropin (Nutropin) is used for treatment of children with growth failure due to lack of growth hormone or to chronic renal failure.

A client is receiving a diuretic as the first-line treatment of mild hypertension. The nurse monitors the client for signs and symptoms of hypokalemia with which agent?

Hydrochlorothiazide Rationale: Hydrochlorothiazide is a thiazide diuretic that promotes the loss of sodium as well as potassium from the body. Subsequently, the client is at risk for hypokalemia. Amiloride, spironolactone, and triamterene are potassium-sparing diuretics. The client using these diuretics would need to be monitored for hyperkalemia because potassium is not lost along with sodium.

A middle-aged client is proactive with personal health maintenance and reports taking 2400 International Units of vitamin D daily, stating, "More is better when it comes to vitamins." The nurse should provide health teaching to the client associated with a risk for which disorder? -hyperphosphatemia -tetany -hypercalcemia -hypocalcemia

Hypercalcemia

Mr. Hon has been admitted to the hospital and placed on anticoagulant therapy. For what blood-related disorder is this therapy used?

Hypercoagulation

The nurse is caring for a client who is taking glyburide as treatment for type 2 diabetes mellitus. The health care provider has added a corticosteroid to this client's medication regimen for treatment of a severe allergic reaction. The nurse knows that this drug combination may cause what adverse effect on this client? -hyperglycemia -hypoglycemia -rash and fever -nausea and vomiting

Hyperglycemia

A patient with diabetes mellitus is prescribed captopril (Capoten) to treat hypertension. What electrolyte imbalance might the patient be prone to develop?

Hyperkalemia Rationale: Hyperkalemia may develop in patients who have diabetes mellitus or renal impairment. Hypokalemia is not a risk in patients with diabetes mellitus. Hypernatremia is not risk in patients with diabetes who are taking captopril. Hypermagnesemia is not a risk in patients with diabetes who are taking captopril.

A community health nurse is providing a health promotion session to a group of seniors. Which of the following is most important to instruct regarding hypertension?

Hypertension will increase the risk of heart disease. Rationale: Hypertension increases risks of myocardial infarction, heart failure, cerebral infarction and hemorrhage, and renal disease. Hypertension does not increase the risk of cancer. An increase in sodium in a patient's diet is not recommended with hypertension. Exercise will decrease the risk of hypertension.

The pharmacology instructor is discussing cardiac glycosides with a class of pre-nursing students. According to the instructor, what physiologic effect do cardiac glycosides trigger?

Increased force of heart contraction

The nurse explains to the client that with heart failure, what may occur?

Increased heart rate

When administering insulin, what would be most appropriate? -Shake the vial vigorously to ensure thorough mixing before drawing up the dose. -Firmly spread the skin of the area of the intended site of injection. -Massage the injection site firmly after removing the needle and syringe. -Insert the needle at a 45-degree angle for injection.

Insert the needle at a 45-degree angle for injection

During a cardiac catheterization and concurrent GP IIb/IIIa receptor antagonist administration, a client begins to bleed from the arterial access site of the catheterization. In what, if any, specific circumstance can the infusion continue?

It can continue if bleeding can be controlled by pressure application.

The nurse is to administer digoxin to a client with heart failure. The nurse auscultates an apical pulse rate of 52. What action should the nurse take?

Notify the physician

A child has been brought to the ambulatory clinic with signs and symptoms that suggest a helminth infection. When planning the care of the client and family, the nurse should perform what action first?

Obtain a stool sample for ova and parasites

A 48-year-old client with acromegaly is not a candidate for other therapy. What medication, administered subcutaneously, would the nurse caring for the client expect the physician to order? a.Gonadorelin hydrochloride b.Octreotide c.Nafarelin d.Gonadorelin acetate

Octreotide (Sandostatin) Feedback: Octreotide (Sandostatin) is a treatment for acromegaly in adults who are not candidates for, or cannot tolerate, other therapy. Gonadorelin hydrochloride, nafarelin, and gonadorelin acetate are not indicated for treating acromegaly.

After teaching a group of students about proton pump inhibitors, the instructor determines that the students have understood the information when they identify which agent as the prototype proton pump inhibitor?

Omeprazole Rationale: Omeprazole is considered the prototype proton pump inhibitor.

A female client presents to the physician's office with increasing stomach acidity. She self-administers calcium antacids. She notes that she seems to be having more issues with stomach acid, so she has been taking the calcium antacids more frequently. The nurse suspects that this may have caused what to occur in this client?

Rebound acidity Rationale: Calcium antacids have high neutralizing capacity and rapid onset. They may cause rebound acidity and hypercalcemia.

A 32-year-old female executive comes to the clinic with severe heartburn. The physician prescribes omeprazole (Prilosec). The patient asks the nurse when the pain will go away. What is an appropriate response by the nurse?

Recommended time for therapy is usually between 2 and 8 weeks. Rationale: Even though heartburn pain will decrease within a couple days of therapy and some patients are on long-term therapy for as long as 5 years, the most important response by the nurse should include that the patient should take the medication at the recommended time. Optimal efficacy requires that omeprazole is taken daily, not just when the patient is symptomatic. The nurse must stress that the patient take the medication as recommended, which could be between 2 and 8 weeks. Usually, it takes longer than a few minutes after the first dose for the pain to subside.

Extractions is the _______ of drugs from the body.

Removal

What should the nurse suggest to assist a client to improve his cholesterol levels?

Smoking cessation Lifestyle changes that can help improve cholesterol levels include a low-fat diet, regular aerobic exercise, losing weight, and not smoking.

What drug would the nurse expect to administer to a client with acquired immunodeficiency syndrome who has developed cachexia? a.Bromocriptine b.Somatropin c.Desmopressin d.Leuprolide

Somatropin (Saizen) Feedback: Indications for somatropin therapy include cachexia, long-term treatment of children with growth failure associated with various deficiencies, girls with Turner's syndrome, AIDS-related wasting, growth hormone deficiency in adults, and treatment of growth failure in children of small gestational age who do not achieve catch-up growth by 2 years of age. Bromocriptine mesylate is indicated for the treatment of Parkinson's disease, hyperprolactinemia associated with pituitary adenomas, female infertility associated with hyperprolactinemia, and acromegaly; and short-term treatment of amenorrhea or galactorrhea. Desmopressin is indicated for the treatment of neurogenic diabetes insipidus and hemophilia A. Leuprolide is used as an antineoplastic agent for treatment of specific cancers.

A client who is receiving a strong iodide solution, potassium iodide, develops iodism. What would the nurse expect to find? -Sore teeth -Sweet taste in the mouth -Constipation -Rash

Sore Teeth

A patient at a health care facility has been prescribed diazoxide for hypoglycemia due to hyperinsulinism. What adverse reactions to the drug should the nurse monitor for in the patient? -Epigastric discomfort -Tachycardia -Myalgia -Flatulence

Tachycardia

The nurse should monitor a client for which effect of a drug interaction in a client taking thyroid hormones and a beta blocker? -Hypoglycemia -Visual changes -Tachycardia -Prolonged bleeding

Tachycardia

A nurse is working with a newly diagnosed diabetic client on understanding hypoglycemia and insulin reactions. Which action would be most important for the client to understand when planning the response to an insulin reaction? -Take an oral dose of some form of glucose as soon as possible. -Inject a prescribed dose of insulin as soon as you suspect the reaction is occurring. -Stay calm and still until the reaction subsides. -Notify your health care provider immediately.

Take an oral dose of some form of glucose as soon as possible.

A patient with congestive heart failure has been digitalized. The patient requires long-term digoxin therapy. Which instructions should the nurse provide the patient on discharge?

Take the drug regularly without skipping a dose.

The client is diagnosed with hepatitis A, diabetes type 1, and portal hypertension. The client develops a DVT, and the health care provider prescribes warfarin. The nurse is concerned for what reason?

The client is more likely to experience bleeding.

A 30-year-old missionary who works in a developing country has been diagnosed with a helminthic infection. This client will be prescribed an anthelmintic. and the nurse should consequently provide what education?

The need to use barrier contraception during treatment

A client with diabetes is undergoing testing for glycosylated hemoglobin. The nurse instructs the client that this test measures average blood glucose over what time period? -the past 7 to 10 days -the past 1 or 2 months -the past 12 to 24 hours -the past 3 or 4 months

The past 3 or 4 months

A triage nurse in the emergency department suspects that a 78-year-old patient is experiencing severe digoxin toxicity with significant cardiac dysrhythmia. If she is correct, which of the following actions is likely to be taken?

The patient will be given digoxin immune fab.

A triage nurse in the emergency department suspects that a 78-year-old patient is experiencing severe digoxin toxicity with significant cardiac dysrhythmia. If the nurse is correct, which action is likely to be taken?

The patient will be given digoxin immune fab.

Mr. Hall, age 65, is admitted to the hospital for pneumonia. Since admission he has not had a bowel movement. The physician prescribes magnesium hydroxide. What would be a realistic outcome of this therapy?

The patient will have a bowel movement after taking the drug.

When speaking of pharmacokinetics, what is bioavailability?

The percentage of medication that enters the circulation and is unchanged.

What can happen when two or more drugs/ substances are taken together?

There's a possibility that an altercation can occur, causing unanticipated effects in the body.

What helminths should the nurse classify as nematodes? Select all that apply.

Threadworm Whipworm Pinworm

Which would the nurse identify as the end of the intrinsic pathway?

Thrombin formation

Pinworms are a type of nematode. Which are characteristics associated with pinworms? Select all that apply.

Transmitted when the worm eggs are ingested, by touch or inhalation. Cause perianal itching. Represent the most common helminthic infection among school-aged children.

An older adult who lives in a long-term care facility has recently begun taking losartan (Cozaar) for the treatment of hypertension. The nurse who provides care for this resident should recognize that this change in the resident's medication regimen make create a risk for

falls. Rationale: Angiotensin II receptor blockers such as losartan are associated with a risk of dizziness and a consequent risk for falls. This risk is more pronounced among older adults. Losartan is not associated with constipation, xerostomia, or depression.

What is the single most important site for biotransformation?

liver

Cardiotonic drugs are used to treat which medical condition? (Select all that apply)

• Heart failure • Atrial fibrillation

The administration of cholestyramine (Questran) with warfarin (Coumadin) would most likely cause a client to present with which of the following? Select all that apply:

• Subtherapeutic INR • Calf pain and warmth Co-administration of warfarin and cholestyramine can result in decreased anticoagulant effect leading to subtherapeutic INR and increase chance of clotting (signs and symptoms of DVT or PE).

A nurse is required to monitor a client for right ventricular dysfunction. Which would the nurse most commonly assess?

Pitting edema

Culture

sample of the bacteria (e.g., from sputum, cell scrapings, urine) to be grown in a laboratory to determine the species of bacteria that causes an infection

A nurse is discussing helminthic infections with the nursing students. How would the nurse explain the action of anthelmintic drugs?

Act on metabolic pathways that are present in the invading worm.

When administering a bulk laxative to a client, which action is important for the nurse to take?

Administer the laxative with a full glass of water or juice.

A nurse is required to monitor a patient for right ventricular dysfunction. Which of the following is one of the most common symptoms associated with right ventricular dysfunction?

Pitting edema

A school-aged child has been prescribed albendazole 15 mg/kg/day PO in divided doses, b.i.d. The child weighs 66 lbs and is 3'11" tall. How many mg of albendazole should the nurse administer for each dose of treatment?

225

An adult client with a helminthic infection has been prescribed praziquantel 20 mg/kg PO t.i.d. The client weighs 198 lbs. How many 600 mg tablets should the nurse administer for each dose?

3

The nurse understands that the drug docusate sodium (Colace) would be most appropriate for what client?

A 66-year-old client who is recovering from a heart attack

A nurse is preparing to administer a client's prescribed anthelmintic medication. What diagnostic results should provide the rationale for the team's choice of medication?

Stool for ova and parasites

A client who has been treated with warfarin after cardiac surgery is found to have an INR of 9.0. Which medication will be administered to assist in the development of clotting factors?

Vitamin K

A client who is receiving warfarin has blood in the urinary drainage bag. What medication will likely be prescribed by the health care provider?

Vitamin K

A client's current condition requires rapid reduction of blood sugar levels. Which type of insulin will have the most rapid onset of action? -Humulin R -insulin lispro -isophane (NPH) -isophane (NPH)

insulin lispro

Bactericidal

substance that causes the death of bacteria, usually by interfering with cell membrane stability or with proteins or enzymes necessary to maintain the cellular integrity of the bacteria

Bacteriostatic

substance that prevents the replication of bacteria, usually by interfering with proteins or enzyme systems necessary for reproduction of the bacteria

A parent asks the nurse how mebendazole will eradicate a child's parasitic worms. How should the nurse best respond to the parent?

"Mebendazole robs the worms of the glucose they need to survive."

A nurse is planning an in-service program for a group of staff nurses about heart failure and its treatment. The nurse would identify which agent as the most commonly used drug for treatment?

Digoxin

Mr. Tan is a 69-year-old man who prides himself in maintaining an active lifestyle and a healthy diet that includes adequate fluid intake. However, Mr. Tan states that he has experienced occasional constipation in recent months. What remedy should be the nurse's first suggestion?

A bulk-forming (fiber) laxative

Explain drug-drug interactions

A patient may be taking 2 medications (cumulative effect) and could cause 2 opposite actions because they are fighting each other

Explain drug-to-lab tests

A patient should be cautious of what medications he/she takes, since some can alter lab results

While discussing the use of laxatives a student, asks the nursing instructor what the best indication of normal bowel elimination is. What would be the best response by the instructor?

"A soft, formed stool."

The nurse is instructing a client how to take a prescribed pramlintide. Which would be most appropriate? -"Give yourself an injection 1 hour before you eat breakfast and dinner." -"Take the drug orally once a day, preferably in the morning." -"Take the drug orally with the first bite of each meal." -"Give it by subcutaneous injection immediately before your major meals."

"Give it by subcutaneous injection immediately before your major meals."

A 60-year-old man has scheduled a follow-up appointment with his primary care provider stating that the omeprazole (Prilosec) which he was recently prescribed is ineffective. The patient states, "I take it as soon as I feel heartburn coming on, but it doesn't seem to help at all." How should the nurse best respond to this patient's statement?

"Prilosec will help your heartburn but it's not designed to provide immediate relief of specific episodes of heartburn." Rationale: Optimal efficacy requires that omeprazole is taken daily, not just when the patient is symptomatic. A lack of immediate symptomatic relief does not necessarily indicate that the drug is ineffective.

A client who has been receiving verapamil for several months comes to the clinic reporting significant dizziness, lightheadedness, and fatigue. He also reports frequent episodes of nausea and swelling of his ankles. Drug toxicity is suspected. Which question would be critical to ask the client?

"Have you been drinking any grapefruit juice lately?" Rationale: Verapamil, like other calcium channel blockers, interacts with grapefruit juice, increasing the concentration of calcium channel blockers and leading to toxicity. Calcium channel blockers do not interact with ibuprofen. Splitting or crushing the pills could lead to a release of the drug all at once, but this is more common when the drug is first taken. Asking about the time the client last took the drug might be important, but it would not address the problem associated with the significant adverse effects.

When interviewing a client with a suspected worm infection, what assessment finding would best address the client's risk of Ascaris?

"Have you eaten any unwashed, fresh vegetables lately?"

Which statement by a patient receiving pancrelipase indicates that the patient has understood his instructions?

"I should take the drug with meals and snacks." Rationale: Pancrelipase is given with meals and snacks so that the enzyme is available when it is needed.

While preparing a client for discharge, the nurse teaches about the proper use of warfarin, which has been prescribed by the physician. Which statement by the client indicates that additional teaching is required?

"I should use aspirin to control my arthritis pain."

The nurse instructs a client with a history of atrial fibrillation on warfarin. Which client statement indicates that teaching about the medication has been effective?

"I will avoid walking barefoot in the house and outdoors."

The nurse has just finished client education with someone who is going home on bulk-forming laxatives. What statement by the client would lead the nurse to believe that the client has understood how to use the medication?

"I will mix the medication with 4 to 8 ounces of liquid and follow it by an additional 4 to 8 ounces."

The nurse educates a client diagnosed with atrial fibrillation about the prescribed warfarin. Which client statements establish the need for further clarification?

"I will stop eating green leafy vegetables, broccoli, yogurt, or cheese."

A patient who is receiving cholestyramine also takes digoxin. The nurse teaches the patient about the drug and administration. The patient demonstrates understanding of the teaching when he states which of the following?

"I will take the digoxin about an hour before the cholestyramine." The absorption of digoxin can be decreased or delayed with colestyramine, a bile acid sequestrant. Therefore, the digoxin should be taken 1 hour before for 4 to 6 hours after the colestyramine.

An older adult client has been using levothyroxine for several years on an outclient basis. Which client statement should the nurse attribute to the decreased effect of levothyroxine? -"I've stopped taking aspirin for my arthritis and started using Tylenol." -"Overall, I'd say that I'm under a lot of stress lately." -"I've been using a lot of antacids lately because of my indigestion." -"My daughter and I have started the Atkin's diet to try to lose some weight."

"I've been using a lot of antacids lately because of my indigestion."

The nurse has just completed discharge instructions to a client who will be using a pen device to deliver his insulin dose. What statement by the client indicates a need for further instruction? -"I need to turn the dial on the pen and listen for the locking ring before administering my insulin." -"This will make it easier for me to give my own insulin because of my poor eyesight." -"I determine my dosage by the number of clicks that I hear." -"If I forget to take my insulin pen, I will be able to use the one that my wife uses."

"If I forget to take my insulin pen, I will be able to use the one that my wife uses."

A female client with hyperthyroidism reports nervousness and "racing" heart one week after starting antithyroid medication. How should the nurse respond to the client's report? -"It may take three to four weeks for the effects of this medication to be seen." -"Have you counted your resting heart rate?" -"Increase the dosage of medication each day until relief of symptoms is achieved." -"I'll notify the primary care provider about increasing the dosage."

"It may take three to four weeks for the effects of this medication to be seen."

How should the nurse best explain the role of the posterior pituitary gland? a."It synthesizes different hormones." b."It makes two different hormones." c."It stores hormones that are produced in your brain." d."It controls many of the metabolic functions of the body."

"It stores hormones that are produced in your brain." Feedback: The posterior pituitary stores two hormones produced in the hypothalamus: antidiuretic hormone (ADH, also known as vasopressin) and oxytocin. The posterior pituitary is anatomically an extension of the hypothalamus and is composed mainly of nerve fibers. Although it does not manufacture any hormones itself, it stores and releases hormones synthesized in the hypothalamus. The hypothalamus of the brain and the pituitary gland interact together to control most metabolic functions of the body and to maintain homeostasis.

A university student has been diagnosed with a helminth infection and tells the nurse, "I've never felt so disgusting in my life. One thing is for sure: I'm not telling a soul about this." What is the nurse's most therapeutic response?

"It's important for you to know that isn't a reflection of your character or your lifestyle."

What statement would be appropriately included in the teaching concerning type 2 diabetes? -"Until you need to start insulin injections, you do not have to check your blood sugar." -"Regular exercise makes your body better able to use the insulin it produces." -"Clients with type 2 diabetes always progress to insulin injections if they do not follow dietary guidelines." -"If you drink alcohol, it may be necessary for you to increase your oral antidiabetic medication."

"Regular exercise makes your body better able to use the insulin it produces."

The nurse is teaching a client how to take his diltiazem. Which instruction would be most appropriate?

"Swallow the drug whole with a large glass of water." Rationale: Diltiazem, like other calcium channel blockers, should be swallowed whole with a large glass of water. The tablet should not be split in half, crushed, or chewed.

A male client who is prescribed digoxin asks the nurse how he should take the drug. Which instruction would be most appropriate?

"Take the drug on an empty stomach at the same time each day."

The nurse is providing client teaching regarding the administration of levothyroxine (Synthroid). What is the nurse's priority teaching point? -"Take the medication before going to bed at night." -"Take the medication with a full glass of water." -"Take the medication after breakfast." -"Remain in the upright position for 30 minutes after taking it

"Take the medication with a full glass of water."

Which client statement demonstrates an accurate understanding of the action of a prescribed anticoagulant?

"This medication will keep unnecessary clots from forming in my blood."

A pregnant client asks about the safety of taking thyroid hormone replacement drugs during her pregnancy. What is the nurse's best response? -"You don't need thyroid hormones anymore if you took fertility drugs." -"Take thyroid hormones if you gain more than the recommended amount of weight during pregnancy." -"Thyroid hormones are pregnancy category A drugs and should be taken during pregnancy." -"Continue thyroid medication only if you have a history of heart disease."

"Thyroid hormones are pregnancy category A drugs and should be taken during pregnancy."

The female client states that her father has had high blood pressure all of his life and he is healthy. She does not understand why the care provider has prescribed medication for her blood pressure. What is the nurse's best response?

"With high blood pressure, blood vessels may be damaged and that can cause heart and kidney disease." Rationale: Hypertension causes damage to blood vessels within the body that may lead to cardiovascular and kidney damage. Hypertension may be treated with or without medications. It is not within the scope of the nurse to decide whether or not the client needs medications, but to provide the education regarding the disease process and medications.

The nurse is providing health education to the parents of a child who has been diagnosed with pinworms and who is being treated with mebendazole. What should the nurse teach the parents in order to promote safe and effective treatment?

"You can give the drug with food to reduce the chance of an upset stomach."

The nurse is caring for a client with severe hypothyroidism and knows to contact the health care provider if which symptoms of myxedema coma occur? (Select all that apply.) -Decreased blood pressure -High blood glucose level -Decreased respirations -Decreased level of consciousness -Fever

-Decreased level of consciousness -Decreased respirations -Decreased blood pressure

The home health nurse is visiting with a client who has been prescribed diphenoxylate with atropine (Lomotil) for chronic diarrhea. Which of the following adverse effects should the nurse be aware of when assessing the client's present condition? Select all that apply

-Headache -Urinary retention

A nurse must recognize the duration of insulin so as not to cause harm to the client with administration of the improper type of insulin. Which insulins are rapid acting? (Select all that apply.) -Insulin aspart (NovoLog) -Insulin glargine (Lantus) -Insulin zinc suspension (Humulin L) -Insuline glulisine (Apidra) -Isophane insulin suspension (Novolin N)

-Insulin aspart (NovoLog) -Insuline glulisine (Apidra)

The nurse is caring for an older adult client who has type 2 diabetes and chronic kidney disease. Which drugs will be used with great caution in this client? (Select all that apply.) -Metformin (Glucophage) -Chlorpropamide (Diabinese) -Miglitol (Glyset) -Regular insulin -Acarbose (Precose)

-Metformin (Glucophage) -Acarbose (Precose) -Miglitol (Glyset)

The nurse monitoring a client receiving insulin glulisine (Apidra) notices the client has become confused, diaphoretic, and nauseated. The nurse checks the client's blood glucose and it is 60 mg/dL (3.33 mmol/L). Which can a nurse give to treat a client with a hypoglycemic episode? (Select all that apply.) -Isulin detemir (Levemir) -Insulin glargine (Lantus) -Orange or other fruit juice -Hard candy -Glucose tablets

-Orange or other fruit juice -Glucose tablets -Hard candy

Laxatives and cathartics are used in a wide variety of ways. The nurse knows that which of the following are appropriate uses for these drugs? Select all that apply.

-Preparation for colonoscopy -Hepatic encephalopathy -Following myocardial infarction -With opioid analgesics

What are the steps in distribution?

1.) protein-binding 2.)blood flow 3.)body tissue affinity 4.) pharmacologic effect

A patient is receiving digoxin. The nurse monitors the patient's digoxin level. Which finding would the nurse interpret as therapeutic?

1.5 ng/mL

Most clients with Stage 2 hypertension will require how many antihypertensives to lower blood pressure to goal initially? (Choose one)

2 Rationale: Most clients with stage 2 hypertension will require 2 medications to lower their blood pressure to goal initially.

A nurse obtains the serum digoxin level and immediately reports a level greater than which value?

2.0 ng/mL

A nurse caring for a patient with atrial fibrillation who is started on digitalis is required to monitor plasma digitalis levels in the patient's blood. Which plasma level of digitalis indicates that the nurse should report to the practitioner?

2.2 ng/mL

A nurse caring for a patient with atrial fibrillation who is started on digitalis is required to monitor plasma digitalis levels in the patient's blood. Which of the following plasma levels of digitalis indicates that the nurse should report to the practitioner?

2.2 ng/ml

A client is receiving anticoagulant therapy and has an INR done. Which result would the nurse interpret as being therapeutic?

2.4

The nursing instructor is discussing the use of histamine2 receptor antagonists in the treatment of peptic ulcer disease. According to the instructor, which of the following is a typical dosing schedule of famotidine during the first few weeks of treatment for gastric ulcer in patients with normal renal function?

20 mg bid Rationale: During the first few weeks of treatment for gastric ulcer, 40 mg of famotidine is administered at bedtime each day or 20 mg is administered twice daily. Thereafter, the typical maintenance dose is 20 mg at bedtime each day. Dosage is typically reduced in patients with impaired renal function.

A client weighing 176 lb (80 kg) is prescribed an intravenous bolus of abciximab 0.25 mg/kg followed by a continuous infusion of 0.125 mcg/kg/min. How many mg of the medication will the client receive in one hour? Record the answer to the nearest tenth.

20.6

A patient with primary hypercholesterolemia has been prescribed 10 mg of ezetimibe per day by a physician. The physician has instructed the nurse to continue the drug therapy for 3 days. The drug is available in 10 mg capsules. How many capsules of the drug should the nurse administer the patient during the entire therapy?

3 capsules The nurse should administer 3 capsules of ezetimibe to the patient during the therapy.

A patient is being administered digoxin (Lanoxin) for treatment of heart failure. At what level should the serum potassium level be maintained?

3.5-5.0 mEq/L

When administering tirofiban, the nurse would administer the drug intravenously over which time frame?

30 minutes

A client asks the nurse what dose of acetylsalicylic acid (aspirin) is needed for antiplatelet effects to prevent heart attacks. What dose is most appropriate to reduce platelet aggregation?

300 mg

After administering ivermectin, when should the nurse expect it will reach peak levels?

4 hours

A male client states that he is seeing halos around lights. The client takes digoxin (Lanoxin) by mouth every day. The health care provider orders the client to have serum digoxin level drawn. Which of the digoxin levels indicate the client is experiencing toxicity?

4.0 nanograms per milliliter

A patient states that he is seeing halos around lights. The patient takes digoxin (Lanoxin) by mouth every day. The physician orders the patient to have serum digoxin level drawn. Which of the digoxin levels indicate the patient is experiencing toxicity?

4.0 nanograms per milliliter

A child weighing 14.5 kg has von Willebrand's disease and has been prescribed desmopressin 0.3 mcg/kg IV. How much desmopressin should the nurse administer? a.4.4 mcg b.9.6 mcg c.10.3 mcg d.21.1 mcg

4.4 mcg Feedback: The normal dosage of desmopressin used to treat von Willebrand's disease is 0.3 mcg/kg. Multiply this dosage times the child's weight: 14.5 kg × 0.3 g = 4.4 g.

A client is diagnosed with Prinzmetal's angina. What drug would the nurse expect to administer to this client? a. Nadolol b. Diltiazem c. Propranolol d. Metoprolol

A

A client with a longstanding diagnosis of angina has been prescribed nitroglycerin and diltiazem. In addition to monitoring the client's heart rate, the nurse should prioritize what assessment? a. Blood pressure b. Level of consciousness C. Daily weights and abdominal girth d. Respiratory status

A

After the client's anginal pain is relieved, he says to the nurse, "That nitroglycerin works great. How does it do that?* What is the nurse's best response? a. "Nitroglycerin decreases the amount of oxygen needed by the heart to function." b. "Nitroglycerin makes the coronary arteries open much wider." C. "Nitroglycerin promotes growth of new, smaller arteries to supply oxygen to the heart. d. "Nitroglycerin decreases preload and afterload."

A

The nurse is caring for a client prescribed ranolazine. The client asks why this drug is different from the beta-blocker that he was previously taking. What is the nurse's best response? a. "This drug does not slow your heart rate. b. "This drug increases myocardial oxygen demand." C. "This drug slows the QT intervals." d. "This maintains blood pressure with no hypotensive effects.

A

The nurse is caring for a client who has been prescribed nitroglycerin sublingually. When providing client education, the nurse should tell the client to expect relief of chest pain within what period of time? a. 1 to 3 minutes b. 5 to 10 minutes c. 15 to 20 minutes d. 30 to 60 minutes

A

The nurse is caring for a client who takes metoprolol for angina. The client asks how long it takes for the medicine to begin working. What is the nurse's best response? a. 15 minutes b. 30 minutes C. 1 hour d. 90 minutes

A

The nurse, caring for a client taking a beta-blocker and a nitrate to treat angina, recognizes the need for careful monitoring because of what comorbidity? a. Chronic obstructive pulmonary disease (COPD) b. Rheumatoid arthritis (RA) C. Irritable bowel syndrome (IBS) d. Chronic urinary tract infection (UTI)

A

What adverse reaction does the nurse anticipate if a client takes a beta-blocker with clonidine? à. Hypertension b. Bradycardia C. Chest pain d. Syncope

A

A nurse will use extreme caution when administering heparin to which patient?

A 38-year-old male with peptic ulcer disease

Heparin therapy is prescribed to a client with deep vein thrombosis. Which client would be at greatest risk of bleeding from this therapy?

A 45-year-old corporate executive with peptic ulcer disease

A parent has informed the nurse that she has been applying an OTC antifungal to her infant's buttocks and perineal region. What question related to safety should the nurse ask the parent? A) "Are there any open wounds on the areas where you're putting the ointment?" B) "Did you take any antifungal medications while you were pregnant?" C) "Did you explore any herbal or alternative remedies before starting to use this ointment?" D) "Does your infant have any food allergies that you're aware of yet?"

A) "Are there any open wounds on the areas where you're putting the ointment?" Explanation: Topical agents should not be used over open or draining areas that would increase the risk of systemic absorption and toxicity. Allergy status is always important, but questions of applying the medication over open areas would be a safety priority; no aspect of the scenario suggests a hypersensitivity response. Previous use of herbal remedies may be relevant but is not a high priority. The mother's use of antifungals while pregnant is not relevant.

A parent has informed the nurse that she has been applying an over the counter antifungal to her infant's buttocks and perineal region for a rash. What question related to safety should the nurse ask the parent? A) "Are there any open wounds on the areas where you're putting the ointment?" B) "Did you take any antifungal medications while you were pregnant?" C) "Did you explore any herbal or alternative remedies before starting to use this ointment?" D) "Does your infant have any food allergies that you're aware of yet?"

A) "Are there any open wounds on the areas where you're putting the ointment?" Explanation: Topical agents should not be used over open or draining areas that would increase the risk of systemic absorption and toxicity. Allergy status is always important, but questions of applying the medication over open areas would be a safety priority; no aspect of the scenario suggests a hypersensitivity response. Previous use of herbal remedies may be relevant but is not a high priority. The mother's use of antifungals while pregnant is not relevant.

A client has been diagnosed with a fungal infection and been prescribed a topical antifungal medication. What assessment question should the nurse ask when addressing the possible etiology of the fungal infection? A) "Have you been prescribed any antibiotics in the recent past?" B) "Have you received any vaccinations in the past few weeks?" C) "When was the last time you visited a hospital or clinic?" D) "Were you prone to acne when you were younger?"

A) "Have you been prescribed any antibiotics in the recent past?" Explanation: A course of antibiotics can often precipitate a fungal infection. Hospital visits, immunizations and a history of acne are not identified as risk factors or causes of fungal infections.

A nurse is teaching a client about the medication regimen surrounding fluoroquinolones. Which statement made by the client would indicate the need for additional education? A) "I will only drink enough water to swallow the drug." B) "I will take all of my antibiotics." C) "If I have trouble breathing, I will call 911." D) "I may have some abdominal pain with this medication."

A) "I will only drink enough water to swallow the drug." Explanation: Clients taking fluoroquinolones should increase, not limit, their fluid intake. It is important that the client take all of the prescribed antibiotics and understand adverse reactions, such as abdominal pain. Difficulty breathing may indicate hypersensitivity and is an emergency.

A client with a fungal infection has been prescribed a miconazole vaginal suppository. What education should the nurse provide about safe and effective administration? A) "If possible, lie down for 10 or 15 minutes after inserting the medication." B) "Insert the medication into your lower vagina so that it's distributed evenly." C) "Take showers rather than baths until you've finished the course of medication." D) "You can douche up to once per day to help distribute the medication."

A) "If possible, lie down for 10 or 15 minutes after inserting the medication." Explanation: Vaginal suppositories should be inserted high in the vagina, with the client lying recumbent after administration. Douching is not recommended and there is no reason to avoid bathing.

A student has presented to the campus medical clinic reporting signs and symptoms that suggest vaginal candidiasis. What assessment question should the nurse prioritize when assessing the safety and effectiveness of drug therapy? A) "Is there any possibility that you could be pregnant?" B) "When was the last time that you had a yeast infection?" C) "Are you allergic to aspirin or any other over-the-counter pain medications?" D) "Would you say that you generally eat a healthy diet?"

A) "Is there any possibility that you could be pregnant?" Explanation: Pregnancy would have a major bearing on the plan of drug therapy because most antifungals are either contraindicated or require cautious use during pregnancy. Consequently, this assessment question would be prioritized over each of the others, even though it is necessary to ask about previous illnesses, nutritional status and allergies.

A client with a cutaneous fungal infection has been prescribed naftifine. When reviewing the prescriber's instructions, what should the nurse explain to the client? A) "Make sure you don't use this for more than one month." B) "Make sure to avoid taking any corticosteroid medications while you're on this drug." C) "If your schedule allows it, apply the ointment four or five times a day." D) "It's important to stay out of the sun until you've finished treatment."

A) "Make sure you don't use this for more than one month." Explanation: Naftifine should be used twice a day for no longer than 4 weeks due to the risk of adverse effects and possible emergence of resistant strains of fungi. Corticosteroids are not necessarily contraindicated and the drug does not cause photosensitivity.

After reviewing information about drugs used to treat lower respiratory system conditions, a group of nursing students demonstrate understanding of the information when they identify which as an example of a short-acting beta-2 agonist (SABA)? A) Albuterol B) Formoterol C) Salmeterol D) Arformoterol

A) Albuterol Explanation: Albuterol is a SABA. Formoterol, salmeterol, and arformoterol are all long-acting beta-2 agonists.

A client is receiving gentian violet as topical treatment for a fungal infection. What instruction should the nurse include in the teaching plan? A) "This medication can stain your skin and clothing." B) "Take this medicine with good to minimize upset stomach." C) "Cover the area with plastic wrap to seal it off." D) "Apply a thin layer of the medicine to any open areas on your skin."

A) "This medication can stain your skin and clothing." Explanation: Gentian violet stains the skin and clothing bright purple. Occlusive dressings should be avoided to prevent systemic absorption; gentian violet is very toxic when absorbed. Gentian violet should not be applied to open lesions. It is topical, not oral.

A client is to receive rifampin. Which would be most important for the nurse to include in the teaching plan for this client? A) "Your urine or sweat may become orange in color." B) "The drug can cause an allergic reaction." C) "You might experience some nausea or stomach upset." D) "Call your health care provider if you experience headache or dizziness."

A) "Your urine or sweat may become orange in color." Explanation: This drug causes body fluids to turn orange. The client needs to be informed of this to avoid being frightened when it occurs. Any drug can cause a hypersensitivity reaction. Although this information is important, it is not the priority. GI adverse effects are common. Although this instruction would be important, it would not be the priority. Headache and dizziness are common CNS effects of the drug that do not need to be reported.

A client has been on sulfonamide therapy for the last six weeks. What client report may cause the health care provider to discontinue the sulfonamide? A) 10 lb weight loss B) Loose stools for the last two days C) Unable to eat spicy foods D) Decreased appetite

A) 10 lb weight loss Explanation: GI side effects with sulfonamide therapy may be mild, such as decreased appetite, inability to eat spicy foods (due to stomatitis), or loose stools for two days. But the drug may need to be discontinued for more serious problems such as pronounced weight loss.

The nurse assesses the serum theophylline of a client. Which finding would the nurse identify as being therapeutic? A) 15 mcg/mL B) 5 mcg/mL C) 25 mcg/mL D) 30 mcg/mL

A) 15 mcg/mL Explanation: Therapeutic theophylline levels range from 10 to 20 mcg/mL. A value of 15 mcg/mL would be considered therapeutic.

The nurse instructs a patient who is using albuterol for exercise-induced bronchospasm to use the inhaler at which time? A) 15 minutes prior to exercising B) One hour before exercise C) Immediately on beginning to exercise D) In the morning of the day when exercise is planned

A) 15 minutes prior to exercising Explanation: Albuterol for exercise induced bronchospasm should be administered 15 minutes prior to exercising.

A patient receives guaifenesin. The nurse would expect the drug to begin acting within which time frame? A) 30 minutes B) 15 minutes C) 45 minutes D) 60 minutes

A) 30 minutes Explanation: Guaifenesin has an onset of action of 30 minutes.

After teaching a group of nursing students about the use of anti-infectives for prophylaxis, the instructor determines that the students need additional teaching when a student identifies what as an example? A) A client with an oral yeast infection who receives antifungal therapy. B) A client who is scheduled for abdominal surgery receives antibiotic therapy. C) A client with a mitral valve replacement receives antibiotics before a tooth extraction. D) A client who receives antimalarial therapy in preparation for travel to Africa.

A) A client with an oral yeast infection who receives antifungal therapy. Explanation: Prophylaxis is the use of anti-infectives to prevent infections before they occur. The use of antifungal agents for an existing yeast infection is treatment, not prophylaxis. Preoperative therapy to prevent a postoperative infection would be an example of prophylaxis. Therapy before a dental procedure is an example of prophylaxis. Use of antimalarials before travel is an example of prophylaxis.

The nurse is caring for several medical clients who have infections. For which client would the administration of a broad spectrum antibiotic be most appropriate? A) A client with signs of infection who is awaiting the results of laboratory testing B) A client with pneumonia who has a confirmed allergy to penicillins C) A client who is being treated for a urinary tract infection for the third time this year D) A client with an infected surgical wound

A) A client with signs of infection who is awaiting the results of laboratory testing Explanation: Broad spectrum antibiotics are effective against a wide variety of microorganisms. None of the other listed clients have obvious indications for using a broad spectrum antibiotic.

A nurse is aware that the concept of selective toxicity is foundational to antimicrobial therapy. Which statement most accurately describes selective toxicity? A) A drug harms microbes without harming human cells. B) A drug's effect on microorganisms is proportionate to dose. C) Most microbes may be collected from a host and cultured on an alternative medium. D) A drug can be isolated and produced in a controlled manner in a laboratory setting.

A) A drug harms microbes without harming human cells. Explanation: An important principle of antimicrobial therapy is selective toxicity, which is the ability to suppress or kill an infecting microbe without injury to the host. This concept does not denote dose-dependent effects, the ability to culture a microorganism, or the production of a drug.

For what client would the use of prophylactic anti-infective be most appropriate? A) A pre-surgical client who has an artificial heart valve B) A client who works in a clinic where many ill children are treated C) A client with type 2 diabetes who lives in a crowded home D) A client who uses street drugs intravenously

A) A pre-surgical client who has an artificial heart valve Explanation: A client with valve replacement is especially prone to the development of subacute bacterial endocarditis because of the vulnerability of their heart valves; prophylactic antibiotic therapy as a precaution. None of the other listed clients has particular indications for the use of prophylactic anti-infectives.

A child prescribed an inhaled corticosteroid agent to decrease respiratory inflammation is now receiving high doses of the drug after only 2 days of treatment. What adverse reaction is the client at risk for developing? A) Adrenal insufficiency B) Tachycardia C) Edema D) Hypoglycemia

A) Adrenal insufficiency Explanation: Adrenal insufficiency is most likely to occur with systemic or high doses of inhaled corticosteroids. The client is not at risk for tachycardia, edema, or hypoglycemia unless related to adrenal insufficiency.

A client is diagnosed with an infection attributable to the gram-negative microorganism Pseudomonas. Which anti-infective agent is most reliable in treating this microorganism? A) Aminoglycoside B) Antifungal C) Aminopenicillin D) GABA analog

A) Aminoglycoside Explanation: Aminoglycosides penetrate the cell walls of susceptible bacteria. As a result, the bacterial cell membrane becomes defective and cannot synthesize the proteins necessary for their function and replication. Aminoglycosides are used to treat infections caused by gram-negative microorganisms, such as Pseudomonas. Antifungals are effective in the resolution of fungus infections. Aminopenicillin agents are not effective in treating gram-negative microorganisms. A GABA analog is used to treat pain related to neuropathy.

A client is being treated for a fungal infection on an outpatient basis. The client calls the clinic and tells the nurse, "The whites of my eyes have gotten a yellowish tinge to them the last few days." What is the nurse's best action? A) Ask the client to be assessed by the provider promptly. B) Have the client self-monitor and call again in 24 hours. C) Have the client present to the emergency department immediately. D) Review the client's allergy status over the phone.

A) Ask the client to be assessed by the provider promptly. Explanation: The client has an early sign of jaundice and should be assessed promptly for possible hepatotoxicity. However, this is not so time-dependent or acute that emergency care is needed. It would be inappropriate to delay assessment by 24 hours. Jaundice is unrelated to medication allergies.

A client is prescribed a leukotriene receptor antagonist. The nurse should evaluate the effectiveness of the medication therapy based on the long term management of symptoms associated with which respiratory condition? A) Asthma B) Emphysema C) Chronic bronchitis D) Pneumonia

A) Asthma Explanation: Leukotriene receptor antagonists block or antagonize receptors for the production of leukotrienes D4 and E4, thus blocking many of the signs and symptoms of asthma. This class of medication is not typically associated with the long term management of any of the other options.

A nurse is caring for a 73-year-old woman who just arrived to the medical surgical floor following a radical mastectomy for breast cancer. The nurse will encourage the patient to cough and deep breathe to help prevent: A) Atelectasis B) Asthma C) Bronchiectasis D) Sinusitis

A) Atelectasis Explanation: Atelectasis most commonly occurs as a result of airway blockage, which prevents air from entering the alveoli, keeping the lung expanded. This occurs when a mucous plug, edema of the bronchioles, or a collection of pus or secretions occludes the airway and prevents the movement of air. Patients may experience atelectasis after surgery, when the effects of anesthesia, pain, and decreased coughing reflexes can lead to a decreased tidal volume and accumulation of secretions in the lower airways. Asthma, bronchiectasis, and sinusitis are not directly related to surgery but result from other conditions.

A nurse is preparing a presentation for a local elementary school parent group about over-the-counter cold medications and their use in children. Which would be most appropriate to include? (Select all that apply.) A) Avoidance of use in children under age 2 B) Importance of reading the label for ingredients and dosage C) Need to follow the directions for how often to give the drug D) Using household measuring devices for dosage measurements E) Using adult formulation with smaller doses

A) Avoidance of use in children under age 2 B) Importance of reading the label for ingredients and dosage C) Need to follow the directions for how often to give the drug Explanation: OTC cough and cold preparations should not be used in children under the age of 4 years. Accurate dosing is essential. Following the directions is essential to prevent inadvertent overdosage. Measuring devices that come with the medication should be used to ensure accurate dosages. Parents should look for the formula for children, infants, or pediatrics.

A client with a positive sputum culture for TB has been started on streptomycin antitubercular therapy. Upon review of the laboratory results, the nurse notes that the client may be experiencing toxicity if which of the following results is abnormal? A) BUN and creatinine B) Amylase and lipase C) Red blood cells and white blood cells D) Sodium and potassium

A) BUN and creatinine Explanation: Streptomycin can cause nephrotoxicity. The nurse would be monitoring the BUN and creatinine levels. Amylase and lipase are related to liver function. Streptomycin does not affect the electrolytes or blood cell counts.

A client is prescribed oxymetazoline. Which assessment data would indicate that the drug is effective? A) Breathing pattern is regular and without difficulty. B) Skin is effectively hydrated. C) Heart rate is maintained at a regular rate and rhythm. D) Urine output is at least 30 mL/hr.

A) Breathing pattern is regular and without difficulty. Explanation: A regular breathing pattern without difficulty would indicate that the drug is effective because oxymetazoline is a decongestant drug. Hydration of the skin, heart rate, and urine output are usually not affected by this drug.

After teaching a group of students about adverse reactions to anti-infective agents, the instructor determines that the students need additional teaching when they identify what as a common adverse effect? A) Cardiac toxicity B) Hypersensitivity C) Kidney damage D) Neurotoxicity

A) Cardiac toxicity Explanation: Cardiac toxicity is not typically associated with anti-infective therapy. Adverse effects commonly associated with anti-infective therapy are direct toxic effects on the kidney, gastrointestinal tract, and nervous system along with hypersensitivity and superinfections.

The nurse is caring for a client who is immunocompromised following a kidney transplant and who has developed an infection. When planning this client's care, the nurse should prioritize what action? A) Carefully maintaining the client's isolation precautions B) Educating the client about the need for frequent hand hygiene C) Monitoring the client's liver function closely D) Monitoring the client for gastrointestinal effects of anti-infective therapy

A) Carefully maintaining the client's isolation precautions Explanation: Even though assessment is an important aspect of nursing care, maintenance of infection control measures is a priority due to the safety risk to the client. The client should be taught about hand hygiene, but it is even more important for nurses and family members to maintain hand hygiene.

Culture and sensitivity testing of a client's urine sample reveals a bacterium that is susceptible to cephalosporins. What medication would be most likely for the nurse to administer? A) Cefaclor B) Imipenem-cilastatin C) Neomycin D) Ciprofloxacin

A) Cefaclor Explanation: Cefaclor is an example of a cephalosporin. Imipenem-cilastatin is an example of a carbapenem. Neomycin is an example of an aminoglycoside. Ciprofloxacin is an example of a fluoroquinolone.

Which would the nurse most likely expect as treatment for a pneumothorax? A) Chest tube insertion B) Anti-infective therapy C) Allergy exposure control D) Surfactant replacement

A) Chest tube insertion Explanation: Chest tube insertion would be important for a pneumothorax to restore the negative pressure. Anti-infective therapy would be appropriate for an infection. Control of allergen exposure would be appropriate for asthma. Surfactant replacement would be appropriate for respiratory distress syndrome.

A client is given theophylline to treat acute asthma symptoms. Which food should the client avoid? A) Chocolate B) Bananas C) Orange juice D) Cranberry juice

A) Chocolate Explanation: Chocolate contains caffeine and is also a xanthine; thus chocolate should be avoided when the client is taking theophylline. Restriction of bananas, orange juice, and cranberry juice is not required.

A client is being treated for viral pneumonia. What assessment data suggests to the nurse that the treatment is effective? (Select all that apply.) A) Client is ambulating in the hall with family B) Respirations are 18 breaths per minute at rest C) Breath sounds are normal D) Temperature is 98.2 degrees F. E) Client is alert and oriented

A) Client is ambulating in the hall with family B) Respirations are 18 breaths per minute at rest C) Breath sounds are normal D) Temperature is 98.2 degrees F. Explanation: Patients report difficulty breathing and fatigue, and they may present with fever, noisy breath sounds, and poor oxygenation. The absence of these signs/symptoms would indicate effective treatment. A client's cognitive functions are not necessary related to the effects of pneumonia.

Incidence of fungal infections has increased with the rising number of people who are immunocompromised. What groups are considered to have a compromised immune system? (Select all that apply.) A) Clients with acquired immune deficiency syndrome (AIDS) and AIDS-related complex (ARC) B) Clients taking immunosuppressant drugs C) Clients who have undergone transplantation surgery or cancer treatment D) The elderly E) Middle-class citizens

A) Clients with acquired immune deficiency syndrome (AIDS) and AIDS-related complex (ARC) B) Clients taking immunosuppressant drugs C) Clients who have undergone transplantation surgery or cancer treatment D) The elderly Explanation: The incidence of fungal infections has increased with the rising number of people who are immunocompromised. This group includes clients with acquired immune deficiency syndrome (AIDS) and AIDS-related complex (ARC), those taking immunosuppressant drugs, those who have undergone transplantation surgery or cancer treatment, and members of the increasingly large elderly population, whose bodies are no longer able to protect against the many fungi found throughout the environment.

A nurse is providing discharge planning for a 45-year-old woman who has a prescription for oral albuterol. The nurse will question the patient about her intake of which of the following? A) Coffee B) Alcohol C) Salt D) Vitamin C

A) Coffee Explanation: The nurse should assess the patient's intake of caffeine, including coffee, tea, soda, cocoa, candy, and chocolate. Caffeine has sympathomimetic effects that may increase the risk for adverse effects. Alcohol, salt, and vitamin C intake is important to assess, but does not potentiate the effects of albuterol.

The nurse is providing education to a client who has been prescribed an antiasthmatic drug. The nurse should instruct the client to avoid excessive intake of what beverage? A) Coffee B) Grapefruit juice C) Green tea D) Acai juice

A) Coffee Explanation: Clients taking antiasthmatic drugs should generally avoid excessive intake of caffeine-containing fluids such as coffee, tea, and cola drinks. These beverages may increase bronchodilation but also may increase heart rate and cause palpitations, nervousness, and insomnia with bronchodilating drugs. None of the other options contain sufficient amounts of caffeine to cause such an affect.

A nurse is asked to explain the difference between community-acquired infections and hospital-acquired infections. What response best describes the difference? A) Community-acquired infections are usually less severe and easier to treat since hospital-acquired infections often involve drug-resistant microorganisms. B) Hospital-acquired infections are typically less severe and easier to treat because they occur in the presence of trained medical staff.\ C) There is no epidemiological difference between the infection types; they are simply categorized as community-acquired or hospital-acquired. D) Only drug-resistant strains of staphylococci, Pseudomonas, and Proteus are categorized as hospital-acquired infections.

A) Community-acquired infections are usually less severe and easier to treat since hospital-acquired infections often involve drug-resistant microorganisms. Explanation: Infections are often categorized as community-acquired or hospital-acquired. Because the microbial environments differ, the two types of infections often have different causes and require different antimicrobial treatment. Generally, hospital-acquired infections are more severe and difficult to manage because they often result from drug-resistant microorganisms and occur in people whose immunity is impaired. A wide variety of organisms can lead to hospital-acquired infections.

A client having an allergic reaction to mold, describes chest tightness and difficulty breathing. Which process is occurring? A) Contraction of smooth muscle in the bronchi B) Suppression of the vagus nerve endings producing tachycardia C) Decreased permeability of the veins and capillaries D) Decreased secretion of the mucous glands

A) Contraction of smooth muscle in the bronchi Explanation: An allergic reaction produces contraction of the smooth muscle in the bronchi and bronchioles. The client will have stimulation, not suppression, of the vagus nerve endings. The client will have increased, not decreased, permeability of the veins and capillaries. The client will have increased secretion from the mucous glands, producing nasal congestion.

A client who takes zinc daily is diagnosed with a severe infection and is ordered levofloxacin (Levaquin). The nurse is aware that taking these two drugs may have what affect on the antibiotic? A) Decreased absorption B) Increased absorption C) Increased elimination D) Decreased elimination

A) Decreased absorption Explanation: Antacids, iron salts and zinc can cause decreased absorption of the antibiotic. There is no affect on elimination of the antibiotic.

A group of students are reviewing the mechanics of oxygen and carbon dioxide exchange in the alveoli. The students demonstrate understanding when they identify which mechanism as being responsible? A) Diffusion B) Active transport C) Facilitated diffusion D) Osmosis

A) Diffusion Explanation: The exchange of oxygen and carbon dioxide in the alveoli occurs via diffusion.

A nurse is describing the action of a prescribed antihistamine to a client. The nurse is describing which medication, by explaining that the drug elicits its antihistamine effects by non-selectively binding to central and peripheral H1 receptors? A) Diphenhydramine B) Cetirizine C) Fexofenadine D) Loratadine

A) Diphenhydramine Explanation: Diphenhydramine is a first-generation antihistamine. First-generation antihistamines elicit their antihistamine effects by non-selectively binding to central and peripheral H1 receptors. Second-generation antihistamines, such as cetirizine, fexofenadine, and loratadine, elicit their effects by being selective for peripheral H1 receptors only.

When describing the action of xanthines, which would the nurse include? (Select all that apply.) A) Direct effect on the mobilization of calcium within the cell B) Stimulation of 2 prostaglandins leading to smooth muscle relaxation C) Inhibition of release of slow-reacting substance of anaphylaxis D) Selectively act at beta-2 receptor sites as agonists E) Block the action of acetylcholine at vagal mediated receptor sites

A) Direct effect on the mobilization of calcium within the cell B) Stimulation of 2 prostaglandins leading to smooth muscle relaxation C) Inhibition of release of slow-reacting substance of anaphylaxis Explanation: Although the exact action is unknown, it is thought that xanthines work by directly affecting the mobilization of calcium within the cell by stimulating two prostaglandins, resulting in smooth muscle relaxation. Xanthines also inhibit the release of slow-reacting substance of anaphylaxis (SRSA) and histamine.

A 76-year-old client has been prescribed an oral antihistamine. What adverse effect presents a priority safety concern for older adult clients? A) Drowsiness B) Dry mouth C) Nasal burning D) Nonproductive cough

A) Drowsiness Explanation: Older adults are more likely to experience anticholinergic effects (dryness of mouth, nose, throat), dizziness, sedation, hypotension, and confusion from the use of antihistamines. Drowsiness and sedation present the greatest safety concerns compared to other effects such as dry mouth and nonproductive cough. Nasal burning may occur due to nasal spray use.

A client with a fungal infection has been prescribed ketoconazole 250 mg PO daily. The client reports nausea and anorexia the day after starting the medication. What is the nurse's best action? A) Encourage the client to try eating small, frequent meals rather than three larger meals B) Have the client withhold the next scheduled dose and come be assessed by the provider C) Collaborate with the care provider to obtain a prescription for ondansetron D) Educate the client about the fact that this adverse effect usually resolves within 72 hours

A) Encourage the client to try eating small, frequent meals rather than three larger meals Explanation: Clients who experience GI effects of antifungals may benefit from smaller, more frequent meals. There is no clear need to have the client discontinue or withhold the medication, though the nurse should follow up with the client. Changes in diet should normally be trialed before seeking prescription antiemetics. There is no guarantee that adverse effects will resolve spontaneously.

A client with an inflammatory disease has been prescribed indomethacin. The nurse is aware of the risk for drug-induced nephrotoxicity and should prioritize what intervention? A) Encouraging the client to increase fluid intake B) Educating the client about sodium limitation C) Administering the medication with meals whenever possible D) Monitoring closely for hematuria

A) Encouraging the client to increase fluid intake Explanation: Adequate hydration is important whenever a client is receiving a potentially nephrotoxic drug. Limiting sodium intake and administering drugs with food do not mitigate this risk. Kidney damage most often results in oliguria and changes to urine and blood chemistry, not hematuria.

When participating in the care of a client who is being treated with antimicrobials, the nurse can promote the appropriate use of these medications in which way? A) Encouraging the use of narrow-spectrum, rather than broad-spectrum, antibiotics B) Promoting the use of prophylactic antibiotics for clients possessing risk factors for infection C) Initiating empiric therapy for all older adult clients admitted to a health care facility D) Promoting the use of herbal treatment for infection rather than antimicrobial drugs

A) Encouraging the use of narrow-spectrum, rather than broad-spectrum, antibiotics Explanation: Guidelines to promote more appropriate use of antimicrobial drugs include using a narrow-spectrum antibacterial drug instead of a broad-spectrum drug, whenever possible, in order to decrease the risk of a superinfection. Herbal alternatives are frequently not available. Antibiotics should not normally be administered in the absence of a diagnosed infection.

After teaching a group of students about antiasthmatic agents, the students demonstrate understanding of the information when they identify what as sympathomimetics? (Select all that apply.) A) Epinephrine B) Terbutaline C) Formoterol C) Budesonide D) Tiotropium E) Cromolyn

A) Epinephrine B) Terbutaline C) Formoterol Explanation: Epinephrine is a sympathomimetic. Terbutaline is a sympathomimetic. Formoterol is a sympathomimetic. Budesonide is an inhaled steroid. Tiotropium is an anticholinergic. Cromolyn is a mast cell stabilizer.

After teaching a group of students about leukotriene receptor antagonists, the instructor determines a need for additional teaching when the students identify which agent as an example? A) Fluticasone B) Montelukast C) Zileuton C) Zafirlukast

A) Fluticasone Explanation: Fluticasone is an inhaled steroid. Montelukast is a leukotriene receptor antagonist. Zileuton is a leukotriene receptor antagonist. Zafirlukast is a leukotriene receptor antagonist.

A pharmacology student is learning about amphotericin B. This drug can be administered via which route? A) IV B) IM C) SQ D) IF

A) IV Explanation: Amphotericin B can be given intravenously. It is not administered via the subcutaneous or intramuscular routes.

Culture and susceptibility tests are performed prior to the prescription of antimicrobial drugs. What is the specific purpose of the culture test? A) Identifies the organism causing the infection B) determines the severity of the infection C) identifies the drugs that might be effective in treatment D) predicts the body's response to the infection

A) Identifies the organism causing the infection Explanation: Culture identifies the infection's causative organism. Susceptibility tests determine which drugs are likely to be effective against the organism.

The nurse is encouraging a client to cough frequently and complete deep breathing exercises. The nurse knows that many factors can cause a decrease of gas exchange for clients. Based on this information, what factors can cause a decrease in gas exchange? Select all that apply. A) Immobility B) Shaken baby syndrome C) Thoracic/pulmonary surgery D) Pneumonia

A) Immobility C) Thoracic/pulmonary surgery D) Pneumonia Explanation: Many factors can cause a decrease in gas exchange, including immobility, thoracic/pulmonary surgery, or pneumonia. Encouraging frequent coughing and deep breathing exercises should assist in improving oxygen delivery to the lungs and tissues. Certain portions of the brain stem are particularly sensitive during infancy; neurons can be destroyed if the child is shaken violently or dropped. This results in "shaken baby syndrome" and can lead to death.

The nurse is caring for a client who has been prescribed micafungin, 150 mg IV daily. What assessment should the nurse prioritize in this client's care? A) Inspection of the client's mouth and throat B) Assessment of the client's perineal skin C) Inspection of the client's scalp D) Temperature monitoring

A) Inspection of the client's mouth and throat Explanation: Micafungin is used in the treatment of clients with esophageal candidiasis. Consequently, the nurse should assess the mucous membranes of the client's mouth and throat.

A neonate is receiving beractant. The nurse understands that this agent is administered by which route? A) Intratracheal B) Subcutaneous C) Intramuscular D) Intravenous

A) Intratracheal Explanation: Beractant is administered intratracheally.

Amphotericin B is ordered for a client with thrush. How does the drug combat the infection? A) It acts on the fungal cell membrane. B) It increases reproduction of fungal cells. C) It increases the activity of the body's immune system. D) It improves fungal protein synthesis.

A) It acts on the fungal cell membrane. Explanation: Amphotericin B binds to sterols in the fungal cell membrane, opening pores in the structure and increasing its permeability. This allows leakage of the cellular components, which results in cell death.

A female client's medical history includes type 2 diabetes, CVA, dysphasia, and chronic renal failure. She develops an infectious process. The health care provider orders erythromycin. Based on the client's medical history, why is erythromycin the drug of choice? A) It is metabolized in the liver. B) It is metabolized in the kidneys. C) It is excreted into the bloodstream. D) It is excreted into the GI tract.

A) It is metabolized in the liver. Explanation: Erythromycin is generally considered safe. Because it is metabolized in the liver and excreted in bile, it may be an alternative in clients with impaired renal function.

A client with a systemic fungal infection and a history of diabetes is to receive an antifungal agent. Based on the nurse's understanding about these agents, which agent would the nurse expect as being the least appropriate? A) Ketoconazole B) Fluconazole C) Itraconazole D) Terbinafine

A) Ketoconazole Explanation: Ketoconazole is not the drug of choice for clients with endocrine problems such as diabetes or those with fertility problems. Fluconazole is not associated with the endocrine problems seen with ketoconazole. Itraconazole and terbinafine are associated with liver failure and toxicity.

When describing the major control mechanism for the inspiratory muscle contraction, which would be included? A) Medulla B) Vagus nerve C) Cilia D) Surfactant

A) Medulla Explanation: The medulla contains the respiratory control center that is the major control for stimulating contraction of the inspiratory muscles. The vagus nerve helps stimulate contraction of the diaphragm and inspiration. Cilia line the upper respiratory tract and moves mucus and any trapped substances down toward the throat. Surfactant is a lipoprotein that decreases the surface tension in the alveoli allowing them to stay open.

A 72-year-old patient with TB is undergoing standard treatment in a health care facility. Which of the following nursing interventions should the nurse perform during ongoing assessment of the treatment? A) Monitor for appearance of adverse reactions. B) Monitor patient's vital signs every 24 hours. C) Assess patient's history of contacts D) Use DOT to administer the drug to the patient.

A) Monitor for appearance of adverse reactions. Explanation: The nurse should monitor for the appearance of adverse reactions in the patient during ongoing assessment of the treatment. The nurse should monitor vital signs of the patient every four hours and not every 24 hours when the patient is hospitalized. The nurse should assess the patient's history of contacts as part of the pre-administration assessment and not as part of the ongoing assessment. DOT can only be used by the nurse to administer antitubercular drugs when the patient is at home, at his place of employment, or in school. DOT is not used when the patient is hospitalized.

A client with candidemia is receiving anidulafungin intravenously. What assessment should the nurse prioritize when assessing this client for adverse effects? A) Monitoring the client for signs of hepatotoxicity B) Monitoring the client's urine output, blood urea nitrogen level and creatinine clearance C) Assessing the client's judgment, arousal and orientation D) Auscultating the client's apical heart rate and rhythm

A) Monitoring the client for signs of hepatotoxicity Explanation: Anidulafungin is potently hepatotoxic and the nurse's plan for assessment should reflect this risk. Cardiac, neurologic and renal effects are less commonly encountered.

What serious adverse effect is most associated with amphotericin B? A) Nephrotoxicity B) Hypovolemia C) Cardiogenic shock D) Septic shock

A) Nephrotoxicity Explanation: Systemic antifungal drugs may cause serious adverse effects. Nephrotoxicity is associated with amphotericin B; hepatotoxicity is associated with azole drugs.

What is the advantage of fluoroquinolones over aminoglycosides? A) Oral administration for more serious infections B) Less hepatotoxicity C) Less nephrotoxicity D) There is no advantage

A) Oral administration for more serious infections Explanation: Ciprofloxacin is available in injectable, oral, and topical forms. Gemifloxacin, lomefloxacin, and moxifloxacin are oral agents. Levofloxacin is available in oral and IV forms. Because of its parenteral availability, it may be preferred for severe infections or for use when the patient cannot take oral drugs. Norfloxacin is only available in an oral form. Ofloxacin can be given IV or orally, and is also available as an ophthalmic agent for the treatment of ocular infections caused by susceptible bacteria.

After teaching a class of students about heart failure and drug therapy, the instructor determines that the teaching has been successful when the students identify which drug as most often used as treatment?

Digoxin

Which condition would be the result of air in the pleural space? A) Pneumothorax B) Pneumonia C) Atelectasis D) Asthma

A) Pneumothorax Explanation: A pneumothorax refers to air in the pleural space that exerts high pressure against the alveoli. Pneumonia is an inflammation of the lungs. Atelectasis refers to the collapse of the once-expanded alveoli. Asthma refers to a condition involving reversible bronchospasm, inflammation, and hyperactive airways

What contraindication to aminoglycosides should a nurse be aware of to prevent complications to the client? Select all that apply. A) Preexisting hearing loss B) Pregnancy C) Parkinsonism D) Diabetes E) Hyperlipidemia

A) Preexisting hearing loss B) Pregnancy C) Parkinsonism Explanation: The aminoglycosides are contraindicated in clients with preexisting hearing loss, myasthenia gravis, Parkinsonism, and during lactation and pregnancy.

A client with a diagnosis of asthma has been prescribed ipratropium 2 puffs QID. What is the most likely goal of this treatment? A) Promoting short-term relief of acute asthma symptoms B) Relieving acute bronchoconstriction C) Promoting long-term management of asthma symptoms D) Promoting blood flow in the alveolar capillaries

A) Promoting short-term relief of acute asthma symptoms Explanation: The anticholinergic bronchodilators are most useful in the long-term management of asthma and other conditions producing bronchoconstriction. These drugs are not used in the management of acute exacerbations of asthma. They do not promote alveolar blood flow.

Which term would the nurse use to describe the delivery of blood to the alveoli? A) Pulmonary perfusion B) Ventilation C) Respiration D) Gas exchange

A) Pulmonary perfusion Explanation: The delivery of blood to the alveoli is referred to as pulmonary perfusion. Ventilation refers to the movement of air in and out of the body. Respiration refers to the act of breathing to allow gas exchange and to the exchange of gases at the alveolar level. Gas exchange refers to the movement of oxygen and carbon dioxide across the respiratory membrane in the alveolar sac.

A client has sustained head trauma that has affected the medulla. The nurse's respiratory assessment should be focused on monitoring which factors? A) Rate and depth of respirations B) Signs of cyanosis C) Respiratory crackles D) Productive cough

A) Rate and depth of respirations Explanation: The medulla controls respiration. The medulla receives input from chemoreceptors (neuroreceptors sensitive to carbon dioxide and acid levels) to increase the rate and/or depth of respiration to maintain homeostasis in the body. Cyanosis may a result of poor respiratory function but not directly associated with the function of the medulla. The other options are seen in pneumonia; an inflammation of the lungs

What should the client be told about nasal drops? (Select all that apply.) A) Recline on a bed and hang your head over the edge. B) Sniff hard for a few minutes after administration. C) After using the drops keep head down and rotate it from side to side. D) Do not share the container with anyone. E) Nasal burning or stinging may occur with the use of topical decongestants.

A) Recline on a bed and hang your head over the edge. C) After using the drops keep head down and rotate it from side to side. D) Do not share the container with anyone. E) Nasal burning or stinging may occur with the use of topical decongestants. Explanation: A client should be counseled to administer nasal drops while reclined on a bed with their head over the edge and after using the drops to keep the head dose and rotate it from side to side. The client may experience some burning or stinging after the administration of the nasal spray. The container should not be shared with anyone.

The critical care nurse is caring for a client whose systemic fungal infection is being treated with amphotericin B. The nurse's review of the client's most recent laboratory results indicates a white cell count of 2050/µL (2.05 ×109/L). What is the nurse's best action? A) Report the client's possible bone marrow suppression to the client's provider B) Introduce standard precautions for infection control C) Relocate the client to a negative pressure isolation room D) Administer immune system stimulants as prescribed

A) Report the client's possible bone marrow suppression to the client's provider Explanation: A decrease in white cell counts is highly suggestive of bone marrow suppression. Standard precautions should be implemented for all clients, not introduced when client's display an increased risk for infection. A negative pressure isolation room would not protect the client from microorganisms from the environment (unlike a positive pressure isolation room). Immune system stimulants are not the most common response to drug-related bone marrow suppression.

The nurse is caring for an older adult client who is being treated for community-acquired pneumonia. Over the past 12 hours, the client has had three episodes of diarrhea despite having no known history of gastrointestinal disease. What is the nurse's best action? A) Report this sign of a possible superinfection to the provider. B) Obtain a stool sample for ova and parasite testing. C) Obtain a stool sample for culture and sensitivity testing. D) Advocate for a change in the client's medication regimen.

A) Report this sign of a possible superinfection to the provider. Explanation: This client has most likely been receiving antibiotics and the rapid onset of diarrhea is suggestive of a superinfection. The care provider should be informed and then any necessary laboratory testing will be ordered. Interventions will likely be necessary but this may or may not include a change in the client's medication regimen.

Cystic fibrosis (CF) involves the exocrine glands of which body systems? (Select all that apply.) A) Respiratory B) Urinary C) Gastrointestinal D) Integumentary E) Reproductive

A) Respiratory C) Gastrointestinal E) Reproductive Explanation: CF, a hereditary disease, involves the exocrine glands of the respiratory, gastrointestinal, and reproductive tracts. CF results in the accumulation of copious amounts of very thick secretions in the lungs.

A client with a systemic fungal infection has been prescribed ketoconazole 350 mg PO daily. What assessment should the nurse prioritize before administration? A) Review of liver enzyme levels B) Assessment of cranial nerve function C) Review of blood urea nitrogen and creatinine levels D) Review of potassium, sodium and chloride levels

A) Review of liver enzyme levels Explanation: Ketoconazole has been associated with severe hepatic toxicity and should be avoided in clients with hepatic dysfunction to prevent serious hepatic toxicity. For this reason, liver function would likely be a priority over cranial nerve function and electrolytes. Excretion is renal, so kidney function should be confirmed, but the consequence of hepatic impairment are even more serious.

Which of the following would the nurse identify as a cardiac glycoside?

Digoxin

A group of students are reviewing information about drugs used to treat tuberculosis. The students demonstrate understanding of the material when they identify which drug as a first-line treatment option? A) Rifampin B) Kanamycin C) Ciprofloxacin D) Capreomycin

A) Rifampin Explanation: Rifampin, along with isoniazid, pyrazinamide, ethambutol, streptomycin, and rifapentine, are considered first-line agents for treating tuberculosis. Kanamycin, ciprofloxacin, and capreomycin are second-line agents.

A client has been prescribed sulconazole for the treatment of tinea pedis. When nursing diagnosis should the nurse identify when planning this client's care? A) Risk for impaired skin integrity related to local irritation B) Impaired urinary elimination related to nephrotoxicity C) Risk for injury related to central nervous system depression D) Imbalanced nutrition: Less than body requirements related to liver damage

A) Risk for impaired skin integrity related to local irritation Explanation: Sulconazole is a topical antifungal. As such, there is no significant risk for liver or kidney damage. The drug does not cause CNS depression, though there is a possibility of localized skin irritation where it is applied.

A 43-year-old man has been diagnosed with active TB. He is prescribed a multiple drug therapy, including INH and rifampin. A priority assessment by the nurse will be to monitor which combination of laboratory test results? A) Serum alanine transaminase, aspartate transaminase, and bilirubin B) Red blood count, white blood count, and differential C) Thyroid-stimulating hormone, thyroxine, and triiodothyronine levels D) Fasting blood sugar and 2-hour postprandial blood sugar

A) Serum alanine transaminase, aspartate transaminase, and bilirubin Explanation: The major adverse effect of INH therapy is hepatotoxicity. In hepatotoxicity the hepatic enzyme levels of aspartate transaminase and alanine transaminase will be elevated. Bilirubin will also be elevated, and the patient may present with jaundice. Red and white blood counts and differential would indicate possible hematologic effects, which could be considered adverse effects of the drug therapy, but would not be diagnostic for hepatotoxicity. Thyroid-stimulating hormone, thyroxine, and triiodothyronine levels would indicate a thyroid glandular concern, not hepatotoxicity. Fasting blood sugar and 2-hour postprandial blood sugar would be indicative of diabetes, not hepatotoxicity.

An instructor is teaching a class about the disorders that may affect the movement of gases into and out of the lungs. The instructor determines the need for additional teaching when the students identify which condition as being involved? A) Sinusitis B) Bronchitis C) Atelectasis D) Cystic fibrosis

A) Sinusitis Explanation: Sinusitis is an upper respiratory tract disorder that is not associated with altered gas exchange. Bronchitis affects the lower respiratory tract and can affect the movement of air in and out of the lungs. Atelectasis affects the lower respiratory tract and can affect the movement of air in and out of the lungs. Cystic fibrosis affects the lower respiratory tract and can affect the movement of air in and out of the lungs.

A patient has been prescribed oral tetracycline for the treatment of acne. Which of the following must the nurse include in the patient teaching plan? A) Take the drug on an empty stomach. B) Take the drug along with a meal. C) Take the drug along with milk or fruit juice. D) Take the drug immediately after meals.

A) Take the drug on an empty stomach. Explanation: Oral preparations of tetracycline should be administered on an empty stomach with a full glass of water to maximize absorption. Tetracycline is not absorbed effectively if taken with food, dairy products, or immediately after meals.

The drug's effect on what best reflects the major reason for avoiding the use of tetracyclines in children under 8 years of age? A) Teeth B) Hearing C) Vision D) Kidneys

A) Teeth Explanation: Tetracyclines should be used with caution in children younger than age 8 years because the drugs can potentially damage developing teeth and bones. They do not affect hearing or vision. They are excreted in the urine, so caution is necessary if the client has underlying renal dysfunction; however, this is not the main reason for avoiding use in children.

The nurse is caring for a 6-year-old child who has pyelonephritis. The use of what group of antibiotics would be contraindicated due to the client's age? A) Tetracyclines B) Penicillins C) Cephalosporins D) Aminoglycosides

A) Tetracyclines Explanation: Tetracyclines can potentially damage developing teeth and bones and thus should be used cautiously or avoided in children under the age of 8 years. Penicillins are safe to give to children and commonly used. Cephalosporins are safe to administer to pediatric clients. Aminoglycosides would not be administered to children lightly, but they can be administered when the benefits outweigh the risks, such as an infection that is resistant to other drugs.

Which statement is true in regards to the oral administration of tetracyclines? A) Tetracyclines should be administered with a full glass of water. B) Tetracyclines should be administered on a full stomach. C) Tetracyclines should only be administered in the evening. D) Tetracyclines should only be administered in the morning.

A) Tetracyclines should be administered with a full glass of water. Explanation: Tetracyclines should always be administered with a full glass of water and on an empty stomach (except minocycline and tigecycline may be taken with food).

The nurse is providing care for an older adult who has a urinary tract infection. What aspect of this client's current health status should the nurse focus on when ensuring safe and effective antibiotic therapy? A) The client has a history of cirrhosis. B) The client has type 2 diabetes controlled with diet. C) The client uses bronchodilators for the treatment of asthma. D) The client had a hip fracture 8 months ago.

A) The client has a history of cirrhosis. Explanation: Cirrhosis causes decreased liver function, which would have a major bearing on medication therapy. Each of the other aspects of the client's history are significant, but none will have as direct an effect as liver disease.

A 30-year-old female client received an insect bite that has progressed to cellulitis over the past several days. What client characteristic should the nurse prioritize when administering anti-infectives? A) The client is in the second trimester of pregnancy. B) The client has known allergies to nonsteroidal anti-inflammatories. C) The client was treated for leukemia when she was a teenager. D) The client admits to using marijuana recreationally.

A) The client is in the second trimester of pregnancy. Explanation: Pregnancy and breastfeeding have major effects on the decision to administer anti-infectives and the choice of medication. An allergy to NSAIDs would not have a significant effect on the administration of anti-infectives. Similarly, marijuana use and a distant history of cancer would not have such as large effect on the choice of medication.

The nurse is caring for a client whose fungal infection is being treated with ketoconazole. What change in the client's health status should the nurse prioritize when updating the provider on the client's condition? A) The client's aspartate aminotransferase (AST) and alanine aminotransferase (ALT) levels are elevated B) The client's sodium level is 136 mEq/L (136 mmol/L) and potassium is 5.0 mEq/L (5.0 mmol/L) C) The client vomited 30 minutes after the most recent scheduled dose of ketoconazole D) The client's temperature is 99.5°F (37.5°C)

A) The client's aspartate aminotransferase (AST) and alanine aminotransferase (ALT) levels are elevated Explanation: An increase in AST and ALT could signal hepatotoxicity, which is a significant adverse effect of ketoconazole. The client's electrolyte levels are within norms and the nurse can normally manage a single episode of vomiting independently. The client's temperature is nominally elevated, so liver enzymes would be a clear priority.

A client is being treated for histoplasmosis with amphotericin B. The nurse has completed a comprehensive assessment of the client and should report what assessment finding? A) The client's urine output is 75 mL over six hours B) The client reports a frontal headache C) The client reports a lack of appetite and refused breakfast D) The client had an episode of diarrhea overnight

A) The client's urine output is 75 mL over six hours Explanation: Decreased urine output could be suggestive of renal damage, which must be addressed immediately. Diarrhea, headache and lack of appetite should be addressed by the nurse but are lower priorities.

A neonate is admitted to the neonatal intensive care unit and has difficulty with respirations. What is the reason for this pathophysiologic effect? A) The neonate does not have mature Type II cells, which produce surfactant. B) The neonate lacks alveoli, which signals the brain in the production of oxygen. C) The neonate must be ventilator-dependent due to decreased muscle development. D) The neonate's trachea is too small to accommodate the respiratory gas exchange.

A) The neonate does not have mature Type II cells, which produce surfactant. Explanation: Type II cells produce surfactant. The neonate has immature Type II cells, which will not provide the needed surfactant. The neonate's brain is immature but is not the primary physiologic effect. The neonate's muscle development is immature but is not the primary physiologic effect. The neonate's trachea size in not a factor in this case.

Why are inhaled steroids used to treat asthma and COPD? A) They act locally to decrease release of inflammatory mediators B) They act locally to improve mobilization of edema C) They act locally to increase histamine release D) They act locally to decrease histamine release

A) They act locally to decrease release of inflammatory mediators Explanation: When administered into the lungs by inhalation, steroids decrease the effectiveness of the inflammatory cells. This has two effects, which are decreased swelling associated with inflammation and promotion of beta-adrenergic receptor activity, that may promote smooth muscle relaxation and inhibit bronchoconstriction

Ketoconazole is available in oral form, a shampoo, and a cream. A) True B) False

A) True

Mucolytics are often used for patients with cystic fibrosis, COPD, or tuberculosis. A) True B) False

A) True Explanation: Mucolytics increase or liquefy respiratory secretions to aid the clearing of the airways in high-risk respiratory patients who are coughing up thick, tenacious secretions. Patients may be suffering from conditions such as chronic obstructive pulmonary disease (COPD), cystic fibrosis, pneumonia, or tuberculosis

When describing the structure of the lungs, the nurse would identify the left lung as consisting of how many lobes? A) Two B) Three C) Four D) Five

A) Two Explanation: The left lung consists of two lobes, and the right lung consists of three lobes.

The client is receiving ketoconazole. The nurse should be concerned about a drug-drug interaction on the basis of what other co-morbidity? A) Type 2 diabetes B) Chronic pain C) Venous ulcers D) Penicillin allergy

A) Type 2 diabetes Explanation: Ketoconazole interacts with hypoglycemics, which are given for the treatment of type 2 diabetes. There is no obvious reason why a client's pain, penicillin allergy or venous ulcer would preclude the safe and effective use of ketoconazole.

The alveolar cells that synthesize pulmonary surfactant are the: A) Type II alveolar cells. B) Type I alveolar cells. C) Type I pneumocytes. D) Alveolar macrophages.

A) Type II alveolar cells. Explanation: Type II alveolar cells synthesize pulmonary surfactant, while the Type 1 alveolar cells (which are also know as Type 1 pneumocytes) form an effective barrier between the air and the components of the alveolar wall.

The nurse is caring for a 23-year-old female client who uses oral contraceptives and has been prescribed ampicillin for treatment of a respiratory infection. What information is most important for the nurse to share with this client? A) Use a type of barrier birth control while you are taking this antibiotic. B) Do not take your oral contraceptives while you are taking this medication. C) Your menstrual cycle will have a heavier flow while on this medication. D) Taking this antibiotic with your oral contraceptives will cause an increased chance of bleeding.

A) Use a type of barrier birth control while you are taking this antibiotic. Explanation: Clients taking oral contraceptives should be encouraged to use a barrier type of birth control during penicillin therapy; the penicillin will cause a decreased effectiveness of the contraceptive agent. The client should not be told to stop taking the oral contraceptive. This will not cause a heavier menses. The interaction between oral contraceptives and ampicillin will not cause an increased chance of bleeding, as that interaction is between large doses of penicillins and anticoagulants

A client is being assessed by the home care nurse for the appropriate use of a metered-dose inhaler. Instructions concerning which intervention will assist a client in the proper use of the device? A) Using a spacer B) Administrating corticosteroid first C) Pushing fluids D) Exhaling immediately after administration

A) Using a spacer Explanation: The client should be instructed to use a spacer to increase compliance and accuracy of administration. An asthma spacer is an add-on device used to increase the ease of administering aerosolized medication from a metered-dose inhaler (MDI). The corticosteroid should be administered after the bronchodilator. The increase in fluids will not affect the administration of the inhaler. The client should hold the breath for several seconds after administration of the inhaler.

When describing the action of decongestants, a nurse integrates knowledge of this class of drugs, identifying which mechanism? A) Vasoconstriction of small blood vessels of the nasal membranes B) Selectively binding to peripheral H1 receptors C) Non-selectively binding to central and peripheral H1 receptors D) Anesthetizing stretch receptors in the respiratory passages

A) Vasoconstriction of small blood vessels of the nasal membranes Explanation: Vasoconstriction of small blood vessels of the nasal membranes is the mechanism of action for decongestants. First-generation antihistamines act to non-selectively bind to peripheral H1 receptors; the mechanism of action for second-generation antihistamines is to selectively bind to peripheral H1 receptors only; and peripherally acting antitussives anesthetize stretch receptors in the respiratory passages.

A nurse is taking a history on a client new to the clinic. The client reports being allergic to penicillin. What additional information about this reported allergy would be important for the nurse to find out? A) What reaction symptoms occurred B) What was done to stop the reaction C) Who was the caregiver at the time of the reaction D) What other allergies the client has

A) What reaction symptoms occurred Explanation: It is important to determine what the allergic reaction was and when the client experienced it (e.g., after first use of drug, after years of use). Some clients report having a drug allergy, but closer investigation indicates that their reaction actually constituted an anticipated effect or a known adverse effect to the drug. It would not necessarily be important to find out what was done to stop the reaction, who the caregiver was at the time of the reaction, or what other allergies the client has.

Which client would be at risk of developing an infection? Select all that apply. A) a client with an impaired immune system B) a client who performs daily hygiene care C) a client experiencing diarrhea chronically D) an elderly client who has one chronic disease E) a client who takes nutritional supplements

A) a client with an impaired immune system C) a client experiencing diarrhea chronically D) an elderly client who has one chronic disease Explanation: Many factors impair the host defense mechanisms and predispose a person to infection by disease-producing microorganisms. These include breaks in the skin and mucous membranes, impaired blood supply, neutropenia and other blood disorders, malnutrition, poor personal hygiene, suppression of normal bacterial flora by antimicrobial drugs, diabetes mellitus and other chronic diseases, and advanced age. A client who has chronic diarrhea can experience malnutrition. A client who performs daily care and a client who takes nutritional supplements are not at risk for infection.

Successful treatment with bacteriostatic antibiotics depends upon what factor? A) adequate duration of drug therapy. B) stopping drug therapy when symptoms have subsided. C) using broad-spectrum antibacterial drugs to treat viral infections. D) the type of drug-resistant bacterial strains that can reproduce in the presence of antimicrobial drugs.

A) adequate duration of drug therapy. Explanation: Successful treatment with bacteriostatic antibiotics depends on the ability of the host's immune system to eliminate the inhibited bacteria and an adequate duration of drug therapy. Stopping an antibiotic prematurely can result in rapid resumption of bacterial growth. Antibiotics are not used to treat viral infections.

A client comes to the clinic reporting mouth sores that appear consistent with an oral yeast infection. The nurse should question the client about recent use of: A) antibiotics. B) oral contraceptive agents. C) antivirals. D) antiprotozoals.

A) antibiotics. Explanation: The client is exhibiting signs of a superinfection, which can occur with the use of antibiotics. Superinfections are not associated with oral contraceptives, multivitamins, or anticoagulants.

Drugs that destroy bacteria are known as which type of drug? A) bactericidal B) bacteriostatic C) bacteriostationary D) bacteriophage

A) bactericidal Explanation: Drugs that destroy bacteria are known as bactericidal. Drugs that slow or retard the multiplication of bacteria are known as bacteriostatic. Bacteriostationary keeps the bacteria in place, and bacteriophage uses enzymatic substances to destroy the bacteria.

After teaching a group of nursing students about the action of penicillins, the instructor determines that the teaching was effective when the students identify natural penicillins as exerting which type of effect on microorganisms? A) bactericidal B) bacteriostatic C) fungicidal D) fungistatic

A) bactericidal Explanation: Natural penicillins exert a bactericidal effect on bacteria. Because penicillin targets bacterial cells, it is of no value to treat fungal infections as it is neither fungicidal nor fungistatic. Since it targets the bacterial cell wall to destroy the bacteria, it does not inhibit growth as in bacteriostatic.

After teaching a group of nursing students about the different classes of drugs, the instructor determines that the teaching was successful when the students identify drugs that slow or retard the multiplication of bacteria as which type of medication? A) bacteriostatic B) bactericidal C) bacteriostationary D) bacteriophage

A) bacteriostatic Explanation: Drugs that slow or retard the multiplication of bacteria are known as bacteriostatic. Drugs that destroy the bacteria are referred to as bactericidal. Bacteriostationary keeps the bacteria in place, and bacteriophage uses enzymatic substances to destroy the bacteria.

A client is febrile and is suspected of having a respiratory infection. A sputum culture has been collected and the results of sensitivity testing are expected within 48 hours. The nurse should anticipate that the client may: A) be immediately prescribed a broad-spectrum antibiotic. B) receive supportive care until an antibiotic can be prescribed in 48 hours. C) be treated with antiviral medication on a short-term basis. D) need to provide a follow-up sputum culture before medications can be prescribed.

A) be immediately prescribed a broad-spectrum antibiotic. Explanation: Broad-spectrum antibiotics are often prescribed pending sensitivity testing. There is no need to completely withhold medications until results are acquired. Antiviral medications are not used for this short-term purpose. Follow-up testing may or may not be needed.

Administration of what type of antibiotic by the nurse would be most likely to cause a superinfection? A) broad spectrum B) narrow spectrum C) bactericidal D) bacteriostatic

A) broad spectrum Explanation: One offshoot of the use of anti-infectives, especially broad-spectrum anti-infectives, is destruction of the normal flora, allowing opportunistic pathogens to invade tissue and cause a superinfection. Narrow-spectrum anti-infectives are less likely to kill normal flora, although it is not impossible. A drug may be bactericidal, meaning it kills the pathogen; or bacteriostatic, meaning it prevents reproduction of the pathogen, but this is not related to superinfections.

A client has just been admitted for an overdose of pseudoephedrine. The nurse's assessment of the client should prioritize what system? A) cardiovascular B) neurological C) gastrointestinal D) renal

A) cardiovascular Explanation: Because pseudoephedrine is a stimulant, cardiac symptoms are a notable adverse effect, especially in the acute stage of the overdose. All the other systems should be monitored, but the cardiac-related adverse reactions have priority.

A client has been diagnosed with an infection. The nurse can help to ensure the success of anti-infective treatment by: A) confirming that the medication prescribed is the drug of choice for the specific microorganism. B) teaching the client that significant adverse effects are expected, and must be endured during treatment. C) administering antihistamines, as ordered, to prevent the development of adverse effects. D) monitoring the client closely for signs of arrhythmias or cardiac ischemia.

A) confirming that the medication prescribed is the drug of choice for the specific microorganism. Explanation: In order for treatment to be effective, it is necessary for an anti-infective to be accurately matched to the offending pathogen. Anti-infectives do not normally cause cardiac adverse effects. The client should be taught that adverse effects can be managed and are not necessarily an inevitability that must be endured. Antihistamines do not prevent most adverse effects.

The nurse is reviewing recent laboratory values for an older client prior to administration of tetracycline. Which finding would lead the nurse to hold the dose? A) creatinine of 2.8 mg/dL (247.52 µmol/L) (high) B) potassium of 3.4 mEq/L (3.40 mmol/L) (low) C) hemoglobin of 10.5 g/dL (105.00 g/L) (low) D) leukocyte count of 11.6/mm3 (high)

A) creatinine of 2.8 mg/dL (247.52 µmol/L) (high) Explanation: An elevated creatinine level is associated with renal dysfunction, which may contraindicate the use of tetracycline in an older adult due to the risk of further kidney damage. Although the other findings (if new) may warrant contacting the health care provider, they do not necessarily contraindicate the use of tetracycline.

An adult client, recently diagnosed with a tinea infection, has begun treatment with griseofulvin. During a scheduled clinic visit, the client states to the nurse, "I'm pretty good at reading my body's signals, so I make sure to take a bit extra when I think my infection is getting worse." This client's statement is suggestive of what nursing diagnosis? A) deficient knowledge related to correct use of griseofulvin B) effective therapeutic regimen management related to symptom identification C) disturbed thought processes related to appropriate use of griseofulvin D) ineffective coping related to self-medication

A) deficient knowledge related to correct use of griseofulvin Explanation: In order to achieve maximum therapeutic benefit, it is important for the client to take the drug exactly as prescribed. The client's statement suggests that the client does not appreciate or understand this fact. There is no evidence of pathological thought patterns or ineffective coping. The statement reflects an inaccurate conclusion regarding the symptom management of this disorder.

Guidelines for the use of topical antifungals in children are frequently the same as those for adults. However, safety and dosage recommendations have not been established for most oral and IV antifungals in the pediatric population. Which antifungal drug is suitable for pediatric use? A) fluconazole B) caspofungin C) voriconazole D) metoprolol

A) fluconazole Explanation: Fluconazole is suitable for use in children. Safety and effectiveness have not been established for caspofungin in children; safety and effectiveness have not been established for voriconazole in children under twelve years of age.

A client diagnosed with malaria has been prescribed treatment with sulfadiazine. The nurse should encourage the client to increase consumption of what dietary component? A) fluids B) proteins C) calories D) vitamin D

A) fluids Explanation: When this medication is used, it is imperative that the client maintain a fluid intake that allows for an output of 1500 mL of urine in 24 hours. While the client should attempt to consume a healthy well-balanced diet, fluid intake is vitally important.

After teaching a group of students about the indications for use for aminoglycosides, the instructor determines that the teaching was successful when the students identify which type of infection as a primary indication? A) gram-negative infections B) gram-positive infections C) fungal infections D) viral infections

A) gram-negative infections

A client who is unsure whether symptoms are attributable to a cold or to allergies is taking pseudoephedrine as well as an antihistamine. This combination of drugs creates a risk for what adverse effect? A) hypertension B) hepatotoxicity C) respiratory depression D) acute kidney injury

A) hypertension Explanation: The combination of antihistamines and pseudoephedrine increase the risk of hypertension due to vasoconstriction. This combination does not result in respiratory depression or damage to the liver or kidneys.

Which nursing diagnosis would provide the clearest indication for the administration of inhaled acetylcysteine? A) ineffective airway clearance B) impaired tissue perfusion C) ineffective breathing pattern D) risk for infection

A) ineffective airway clearance Explanation: Acetylcysteine reduces the viscosity of mucous secretions with the ultimate goal of allowing the client to establish and maintain a clear airway. This drug does not directly influence the pattern of breathing. Impaired tissue perfusion can have multiple causes, many of which are unrelated to the presence of mucus secretions.

A nursing instructor is preparing a teaching plan for a nursing pharmacology class on the action of fluoroquinolones. Which would the instructor include? A) interfering with DNA synthesis in the bacterial cell B) interfering with protein synthesis C) disrupting the bacterial cell wall D) blocking ribosomal reading of mRNA

A) interfering with DNA synthesis in the bacterial cell Explanation: The fluoroquinolones exert their bactericidal effect by interfering with the synthesis of bacterial DNA by not allowing the cell to reproduce. The tetracyclines are bacteriostatic and exert their effect by inhibiting bacterial protein synthesis. Penicillins act to disrupt the bacterial cell wall. The aminoglycosides exert their bactericidal effect by blocking the ribosome from reading the mRNA, a step in protein synthesis necessary for bacterial multiplication.

A client is receiving aminoglycoside therapy. The nurse would be alert for: A) kidney dysfunction. B) hallucinations. C) lethargy. D) loss of vision.

A) kidney dysfunction. Explanation: Kidney dysfunction (i.e., nephrotoxicity) is associated with aminoglycosides. Hallucinations are not associated with the use of aminoglycosides. Lethargy is not associated with aminoglycoside therapy. Loss of vision is more commonly associated with neurotoxic agents such as chloroquine. Hearing loss is a possibility with aminoglycosides.

The nurse is preparing to administer the initial intravenous (IV) dose of caspofungin to a client. Before giving the dose, the nurse should review the latest results of what diagnostic testing? Select all that apply. A) liver function B) renal function C) arterial blood gases D) electrocardiogram E) complete blood count

A) liver function B) renal function E) complete blood count Explanation: Prior to administering the first dose of caspofungin, the nurse assesses baseline liver and renal function along with electrolytes, complete blood count, and platelet count. Neither respiratory nor cardiac function is generally affected by this medication, so such assessment is not required.

A group of nursing students are reviewing information about fluoroquinolones. The students demonstrate a need for additional review when they identify which as an example? A) metronidazole B) ciprofloxacin C) levofloxacin D) gemifloxacin

A) metronidazole Explanation: Metronidazole is not an example of a fluoroquinolone. Ciprofloxacin is an example of a fluoroquinolone. Levofloxacin is an example of a fluoroquinolone. Gemifloxacin is an example of a fluoroquinolone.

Tetracycline is often a viable treatment option when a client has an allergy to what antibiotics? A) penicillins B) aminoglycosides C) fluoroquinolones D) macrolides

A) penicillins Explanation: Prescribers order tetracycline when penicillin is contraindicated to treat infections caused by Klebsiella, Neisseria gonorrhoeae, Treponema pallidum, Listeria monocytogenes, Clostridium, Bacillus anthracis, Fusobacterium fusiforme, and Actinomyces.

The nurse administering an anti-infective agent recognizes that the drug will destroy some human cells as well as pathogens because of the absence of: A) selective toxicity. B) virulence. C) resistance. D) antigens.

A) selective toxicity. Explanation: Although anti-infective agents target foreign organisms infecting the body of a human host, they do not possess selective toxicity, which is the ability to affect certain proteins or enzyme systems used only by the infecting organism but not by human cells. Virulence would apply to the destructive power of the infection, not the drug. Resistance is the pathogen's ability to no longer respond to specific anti-infectives. Antigens are proteins bound to the cell that help the body identify a cell as belonging (or not) in the body, and are not the cause of human cell destruction.

The nurse has administered a client's first scheduled dose of amoxicillin. What subsequent client report should prompt the nurse to seek the discontinuation of this drug? A) shortness of breath B) nausea C) heartburn D) frontal lobe headache

A) shortness of breath Explanation: Shortness of breath is suggestive of a hypersensitivity reaction and would likely warrant discontinuation of the drug. Nausea, reflux, and headache should each be addressed, but none likely necessitates discontinuation of drug therapy.

People taking antimicrobial drugs should be instructed to report any problem that could indicate adverse drug effects, lack of therapeutic response, or emergence of another infection. One such problem is: A) skin rash. B) headache. C) constipation. D) muscle soreness.

A) skin rash. Explanation: Clients should be instructed to report nausea, vomiting, diarrhea, skin rash, recurrence of symptoms for which the antimicrobial drug was prescribed, or signs of new infection (e.g., fever, cough, sore mouth, drainage). These problems may indicate adverse effects of the drug, lack of therapeutic response to the drug, or another infection.

A client has been diagnosed with osteomyelitis and has been prescribed clindamycin, a narrow spectrum antibiotic. When planning this client's care, the nurse should understand that: A) the microorganism causing the infection is likely known. B) the client likely has compromised immune function. C) broad spectrum antibiotics were likely administered without success. D) the care team wished to reduce the client's risk of adverse effects.

A) the microorganism causing the infection is likely known. Explanation: Narrow spectrum antibiotics are appropriate when the identity of the microorganism is known or strongly suspected. This is unrelated to the client's immune function. Broad spectrum drugs are not necessarily trialed before using narrow spectrum drugs. The risk of adverse effects is not the determining factor.

An older adult client is assessed in the clinic for signs and symptoms of chronic bronchitis related to pneumococci. Which sulfonamides will most likely be prescribed? A) trimethoprim-sulfamethoxazole B) tetracycline C) doxycycline D) demeclocycline

A) trimethoprim-sulfamethoxazole Explanation: Sulfonamides are bacteriostatic drugs that were once effective against a wide range of gram-positive and gram-negative bacteria. Trimethoprim-sulfamethoxazole is used to treat chronic bronchitis due to pneumococci. Tetracycline, doxycycline, and demeclocycline are not sulfonamides but rather antibiotics.

What would the nurse teach the client about the safe and effective use of nitroglycerin? Select all that apply. a. "The tablet should fizzle or burn when placed under the tongue." b. "Protect the drug from heat and light." c. "Always replace when past the expiration date." d. "Older tablets may require you to use two tablets at one time." e. "Nitroglycerin does not lose its potency easily."

A, B, C

A patient is taking cholestyramine (Questran) to reduce LDL cholesterol. Cholestyramine will cause a decrease in absorption of which of the following medications?

Digoxin (Lanoxin) Bile acid sequestrants may decrease absorption of digoxin (Lanoxin). Cholestyramine will not decrease the ibuprofen, aspirin, or acetaminophen.

Which of the following is to treat a massive digoxin overdose? (Choose one)

Digoxin immune Fab (Digibind)

The nurse is caring for a 26-year-old patient who has been diagnosed with roundworms. The patient is prescribed pyrantel. What adverse effect would the nurse inform the patient about?

Abdominal discomfort or pain

Which of the following assessment findings may indicate the client is experiencing digitalis toxicity? Select all that apply.

Abdominal pain • Diarrhea • Drowsiness • Vomiting

When describing proton pump inhibitors, which of the following would be appropriate to include?

Acid labile Rationale: Proton pump inhibitors are acid labile. Proton pump inhibitors are rapidly absorbed from the GI tract. Proton pump inhibitors are extensively metabolized by the liver. Proton pump inhibitors are excreted in the urine.

A patient is being sent home with orders for a laxative PRN. The nurse is conducting patient teaching on the use of a laxative. What will the nurse inform the patient is one of the most common adverse effects of a laxative?

Abdominal cramping

An 86-year-old client has been prescribed ranitidine (Zantac). The nurse knows that an older adult who takes H2RAs may experience what adverse effect?

Agitation Rationale: With H2RAs, older adults are more likely to experience adverse effects, especially confusion, agitation, and disorientation, with cimetidine. In addition, older adults often have decreased renal function, and doses need to be reduced.

A nurse is educating a group of nursing students on the mechanism of actions of angiotensin converting enzyme inhibitors (ACEI). Which of the following should the nurse explain as the hormone produced by the adrenal cortex?

Aldosterone Rationale: The nurse should explain that aldosterone is the hormone produced by the adrenal cortex. Renin and angiotensin are hormones produced in the kidneys, and not by the adrenal cortex. Enalapril is not a hormone; it is an ACEI antihypertensive drug.

A group of students are reviewing the various antihypertensive agents available. The students demonstrate understanding of the information when they identify which agent as an example of a renin inhibitor?

Aliskiren Rationale: Aliskiren is a renin inhibitor. Mecamylamine is a ganglionic blocker. Candesartan is an angiotensin II receptor blocker; captopril is an ACE inhibitor.

The patient has called the office to see if using bismuth subsalicylate will work to treat simple diarrhea. The nurse knows this will be a safe and effective treatment for diarrhea unless the patient has:

Allergy to aspirin

When describing the possible adverse effects associated with omeprazole therapy, which of the following would the nurse identify as least common?

Alopecia Rationale: Alopecia can occur with proton pump therapy, but it is not a common adverse effect. Common adverse effects include dizziness, headache, and cough.

A nurse who provides care in a busy clinic is aware of the high incidence and prevalence of hyperlipidemia and the consequent need for antihyperlipidemics in many patients. Treatment of high cholesterol using statins would be contraindicated in which of the following patients?

An obese male client who is a heavy alcohol user and who has cirrhosis of the liver Active liver disease is a contraindication to the use of statins. As well, heavy alcohol use increases the risk of liver dysfunction. Respiratory disease, recent surgery, and organic cognitive deficits do not preclude the use of statins for high cholesterol.

What does the basic nursing care of patients with shock require?

An ongoing systematic assessment Rationale: Nursing care of patients with shock requires ongoing systematic assessment. Options A, B, and C are incorrect; basic nursing care of patients with shock does not require understanding the causes of shock, understanding the primary prevention of shock, or restoring intravascular volume.

Mr. Emmeott is seen by the physician for follow-up for diabetic nephropathy. The physician prescribed losartan for the treatment of this condition because the medication has been shown to reduce the rate of end-stage renal disease. Which drug class does this drug belong to?

Angiotensin II receptor blockers Rationale: Losartan is an angiotensin II receptor blocker (ARB) and is recommended for patients with diabetic renal disease because ARBs have been shown to reduce the rate of end-stage renal disease.

The nurse is caring for a client receiving octreotide for the treatment of acromegaly. The nurse should emphasize the need for what form of baseline and periodic testing? a.Ultrasound evaluation of the gallbladder b.Magnetic resonance imaging (MRI) of the brain c.Serum glucose levels d.Complete blood counts

Ans: A Feedback: Arrange for baseline and periodic ultrasound evaluation of the gallbladder for patients receiving octreotide because common adverse effects of the drug are acute cholecystitis, cholestatic jaundice, biliary tract obstruction, and pancreatitis. A patient taking octreotide does not need baseline and periodic MRIs of the brain, serum glucose levels, or complete blood counts.

The nurse is caring for a client with infertility related to hyperprolactinemia. What drug would best treat this problem? a.Bromocriptine mesylate b.Somatropin c.Leuprolide d.Desmopressin

Ans: A Feedback: Bromocriptine mesylate is indicated for the treatment of female infertility associated with hyperprolactinemia. Somatropin is indicated for the treatment of growth failure, Turners syndrome, AIDS wasting and cachexia, and growth hormone deficiency in adults. Leuprolide is used as antineoplastic agent for treatment of specific cancers and for treatment of endometriosis and precocious puberty that results from hypothalamic activity. Desmopressin is used for the treatment of neurogenic diabetes insipidus, von Willebrand's disease, hemophilia; and is currently being studied for the treatment of chronic autonomic failure.

The nurse administers desmopressin to the client to treat diabetes insipidus. What assessment finding would indicate to the nurse that the desmopressin is producing a therapeutic effect? a.Decreased urine output b.Decreased water reabsorption c.Increased plasma osmolarity d.Decreased blood volume

Ans: A Feedback: Desmopressin produces its antidiuretic activity in the kidneys, causing the cortical and medullary parts of the collecting duct to become permeable to water, thereby increasing water reabsorption and decreasing urine formation. These activities reduce plasma osmolarity and increase blood volume.

When providing health education to a client taking octreotide, the nurse should warn the client about the possibility of what potential adverse effect? a.Abdominal pain b.Alteration in consciousness c.Changes in vision d.Tinnitus

Ans: A Feedback: Octreotide and lanreotide have commonly been associated with the development of acute cholecystitis, cholestatic jaundice, biliary tract obstruction, and pancreatitis, which would present with abdominal pain, so patients should be taught to report this symptom. The drug is not associated with alteration in consciousness, changes in vision, or muscle cramps.

A 58-year-old man is admitted to the emergency department. A diagnosis of severe digoxin toxicity is made. Bradycardia is present, and an electrocardiogram (ECG) confirms toxicity. The nurse will administer which drug?

Digoxin immune fab

What is the nurse's priority assessment when administering hypothalamic or pituitary agents to older adults? a.Hydration and nutrition b.Balance and fall risk c.Cognitive function and adherence to treatment d.Bowel function

Ans: A Feedback: Older adults may be more susceptible to the imbalances associated with alterations in the endocrine system. They should be evaluated periodically during treatment for hydration and nutrition, as well as for electrolyte balance. These drugs would not be expected to impact balance, cognitive function, or to cause incontinence so these would not be the priority assessment.

What is the main purpose of the hormones secreted by the hypothalamus? a.Stimulating or inhibiting release of hormones from the pituitary b.Stimulating organs within the body to secrete hormones c.Allowing the secretion of hormones from the hypothalamus d.Stimulating other glands to release hormones

Ans: A Feedback: The hypothalamus uses various hormones or factors to either stimulate or inhibit the release of hormones from the anterior pituitary. These do not stimulate other organs, the hypothalamus, or other glands to release hormones.

What hormone does the posterior pituitary gland store and release? a.Antidiuretic hormone (ADH) b.Follicle-stimulating hormone (FSH) c.Growth hormone (GH) d.Thyroid-stimulating hormone (TSH)

Ans: A Feedback: The posterior pituitary stores two hormones produced in the hypothalamus: ADH, also known as vasopressin and oxytocin. The posterior pituitary does not store FSH, GH, or TSH.

The nurse administers desmopressin to treat a client with diabetes insipidus. What change in the client's laboratory studies would indicate the drug is working? a.Increased serum sodium levels b.Increased red blood cell count c.Decreased urine specific gravity d.Reduced urine glucose levels

Ans: A Feedback: Patients with diabetes insipidus (DI) produce large amounts of dilute urine with a decrease in serum sodium levels. Administering desmopressin would reduce urine output and allow sodium levels to rise. Urine specific gravity would increase as the urine production slows and urine becomes more concentrated. There is no impact on red blood cell production with diabetes insipidus or its treatment. The urine of a patient with DI does not contain glucose.

The nurse would administer desmopressin cautiously, with close monitoring, to the client with what comorbidities? Select all that apply. a.Hyponatremia b.Asthma c.Severe renal dysfunction d.Gastrointestinal (GI) disease e.Epilepsy

Ans: A, B, E Feedback: Caution should be used with any known vascular disease because of its effects on vascular smooth muscle, epilepsy, asthma, and with hyponatremia, which could be exacerbated by the effects of the drug. The drug is contraindicated and should not be used in patients with severe renal dysfunction. GI disease is not a caution or contraindication.

For what purposes might the nurse administer a hypothalamic hormone? Select all that apply. a. Diagnostic testing b. Synthesis of growth factors c. Prevent aging d. Treating some forms of cancer e. Adjuncts in fertility programs

Ans: A, D, E Feedback: The hypothalamic hormones are not all available for pharmacological purposes. Those available are used mostly for diagnostic testing, for treating some forms of cancer, or as adjunctive therapies in fertility programs. They would not be used to prevent aging or for the synthesis of growth factor.

The parent of a child awaiting a renal transplant asks the nurse whether growth hormone could be effective for the child. What is the nurse's best response? a."Growth hormone is only used to treat short stature in children who do not produce enough growth hormone naturally." b."Growth hormone has been used successfully in children with renal failure but there is a lot to consider and you should talk to the doctor about it." c."Administration of growth hormone requires frequent injections so it would be very painful for a child with existing medical problems." d."You do not want to use growth hormone if you do not have to, because there are serious adverse effects that go along with it?"

Ans: B Feedback: Growth hormone therapy is used with children with renal failure but the decision carries risks as well as benefits so it is important for the mother to talk to the child's primary physician and nephrologist to get recommendations for its safety. It is not just used in children with inadequate growth hormone secretion. It is never correct for the nurse to castigate the mother for a question so telling her it would be cruel or that she should not use it if it is not essential would destroy the patient/nurse relationship.

A client is brought to the emergency department after a motor vehicle accident. The client is hemorrhaging, indicating that which hormone is being secreted to restore blood volume? a.Growth hormone (GH) b.Follicle-stimulating hormone (FSH) c.Antidiuretic hormone (ADH) d.Adrenocorticotropic hormone (ACTH)

Ans: C Feedback: ADH possesses antidiuretic, hemostatic, and vasopressor properties. During hemorrhage, GH, FSH and ACTH are not involved in blood volume restoration.

A client is taking leuprolide to treat prostatic cancer. The nurse caring for this client should monitor most closely for what? a.Diarrhea b.Urinary retention c.Peripheral edema d.Increased appetite

Ans: C Feedback: Peripheral edema is an identified adverse effect of leuprolide therapy. Constipation not diarrhea; urinary frequency not urinary retention; and anorexia not increased appetite are also identified adverse effects.

he nurse is asked to explain how to administer somatropin to the parent of a 6-year-old. How would the nurse explain how this drug is administered? a."It requires only a very small needle and doesn't hurt much at all." b."There will no longer be any need to rotate sites because it uses a needleless system." c."This system will be used until your son gets older and is more tolerant of needles." d."This delivers a fine mist through the skin without needles and far less discomfort."

Ans: D Feedback: Saizen uses the cool.click delivery system, which is a neon-colored, needle-free system that delivers the drug through the skin using a fine mist. Tests have shown a bioequivalency of this method with standard injection techniques, and the young patients who must use this drug are much less resistant to the dosing. There are no needles but site of injection still require rotation to avoid skin damage. The child does not have to go back to a needle system when he gets older.

The nurse is working with a child who has impaired growth due to a deficiency of endogenous growth hormone. What change in the client's health status would contraindicate the safe and effective use of somatropin? a.The client has begun playing organized sports b.The client is developing increased amounts of abdominal fat c.The client has reached 5 feet in height d.The client's epiphyses have begun to close

Ans: D Feedback: Somatropin is contraindicated in the presence of closed epiphyses; it can be given at any time before closure of the epiphyses. Height does not impact when the drug can be given, so long as the epiphyses remain open. Similarly, participation in sports and increased adipose tissue do not mean that somatropin cannot be given safely.

A 55-year-old client has been diagnosed with coronary artery disease and begun antiplatelet therapy. The client has asked the nurse why a "blood thinner like warfarin" hasn't been prescribed. What is the most likely rationale for the clinician's use of an antiplatelet agent rather than an anticoagulant?

Antiplatelet agents are more effective against arterial thrombosis; anticoagulants are more effective against venous thrombosis.

A client who began treatment for type 2 diabetes 8 months ago is now meeting with a diabetic nurse for a scheduled follow-up. How can the nurse best assess the client's glycemic control since beginning treatment? -Arrange to have the client's random blood glucose measured. -Dialogue with the client about implemented management strategies. -Assess the most recent hemoglobin A1C levels. -Review and discuss the data contained in the client's written blood glucose log.

Assess the most recent hemoglobin A1C levels

What is the most accurate method for preventing adverse effects associated with the use of digoxin?

Assess the patient's heart rate and hold administration if it is less than 60.

A 9-year-old female client has been prescribed nafarelin 1,600 mcg nasal spray daily. What assessment should the nurse prioritize when following up this client's treatment? a.Assessing for signs and symptoms of excess fluid volume b.Assessing the client's cognition and orientation c.Assessing the client's height and body mass index d.Assessing for the development of secondary sex characteristics

Assessing for the development of secondary sex characteristics

An adult client's presentation is suggestive of ascaris infection and anthelmintic medications are being considered by the care team. While awaiting the results of diagnostic testing, what assessment would best address the client's risk factors for this infection?

Assessment of the client's recent travel history

A female client presents to the emergency department with nausea, vomiting, and a heart rate of 45 beats per minute. Her husband states that she takes digoxin, Lasix, and nitroglycerin for chest pain. Laboratory results confirm digoxin toxicity. The nurse would expect the health care provider to order what medication to treat the bradycardia?

Atropine

A female client presents to the emergency department with nausea, vomiting, and a heart rate of 45 beats per minute. Her husband states that she takes digoxin, Lasix, and nitroglycerin for chest pain. Laboratory results confirm digoxin toxicity. The nurse would expect the physician to order what medication to treat the bradycardia?

Atropine

When caring for a patient who has been digitalized for his heart failure, the nurse observes that the patient is experiencing bradycardia. Which drug should be administered to patients who develop bradycardia?

Atropine

When caring for a patient who has been digitalized for his heart failure, the nurse observes that the patient is experiencing bradycardia. Which of the following drugs should be administered to patients who develop bradycardia?

Atropine

A client who currently works in Southeast Asia has been treated for schistosomiasis using praziquantel. What should the nurse teach the client about preventing reinfection?

Avoid entering bodies of fresh water to swim or wade

A nurse is assigned to administer glargine to a patient at a health care facility. What precaution should the nurse take when administering glargine? -Administer insulin that has been refrigerated. -Administer glargine via IV route. -Avoid mixing glargine with other insulin. -Shake the vial vigorously before withdrawing insulin.

Avoid mixing glargine with other insulin

A patient is experiencing nausea and visual disturbances when taking digoxin (Lanoxin). Which medication will be administered?

Digoxin immune fab (Digibind)

A client who has been taking cyclosporine to prevent rejection of a kidney transplant has had diltiazem ordered. What is the nurse's best action? a. Assess the client for signs and symptoms of diltiazem toxicity b. Speak to the prescriber about the potential for elevated or toxic cyclosporine levels C. Monitor the client closely for signs or symptoms of kidney rejection d. Ensure the client's cyclosporine is administered at least 2 hours before or after diltiazem

B

An older adult client who is taking metformin has just been seen in the clinic. The health care provider has prescribed metoprolol for angina. What assessment data should the nurse prioritize due to this drug combination? a. White cell differential b. Blood glucose C. Urine specific gravity d. Intake and output

B

The client, diagnosed with angina, reports chest pain. There is an order for oral sublingual nitroglycerin as needed. What action should the nurse take? a. Place two nitroglycerin tablets under the client's tongue and call the provider b. Place one tablet under the client's tongue and repeat every 5 minutes to a maximum of three tablets until pain has been relieved C. Have the client swallow a tablet with a full glass of water and repeat in 10 minutes d. Apply a nitroglycerin transdermal patch to the client's back

B

The nurse is caring for a client with angina who has been prescribed nitroglycerin. What health teaching about the client's lifestyle should the nurse provide? a. "'It's important not to take nitroglycerin unless you're actually experiencing chest pain." b. "It's best to eliminate or reduce your intake of alcoholic beverages. C. "Avoid drinking grapefruit juice because it will interfere with how your body processes nitroglycerin." d. "Make sure that you're able to get at least 6 hours of sleep each night, if possible."

B

The nurse is caring for a client with angina who is taking a calcium channel blocker. What adverse effects would the nurse caution this client about? a.Hypertension and tachycardia b.Headache and dizziness c.Itching and rash d.Nausea and diarrhea

B

The nurse is working with a client who is taking nadolol for angina. What nursing action should the nurse perform to best address the likely adverse effects? a. Ensure the client sips water throughout the day to relieve dry mouth b. Spread out the client's activities to prevent fatigue or overexertion C. Administer stool softeners as prescribed d. Cue the client to void on a scheduled basis at the beginning of therapy

B

What statements by the 54-year-old client indicates an understanding of the nurse's teaching about how to take sublingual nitroglycerin? a. "A headache means a toxic level has been reached." b. "I can take up to three tablets at 5-minute intervals." C. "I can take the drug as I require it because it is not habit forming." d. "If I become dizzy after taking the medication, I should stop taking it."

B

The nurse is preparing to administer sublingual nitroglycerin to a client for the first time and warns that the client may experience what right after administration? a.Nervousness or paresthesia b.Throbbing headache or dizziness c.Drowsiness or blurred vision d.Tinnitus or diplopia

B Feedback: Headache and dizziness commonly occur at the start of nitroglycerin therapy. When administering nitroglycerin, the nurse must use caution to avoid self-contamination, especially with the topical paste formulation because the nurse can experience the same symptoms. However, the patient usually develops a tolerance. Nervousness, paresthesia, drowsiness, blurred vision, tinnitus, and diplopia do not occur as a result of nitroglycerin therapy.

A client asks the nurse how long to use oxymetazoline (Afrin) nasal spray for nasal congestion. What is the nurse's best response? A) "Over-the-counter medications are safe for long-term use." B) "Consult with your primary care provider." C) "Afrin can be used as long as nasal burning and stinging do not occur." D) "Use the medication until your symptoms are completely relieved."

B) "Consult with your primary care provider." Explanation: Overuse of topical nasal decongestants, such as Afrin, can make the symptoms worse, causing rebound congestion. The client should consult with the primary care provider. Not all over-the-counter medications are safe for long-term use. Afrin should not be used long term without nasal burning or stinging, or until symptoms are relieved. The primary care provider should be notified to provide medication use information.

The nurse has finished teaching a 15-year-old client how to use an inhaler to treat asthma. What statement by the client suggests an understanding of the teaching? A) "I should take a deep breath, hold it while I administer the medication, and then exhale." B) "I need to shake the inhaler well before taking the medication." C) "I need to take three short, quick breaths to inhale the medication." D) "I need to wait at least 30 minutes after the first inhalation before taking a second."

B) "I need to shake the inhaler well before taking the medication." Explanation: Just before each use, the client should shake the inhaler well. After shaking, proper technique involves exhaling before placing the inhaler in the mouth; taking a slow, deep breath while delivering the medication into the mouth; and holding the breath for approximately ten seconds before exhaling slowly. A subsequent dose can be administered within a few minutes of the first.

The nurse is providing education to a client who has been prescribed griseofulvin for a tinea infection. What statement would the nurse include in the teaching? A) "Take this medication at bedtime because it makes most people quite drowsy." B) "Make sure to take your medication with food, preferably with a high-fat meal." C) "Avoid grapefruit juice until you have finished your treatment." D) "You need to take this medication only once per week, but it's important that it be on the same day every week."

B) "Make sure to take your medication with food, preferably with a high-fat meal." Explanation: It is necessary to administer griseofulvin with meals to decrease gastric upset. Greater absorption occurs when the drug is given with a high-fat meal. It does not produce drowsiness, and it is not taken on a weekly basis.

The nurse is providing education to a client who has been prescribed albuterol. What adverse reaction should the nurse discuss during teaching? A) Polydipsia B) Tachycardia C) Hypotension D) Diarrhea

B) Tachycardia Explanation: The symptoms of cardiac stimulation, including tachycardia, are noted with the administration of albuterol. The client will not experience polydipsia, hypotension, or diarrhea because of the effects of albuterol

The nurse is discussing the use of pseudoephedrine with parents of children of various ages. What statement made by a parent demonstrates an understanding of the safe administration of a decongestant? A) "Our 3 year old can be given pseudoephedrine but only in liquid form." B) "Only my children who are 12 years of age or older can be given the extended-release form of pseudoephedrine." C) "It's good to know that pseudoephedrine is safe except for children with chronic respiratory issues." D) "Over-the-counter use of pseudoephedrine is considered as safe as prescription pseudoephedrine in children 2 years of age and older."

B) "Only my children who are 12 years of age or older can be given the extended-release form of pseudoephedrine." Explanation: The extended-release tablets of pseudoephedrine should not be administered to children younger than 12 years of age. Overall, pseudoephedrine appears to be effective in children older than 4 years of age, but the Food and Drug Administration (FDA) does not recommend OTC use of the drug in this age group due to the risk of serious and life-threatening adverse effects, including seizures, decreased level of consciousness, tachycardia, and death. This information makes all the other options incorrect.

A client is prescribed salmeterol as treatment for exercise-induced asthma. The nurse would instruct the client to use the inhaler at which time? A) Immediately after beginning to exercise B) 30 minutes before engaging in exercise C) Right before and after exercising D) Midway during the exercise routine

B) 30 minutes before engaging in exercise Explanation: To prevent an exercise-induced asthma attack, the client should take one inhalation of salmeterol 30 minutes before starting exercise. The options would not be effective in preventing an attack.

A client is admitted after overdose with acetaminophen. What medication would the nurse expect to be administered? A) Oxymetazoline hydrochloride B) Acetylcysteine C) Daptomycin D) Darbepoetin alfa

B) Acetylcysteine Explanation: Acetylcysteine is effective in the treatment of acetaminophen overdose. Oxymetazoline hydrochloride is used for nasal congestion, not acetaminophen overdose. Daptomycin is an antibiotic that fights bacteria in the body and used to treat bacterial infections of the skin and underlying tissues. Darbepoetin alfa injection causes the bone marrow to produce red blood cells.

What home remedies are effective for mouth dryness and cough? (Select all that apply.) A) Administration of over-the-counter antihistamine B) Adequate fluid intake C) Humidification of the environment D) Sucking on hard candy or throat lozenges D) Swishing the mouth with astringent mouthwash.

B) Adequate fluid intake C) Humidification of the environment D) Sucking on hard candy or throat lozenges Explanation: An adequate fluid intake, humidification of the environment, and sucking on hard candy or throat lozenges can help relieve mouth dryness and cough. The use of astringent mouthwash will only increase mouth dryness.

The client has been diagnosed with asthma and is being treated with two inhalers, albuterol and flunisolide. The nurse teaches the client that the two medications should be administered in what order? A) Flunisolide first, wait five minutes and then follow with albuterol B) Albuterol first, wait five minutes and then follow with flunisolide C) Flunisolide followed immediately by albuterol D) Albuterol followed immediately by flunisolide

B) Albuterol first, wait five minutes and then follow with flunisolide Explanation: Albuterol is a bronchodilator and should be used first, given time to open the airways, and then the inhaled corticosteroid is administered.

A nurse is caring for a client who asks why she has been prescribed posaconazole following kidney transplantation. The nurse responds that posaconazole is used as prophylaxis to prevent: A) blastomycosis. B) Aspergillus infection. C) mucormycosis. D) coccidioidomycosis.

B) Aspergillus infection. Explanation: Posaconazole is an azole with activity against Candida and Aspergillus infections.

Which of the following would a nurse identify as a surfactant? A) Cromolyn B) Beractant C) Zileuton D) Theophylline

B) Beractant Explanation: Beractant is a lung surfactant. Cromolyn is a mast cell stabilizer. Zileuton is a leukotriene receptor antagonist. Theophylline is a xanthine.

A nurse is caring for a patient who has been prescribed a nasal decongestant. Which of the following factors should the nurse assess before administering the drug? A) Motor coordination B) Blood pressure C) Vision D) Auditory tests

B) Blood pressure Explanation: As a pre-administration assessment before giving a nasal decongestant, the nurse should assess the patient's blood pressure, pulse, and respiratory rates. Assessments for disturbed coordination, blurred vision, or auditory tests may be done as an ongoing assessment to check for signs of adverse effects.

With which condition are the bronchial epithelial cells replaced by a fibrous scar tissue? A) Asthma B) Bronchiectasis C) Bronchitis D) Pneumonia

B) Bronchiectasis Explanation: Bronchiectasis is a chronic disease characterized by dilation of the bronchial tree and chronic inflammation of the bronchial passages. The chronic inflammation leads to replacement of the bronchial epithelial cells by fibrous scar tissue. Asthma is an obstructive disorder characterized by reversible bronchospasm, inflammation, and hyperactive airways. Bronchitis is an acute inflammation of the bronchi. Pneumonia is an inflammation of the lungs.

A client develops itching and burning of the vaginal vault while taking an anti-infective to treat strep throat. What fungal agent has caused the burning and itching? A) Cryptococcus neoformans B) Candida albicans C) Aspergillus D) Dermatophytes

B) Candida albicans Explanation: Growth of Candida organisms is normally restrained by intact immune mechanisms and bacterial competition of nutrients. When these restraining forces are altered (antibacterial drug therapy), fungal overgrowth and opportunistic infection can occur. Cryptococcus neoformans organisms evade normal immune defense of phagocytosis. Aspergillus organisms produce protease. Dermophyte grow on cool body surfaces.

A nurse would expect to increase the dosage of theophylline if the client has a current history of which of the following? A) Hyperthyroidism B) Cigarette smoking C) Gastrointestinal upset D) Alcohol intake

B) Cigarette smoking Explanation: Nicotine increases the metabolism of xanthines; therefore, an increased dosage would be necessary. Hyperthyroidism, gastrointestinal, upset or alcohol intake requires cautious use of the drug because these conditions may be exacerbated by the systemic effects of the drug. The drug dosage may need to be decreased in these situations.

A patient is experiencing nausea and visual disturbances when taking digoxin (Lanoxin). Which of the following medications will be administered?

Digoxin immune fab (Digibind)

x After 3 weeks of drug therapy for tuberculosis (TB), a client is not showing reduction of symptoms. Repeated laboratory work shows positive cultures. What issues should the nurse consider? Select all that apply. A) Defective medication B) Client noncompliance C) Intermittent administration D) Drug resistance E) Misdiagnosis

B) Client noncompliance D) Drug resistance Explanation: Adequate drug therapy of clients with active disease usually produces improvement within 2 to 3 weeks. Intermittent administration is not recommended for multidrug-resistant TB. Defective medication should not be a factor unless indicated by the Food and Drug Administration in a bulletin. Medication therapy would not be initiated unless the diagnosis of TB had been confirmed.

A client is to be started on an antibiotic. Which is most important to take into consideration before beginning the antibiotic regimen? A) Duration of symptoms B) Culture and susceptibility C) Client's hydration status D) Client's age and weight

B) Culture and susceptibility Explanation: Culture identifies the causative organism, and susceptibility tests determine which drugs are likely to be effective against the organism. The duration of symptoms and the client's hydration status, age, and weight are important, but not imperative, in determining the antibiotic of choice.

A client with an upper respiratory tract infection was prescribed roxithromycin, an antibiotic. The nurse tells the client that irregular administration of this medication could lead to what outcome? A) Decreased absorption of the drug B) Development of drug resistance C) Increased rate of elimination of the drug D) Increased chances of serious adverse events

B) Development of drug resistance Explanation: When there is irregular or indiscriminate use of antibiotics, certain pathogens may mutate or build a tolerance to the drug. The antibiotic then becomes ineffective against that organism. Irregular use does not affect the absorption or elimination of the drug or increase chances of serious adverse events of the antibiotic.

Which mechanism is involved in the movement of oxygen and carbon dioxide at the alveolar level? A) Active transport B) Diffusion C) Facilitated diffusion D) Osmosis

B) Diffusion Explanation: At the alveolar level, oxygen and carbon dioxide move via diffusion. Active transport involves the use of energy to move substances. Facilitated diffusion requires the use of a carrier molecule. Osmosis refers to the movement of water.

A client is taking phenelzine (Nardil), a monoamine oxidase inhibitor, for depression. The client started taking dextromethorphan (Robitussin) over the counter without consulting his physician. The use of these two medications is contraindicated. Which symptoms might the patient exhibit? (Select all that apply.) A) Hypertension B) Fever C) Coma D) Constipation D) Shortness of breath

B) Fever C) Coma Explanation: Co-administration of dextromethorphan and a monoamine oxidase inhibitor may result in hypotension, fever, nausea, leg jerking, and coma.

A client presents at the clinic with a dry, nonproductive cough. The client is diagnosed with bronchitis, and it is determined that they will need help thinning sputum so that the cough can become productive. What does the nurse expect will be prescribed for the client? A) Codeine B) Guaifenesin C) Dextromethorphan D) Aspirin

B) Guaifenesin Explanation: Expectorants, such as guaifenesin, are agents given orally to liquefy respiratory secretions and allow for easier removal. Dextromethorphan and codeine are antitussives used to suppress coughing.

A nurse is caring for a 29-year-old man who is being treated for meningitis. He is receiving flucytosine (5-FC). Which will the nurse monitor weekly? A) Blood pressure B) Hematologic status C) Visual acuity D) Blood glucose

B) Hematologic status Explanation: Flucytosine is toxic to rapidly proliferating tissues, such as the bone marrow and the lining of the GI tract. Therefore, common hematologic adverse effects include anemia, leukopenia, and thrombocytopenia. The nurse will also monitor renal and hepatic status. Abnormal blood pressure, visual impairment, and changes in blood glucose levels are not identified adverse effects of flucytosine.

A client experiences bronchospasm with asthma. The nurse understands that this is due to: A) Cytokines B) Histamine C) Norepinephrine D) Serotonin

B) Histamine Explanation: The bronchospasm associated with asthma is due to the immediate release of histamine.

A nurse is providing health education to a client recently diagnosed with asthma and prescribed albuterol and ipratropium. Which of the client's statements suggests a need for clarification by the nurse? A) "I'll try to make sure that I drink plenty of fluids each day." B) I'll keep taking my medications until I'm not experiencing any more symptoms." C) "I'll make sure I don't take my inhalers more often than they've been prescribed." D) "I'm a heavy coffee drinker, but I know that I now know I need to cut down on this."

B) I'll keep taking my medications until I'm not experiencing any more symptoms." Explanation: Antiasthma medications should normally be taken on a regular schedule, not solely based on immediate symptoms. They should not be discontinued in the absence of symptoms. Increasing fluid intake, limiting caffeine, and adhering to the administration schedule are correct actions. Reference:

The pharmacology instructor is discussing ways to decrease the incidence and severity of adverse effects among children taking anti-infective agents. What would the instructor mention is one way to do this in children? A) Decrease dietary potassium. B) Keep clients well hydrated. C) Assess liver function weekly. D) Increase sodium in diet.

B) Keep clients well hydrated. Explanation: Because children can have increased susceptibility to the gastrointestinal and nervous system effects of anti-infectives, monitor hydration and nutritional status carefully. Clients should be encouraged to drink fluids. Decreasing dietary potassium, weekly assessment of liver function, or increasing sodium in the diet will not decrease the severity of adverse effects in children.

The pharmacology instructor is discussing ways to decrease adverse effects on clients taking aminoglycosides. Which would the instructor include? A) Decreasing dietary potassium B) Keeping the client well hydrated C) Assessing liver function weekly D) Increasing dietary sodium

B) Keeping the client well hydrated Explanation: Aminoglycosides are nephrotoxic. Keeping the client adequately hydrated (by increasing daily fluid intake to two to three liters) helps decrease the risk of adverse effects related to renal function. Renal function tests should be performed regularly during aminoglycoside therapy.

Based on the nurse's knowledge about thyroid hormone supplements, the nurse administers levothyroxine cautiously to a client who has recently had which condition or disorder? -myocardial infarction -hypoglycemic episode -cataract surgery -seizure

myocardial infarction

What information about antibiotic therapy should the nurse include in a client's medication education? A) Bacteria can remain in the body for 48 to 72 hours after the completion of antibiotic therapy. B) Maintenance of normal bacterial flora is essential to health during the therapy. C) The healthy human body hosts dozens of minor infections at any given time. D) An aseptic internal environment only exists in young adults.

B) Maintenance of normal bacterial flora is essential to health during the therapy. Explanation: Normal flora protects the human host in a variety of ways. For example, normal bowel flora synthesizes vitamin K and vitamin B complex. The intestinal flora also plays a role in digestion. Furthermore, by competing with potential pathogens for nutrients and by preventing adhesion and growth of pathogens, beneficial microorganisms interfere with the ability of potential pathogens to cause infections. None of the other options are accurate statements regarding antibiotic therapy.

A female client calls the pediatrician's office for a suggestion regarding the best over-the-counter cough and cold medicine for her 6-month-old child. The pediatrician advises against the medication for what reason? A) The medication is not effective for croup. B) Misuse could result in overdose. C) The medication is contraindicated for the child's symptoms. D) The medication is contraindicated for viral infections.

B) Misuse could result in overdose. Explanation: Several OTC cough and cold medicines for use in infants have been recalled voluntarily due to concerns about possible misuse that could result in overdoses.

A 75-year-old patient with a history of renal impairment is admitted to the primary health care center with a UTI and has been prescribed a cephalosporin. Which of the following interventions is most important for the nurse to perform when caring for this patient? A) Monitoring fluid intake. B) Monitoring blood creatinine levels. C) Testing for occult blood. D) Testing for increased glucose levels.

B) Monitoring blood creatinine levels. Explanation: An elderly patient is more susceptible to the nephrotoxic effects of the cephalosporins. Since renal impairment is present, it is important for the nurse to closely monitor the patient's blood creatinine levels. The nurse should conduct a test for occult blood if blood and mucus occur in the stool and monitor the fluid intake if there is a decrease in urine output. The nurse does not need to monitor for increased glucose levels unless the patient has a history of diabetes.

A newborn, experiencing ineffective alveolar expansion, is receiving treatment. What intervention should the nurse implement to assure the administration of the prescribed medication has been effective? A) Suctioning the infant every 30 minutes for 2 hours after the treatment has been initiated B) Monitoring respirations for bilateral chest movement C) Changing the newborn's position frequently to assure effective distribution of medication D) Supply supplemental oxygen as prescribed

B) Monitoring respirations for bilateral chest movement Explanation: Ensure proper placement of the endotracheal tube with bilateral chest movement and lung sounds to provide adequate delivery of the drug. Suctioning should be avoided for at least 2 hours after the surfactant is instilled into the newborn's endotracheal tube. Neither the delivery of supplemental oxygen nor position changes are related to the delivery of the medication into the infant's lungs.

Which of the following should a nurse carefully monitor in a patient who has been administered cephalosporin as well as aminoglycosides for a wound infection? A) Nausea B) Nephrotoxicity C) Increased bleeding D) Respiratory difficulty

B) Nephrotoxicity Explanation: When cephalosporin is administered with aminoglycosides, it increases the risk for nephrotoxicity and should be closely monitored. Nausea is an adverse reaction of cephalosporins in patients with gastrointestinal tract infection. The risk of bleeding increases when cephalosporin is administered with oral anticoagulants. Risk for respiratory difficulty increases if alcohol is consumed within 72 hours after certain cephalosporin administration.

To ensure that the most appropriate drug is being used to treat a pathogen, which would need to be done first? A) Using combination therapy B) Obtaining sensitivity testing C) Checking client allergies D) Evaluating the bactericidal effects

B) Obtaining sensitivity testing Explanation: Performing sensitivity testing on cultured microbes is important to evaluate the bacteria and determine which drugs are capable of controlling the particular organism. Once the sensitivity testing is completed, then the decision for the drug can be made. Combination therapy is used when appropriate after culture and when sensitivity testing has been completed. Checking client allergies also would be done after sensitivity testing but before administering the drug. The bactericidal effects of a drug may or may not play a role in the selection of the drug.

What is an adverse reaction commonly experienced by people taking nasal decongestants? A) Diarrhea B) Rebound congestion C) Rash D) Headache

B) Rebound congestion Explanation: An adverse effect that accompanies frequent or prolonged use of these drugs is a rebound congestion, officially called rhinitis medicamentosa.

The nurse is administering vancomycin to a client and will monitor the client for adverse effects related to which body systems? (Select all that apply.) A) Cardiac B) Renal C) Hepatic D) Hearing E) Eyesight

B) Renal D) Hearing Explanation: Vancomycin should not be administered to clients with renal or hearing impairment, because this drug may cause increased problems in these areas. It is not known to aggravate cardiac, hepatic, or vision problems.

A client is started on sulfamethoxazole-trimethoprim for a urinary infection. What adverse effect should the nurse assess with this client? A) Liver toxicity B) Renal damage C) Bone marrow depression D) Congestive heart failure

B) Renal damage Explanation: Renal impairment is a possible side-effect to the administration of SMZ-TMP. It does not cause liver toxicity, bone marrow depression, or congestive heart failure.

A gerontological nurse has encouraged a group of caregivers who work with older adults to avoid administering first-generation H1 receptor antagonists to their clients. The nurse's cautionary message is an acknowledgment of what possible nursing diagnosis? A) Risk for infection related to adverse effects of antihistamines B) Risk for falls related to sedation C) Risk for deficient fluid volume related to diuresis D) Risk for impaired skin integrity related to urticaria

B) Risk for falls related to sedation Explanation: First-generation antihistamines cause drowsiness and impaired cognition that can result in falls. These drugs are not known to cause diuresis, impaired skin integrity, or infection.

The health care provider considers prescribing an oral tetracycline for a client suspected of having a urinary tract infection. What action needs to be completed before the medication can be prescribed? A) The client should be assessed for penicillin allergy to rule out cross-allergenicity. B) The health care provider should order culture and susceptibility studies. C) The client should be warned of the risk for fever. D) The client should be taught how to store the medication appropriately.

B) The health care provider should order culture and susceptibility studies. Explanation: Culture and susceptibility studies are needed before tetracycline therapy is started, because many strains of organisms are either drug resistant or vary greatly in drug susceptibility. Teaching the client about proper storage and adverse effects, including photosensitivity, are appropriate after the medication is ordered. There is no cross-allergenicity with penicillin, so prescribing a tetracycline does not require assessment penicillin hypersensitivity.

A client, being treated for latent tuberculosis (TB) on an out-client basis, tells the nurse, "I've been feeling pretty good lately, so I haven't actually been all that consistent with taking my drugs." Subsequent health education by the nurse should focus on what subject? A) The fact that nonadherence to treatment exacerbates the risks of adverse effects B) The need to consistently take the prescribed drugs in order to cure TB C) The need to match drug dosages carefully to signs and symptoms D) The fact that nonadherence will necessitate the use of antiretrovirals

B) The need to consistently take the prescribed drugs in order to cure TB Explanation: Consistent adherence to treatment is imperative to ensure successful treatment of TB. Nonadherence leads to resistance and unsuccessful treatment, not increased adverse effects. Nonadherence has no relationship with matching dosage with signs and symptoms. Antivirals are ineffective against TB.

A 30-year-old woman who is in the first trimester of pregnancy has presented to her primary care provider with a 4-day history of a reddened, itchy left eye that is crusted with purulent exudate. The clinician suspects a bacterial, rather than viral, etiology. How will the client's pregnancy affect the potential use of ciprofloxacin to treat her conjunctivitis? A) Ciprofloxacin is safe to use in pregnancy and the client may use to same dose and route as a nonpregnant client. B) The use of ciprofloxacin is contraindicated in pregnancy. C)It is safe for the client to use topical ciprofloxacin but the oral route is potential teratogenic. D) The client will require a lower dose and longer course of ciprofloxacin than a nonpregnant, adult client.

B) The use of ciprofloxacin is contraindicated in pregnancy. Explanation: Ciprofloxacin is contraindicated in clients who are pregnant or lactating. Alternative routes and dosages do not mitigate the risks during pregnancy.

The nurse should advise the client to avoid taking which medication at the same time as a fluoroquinolone? A) antihypertensives B) antacids C) antidiabetic agents D) oral contraceptives

B) antacids Explanation: The client should not take antacids or drugs containing iron or zinc at the same time as taking a fluoroquinolone because these drugs will decrease the absorption of the fluoroquinolones. There are no listed contraindications of giving fluoroquinolones with antihypertensives, antidiabetic agents, or oral contraceptives.

The nursing instructor is teaching about antitussives and their side effects. What ingredient found in some antitussives does the instructor tell students can cause drowsiness? A) eucalyptus B) antihistamine C) acetaminophen D) Advil

B) antihistamine Explanation: Antitussives that contain an antihistamine may cause drowsiness. The others are not found in antitussives and therefore do not cause the adverse reaction of drowsiness.

A client prescribed rifaximin for diarrhea has developed frank bleeding in the stool. What intervention should the nurse anticipate being implemented to best ensure client safety? A) increasing the dose of rifaximin B) changing to a different antibiotic C) supplementing the antibiotic with vitamin K D) changing to parenteral administration of rifaximin

B) changing to a different antibiotic Explanation: Because of its very limited systemic absorption (97% eliminated in feces), health care providers cannot use rifaximin to treat systemic infections, including infections due to invasive strains of E. coli. Therefore, diarrhea occurring with fever or bloody stools requires treatment with alternative agents. Changing the route or supplementing with vitamin K will not aid in treatment.

A client reports to the nurse, "My head is stuffed up and I can't breathe through my nose." Nasal discharge is noted upon examination. The client is diagnosed with a common cold. The nurse should prepare to provide medication instructions related to what classification of medication directed at the client's nasal mucosa? A) antihistamine B) decongestant C) antitussant D) expectorant

B) decongestant Explanation: Nasal decongestants relieve nasal congestion and swelling by constricting arterioles and reducing blood flow to nasal mucosa. Antihistamines refer to a type of medication that treats allergy symptoms. The goal of antitussive therapy is to suppress nonpurposeful coughing. Expectorants are agents given orally to liquefy respiratory secretions and allow for their easier removal.

The nurse is assessing a client who has been prescribed tetracycline for possible contraindications to therapy. The nurse understands that what risks are posed by tetracycline therapy during pregnancy? Select all that apply. A) cardiomegaly B) hyperbilirubinemia C) spinal cord malformations D) impaired tooth development E) impaired bone development

B) hyperbilirubinemia D) impaired tooth development E) impaired bone development Explanation: If a fetus or young infant receives a sulfonamide, such as tetracycline, by placental transfer, in breast milk, or by direct administration, the drug displaces bilirubin from binding sites on albumin. Thus, bilirubin may accumulate in the bloodstream (hyperbilirubinemia) resulting in life-threatening toxicity. Tetracycline is generally contraindicated in pregnancy because it may interfere with bone and tooth development in the fetus. Tetracycline therapy during pregnancy is not associated with congenital heart or spinal cord conditions.

A 35-year-old man being treated with isoniazid (INH) for exposure to TB has this medical history: diabetes mellitus type 2, hypertension, hyperlipidemia, and coronary artery disease. He drinks one to two glasses of wine on the weekend and smokes two packs of cigarettes per day. Which adverse reaction should the nurse alert the client to report to his health care provider right away? A) changes in his bowel elimination pattern B) numbness and tingling in his feet C) a slight decrease in his blood glucose levels D) a persistent cough since starting the medication

B) numbness and tingling in his feet Explanation: INH should also be given with caution to clients with diabetes mellitus, malnutrition, or alcoholism because its effects (antagonism or increased excretion) on pyridoxine (vitamin B6) can cause peripheral neuropathy in these clients. Pyridoxine may be given concurrently with INH to decrease the risk for this adverse effect. Another frequent adverse effect is peripheral neuropathy. This effect may cause paresthesias in the hands and feet. As previously mentioned, malnourished people and those with diabetes and alcoholism have a higher risk for this adverse effect.

A client is receiving levofloxacin (Levaquin) IV. During an infusion, the client reports pain and the nurse notes a reddened area along the vein. What does the nurse suspect? A) extravasation B) phlebitis C) allergic reaction D) adverse reaction

B) phlebitis Explanation: Tenderness, pain, and redness along the vein may indicate phlebitis or thrombophlebitis. Extravasation occurs when the fluids leak into the surrounding tissue. Adverse reactions and allergic reactions do not occur at the IV site.

A high school student was diagnosed with asthma when he was in elementary school and has become accustomed to carrying and using his "puffers". In recent months, he has become more involved in sports and has developed a habit of administering albuterol up to 10 times daily. The nurse should teach the student that overuse of albuterol can lead to A) permanent liver damage. B) rebound bronchoconstriction. C) community-acquired pneumonia. D) severe anticholinergic effects.

B) rebound bronchoconstriction. Explanation: Patients who self-administer albuterol may use their MDIs more frequently than recommended. This practice can result in rebound bronchoconstriction, which may motivate the patient to increase MDI use, stimulating the cycle of rebound congestion. Overuse of albuterol is not linked to pneumonia or hepatotoxicity. Albuterol is not an anticholinergic drug.

Which substance would a group of students identify as being responsible for breaking up dietary fats into smaller units?

Bile acids Bile acids act like a detergent in the small intestine and break up fats into small units. These small units are called micelles. High levels of cholesterol are part of bile acids. Chylomicrons are carriers for micelles.

A client is receiving glipizide as treatment for his type 2 diabetes. The nurse understands that this drug acts by: -Inhibiting alpha-glucosidase to delay glucose absorption -Binding to potassium channels on pancreatic beta cells -Increasing the uptake of glucose -Decreasing insulin resistance

Binding to potassium channels on pancreatic beta cells

A client taking warfarin after open heart surgery reports pain in both knees that began this week. The nurse notes bruises on both knees. Based on the effects of the medications and the report of pain, what should the nurse suspect is the cause of the pain?

Bleeding

A client with atrial fibrillation who is receiving oral anticoagulant therapy is receiving atorvastatin. The nurse would monitor this client for which of the following?

Bleeding Increased serum levels and resultant toxicity can occur if a statin is combined with warfarin, an oral anticoagulant. This would increase the client's risk for bleeding. Abdominal pain and cataract development are related to the use of atorvastatin alone. Liver failure also is associated with atorvastatin use alone.

When describing the action of atorvastatin, which of the following would the nurse include?

Blocking the enzyme that is involved in cholesterol synthesis HMG-CoA reductase inhibitors, such as atorvastatin, block the enzyme involved in cholesterol synthesis. Bile acid sequestrants block bile acids to form insoluble complexes for excretion in the feces. Fibrates stimulate the breakdown of lipoproteins from the tissues and their removal from the plasma. Cholesterol absorption inhibitors work in the brush border of the small intestine to decrease absorption of dietary cholesterol from the small intestine.

A patient with atrial fibrillation is receiving warfarin therapy. The patient is also prescribed ezetimibe. The nurse would instruct the patient to watch for which of the following?

Blood in urine or stool Warfarin levels increase with ezetimibe increasing the patient's risk for bleeding. Abdominal pain is a common adverse effect of ezetimibe and is unrelated to the combination therapy. Yellowing of the skin suggest liver dysfunction, which is not associated with the combination therapy. Blurred vision is not associated with the combination therapy.

The nurse knows that which assessment finding suggests hyperthyroidism? -Heart rate 55 beats per minute -Blood pressure 145/87 -Hard, thick nails -Cool, dry skin

Blood pressure 145/87

A home care nurse is caring for a 70-year-old female client who has been diagnosed with osteoporosis. When developing a plan of care for this client, the nurse should include measures to prevent what complication of the disease? -Hypocalcemia -Bone fracture -Bone hardening -Low estrogen levels

Bone fracture

A 5 year-old child has been diagnosed with whipworms and is currently undergoing treatment with mebendazole. What assessments should the nurse prioritize? Select all that apply.

Bowel movements Nutritional status Fluid balance

The ICU nurse is caring for a patient in neurogenic shock. What would the nurse know is a characteristic of neurogenic shock?

Bradycardia Rationale: In neurogenic shock, the sympathetic system is not able to respond to body stressors. Therefore, the clinical characteristics of neurogenic shock are signs of parasympathetic stimulation. It is characterized by dry, warm skin rather than the cool, moist skin seen in hypovolemic shock. Another characteristic is hypotension with bradycardia, rather than the tachycardia that characterizes other forms of shock.

during biotransformation what is the liver responsible for?

Breaking down medication further

How is a drugs halftime affected?

By absorption, distribution, metabolism, and excretion

A nurse is educating a patient with hypertension who is prescribed losartan on the mechanism of action of the drug. Which of the following modes of action helps losartan to bring about its antihypertensive effect?

By blocking the angiotensin II receptors Rationale: Losartan brings about an antihypertensive effect by blocking the angiotensin II receptors. Losartan is an angiotensin II receptor antagonist. By blocking the angiotensin II receptor, the renin-angiotensin system is stopped and consequently blood pressure is reduced. Drugs such as captopril prevent the conversion of angiotensin I. Losartan does not prevent renin secretion. Losartan does not block aldosterone receptors.

A 60-year-old client experienced a sudden onset of chest pain and shortness of breath and was subsequently diagnosed with a pulmonary embolism in the emergency department. The client has been started on an intravenous heparin infusion. How does this drug achieve therapeutic effect?

By inactivating clotting factors and thus stopping the coagulation cascade

A 46-year-old man with a high body-mass index and a sedentary lifestyle has been diagnosed with hypertension by his primary care provider after serial blood pressure readings. The clinician has opted to begin the patient on captopril (Capoten). The patient's nurse should recognize that the therapeutic effect of this drug is achieved in what way?

By inhibiting the transformation of angiotensin I to angiotensin II Rationale: Captopril inhibits the ACE needed to change the inactive angiotensin I to the active form angiotensin II. This reduction of angiotensin II decreases the secretion of aldosterone, thus preventing sodium and water retention. Captopril therefore decreases peripheral vascular resistance and lowers blood pressure. Calcium channel blockers such as verapamil block the movement of calcium ions into arterial smooth muscles and aldosterone blockers such as Eplerenone (Inspra) inhibit aldosterone from binding to mineralocorticoid receptors. ACE inhibitors do not have a direct effect on vascular smooth muscle.

A 49-year-old client is admitted with uncontrolled chest pain. He is currently taking nitroglycerin. His physician orders nifedipine added to his regimen. The nurse should observe the client for What adverse effects? a. Hypokalemia b. Renal insufficiency C. Hypotension d. Hypoglycemia

C

A client with angina has been prescribed nifedipine 15 mg PO t.i.d. The client has received the first two doses of the medication and reports dizziness. What is the nurse's best action? a. Reassure the client that this is an expected adverse effect b. Report this finding to the client's care provider C. Assess the client's blood pressure d. Withhold the next scheduled dose and implement falls precautions

C

A nurse is providing education to a client who has been experiencing unstable angina. What is the nurse's best explanation for this condition? a. "A coronary vessel has become completely plugged and is unable to deliver blood to your heart." b. "The pain is caused by a spasm of a blood vessel, not just from the vessel narrowing. C. There is serious narrowing of a coronary artery that is causing a reduction in oxygen to the heart." d. "Your body's response to a lack of oxygen in the heart muscle is pain."

C

A nurse is teaching the client newly prescribed sublingual nitroglycerin how to take the medication. What action should the nurse instruct the client to do first? a. To check his radial pulse b. To place the tablet in the buccal cavity C. To take a sip of water d. To lie down for 15 minutes before administration

C

An adult client has had symptoms of unstable angina during admission to the hospital. What is the most appropriate nursing diagnosis? a. Deficient knowledge about underlying disease and methods for avoiding complications b. Anxiety related to fear of death C. Ineffective tissue perfusion related to reduced oxygen supply to the heart d. Noncompliance related to failure to accept necessary lifestyle changes

C

The nurse is caring for a client who is reporting chest pain. The nurse is to administer 40 mg of isosorbide dinitrate to the client. What assessment finding would contraindicate the safe use of this drug? a. Platelets 202 * 10° L (202 x 103,UL) b. Orientation to person and place but not time C. Hemoglobin 88 g/L (8.8 g/dL) d. Active sinusitis

C

When providing drug education about nitroglycerin to the client, what should the nurse include in the teaching plan about a nitroglycerin patch? a. "It only has to be changed every 3 days. b. "It is more effective than tablets in treating angina. C. "One patch lasts an entire day. d. "It is faster acting than the tablets

C

When the nurse administers a beta-adrenergic blocker to the client with angina, the nurse expects the drug will help to control angina, but it also has what other effect? a. Increased heart rate b. Increased oxygen consumption C. Decreased strength of heart muscle contraction d. Decreased urinary output

C

A nurse on rounds observes that a client has been eating food rather hastily. Based on this information the nurse knows that what structure of the respiratory system prevents foreign matter from entering the lower respiratory system? A) Trachea B) Esophagus C) Epiglottis D) Pharynx

C) Epiglottis Explanation: The pharynx is divided into three areas: nasopharynx, oropharynx, and laryngopharynx. The epiglottis is a protective flap that covers the trachea during swallowing to prevent foreign matter from entering the respiratory system. The trachea and esophagus are both located in the pharynx.

A nurse is teaching a 15-year-old female client how to use a vaginal preparation of an antifungal agent. Which statement would be important to include in the instruction? A) "Be sure to use a second method of contraception while you're on this medication." B) "Stop applying this medication during menstruation." C) "Administer this medication high into the vagina." D) "Continue using tampons normally."

C) "Administer this medication high into the vagina." Explanation: Vaginal antifungals should be administered high into the vagina (except during pregnancy). During menstruation, clients should continue using these preparations; however, they should use minipads rather than tampons. Clients should avoid sexual intercourse while taking these medications.

An older adult client has reported "seeing things that aren't there" after starting to take over-the-counter dextromethorphan for a nonproductive cough associated with a cold. What assessment question will provide the most relevant information regarding the possible cause of this adverse effect? A) "Have you been nauseated as well since taking the dextromethorphan?" B) "Have you been taking the dextromethorphan at bedtime?" C) "How much dextromethorphan have you been taking each day?" D) "Is the dextromethorphan making you dizzy?"

C) "How much dextromethorphan have you been taking each day?" Explanation: At normal doses, dextromethorphan is known to cause nausea, drowsiness, rash, and difficulty breathing but doses exceeding recommendations can produce hallucinations and disassociation. The question concerning dosage is the most relevant regarding the cause of hallucinations. Reference:

A nurse is teaching a 55-year-old client about fexofenadine prescribed to treat allergic symptoms. Which client statement suggests an understanding of this teaching? A) "Taking this medication shouldn't stop me from having my after-dinner drink." B) "It's safe to take this medication with the nighttime sinus medication that I've been using." C) "I should avoid driving after taking this medication." D) "This medication has no serious side effects."

C) "I should avoid driving after taking this medication." Explanation: After taking fexofenadine or other antihistamines, clients should avoid activities that require being alert (e.g., driving). They should not combine alcohol or other sedating agents with these drugs, since doing so can result in excessive sedation. To avoid the adverse effects of combining multiple antihistamines, clients should consult the prescriber before taking any other medications, particularly cold remedies and products labelled "nighttime" or "PM."

The nurse is caring for a client with cystic fibrosis who is receiving dornase alfa by nebulizer to help thin secretions. What statement by the client indicates a need for further instruction? A) "This medication will loosen up the sticky mucus that is in my lungs." B) "I will try to cough after I receive this medication." C) "I'm glad that this medication will address all of my symptoms." D) "I will continue to use postural drainage and take my enzymes as ordered."

C) "I'm glad that this medication will address all of my symptoms." Explanation: Cystic fibrosis clients who receive dornase alfa should be cautioned about the need to continue all therapies for their cystic fibrosis because dornase alfa is only a palliative therapy that improves respiratory symptoms, and other therapies, such as coughing, postural drainage, and enzymes, are still needed. This medication will not relieve all of the client's symptoms.

The nurse is educating a client diagnosed with a urinary tract infection about the prescribed trimethoprim-sulfamethoxazole (TMP-SMZ). The client has a history of type 2 diabetes and currently takes a sulfonylurea. Which statement made by the client establishes the need for further clarification? A) "I will monitor my blood sugar carefully since the drug may lower it." B) "I will take the drug with 8 ounces of water before or after meals." C) "The drug will not affect the herbal supplement, ginseng that I take." D) "If a rash develops, the drug will be discontinued and my prescriber notified."

C) "The drug will not affect the herbal supplement, ginseng that I take." Explanation: The nurse needs to clarify the statement that the client made in regard to continuing the ginseng because the interaction with TMP-SMZ will result in increasing the effect of the bacteriostatic drug. The other statements are correct. TMP-SMZ interacts with sulfonylurea drug leading to hypoglycemia, so close monitoring of blood sugars is warranted. The drug needs to be taken before or after a meal with at least 8 ounces of water to help prevent crystalluria. The client should take at least 2 to 3 L of noncaffeinated fluids each day to properly hydrate and flush kidneys of wastes. The drug may lead to a serious life-threatening condition, Stevens-Johnson syndrome, which has a sudden development of a flat red rash that forms blisters in the center and areas of peeling skin. The client should stop the drug immediately and notify the prescriber of any rash.

A client is being treated with amphotericin B. Which statement indicates that the client has understood the client teaching? A) "The medication may cause diabetes." B) "The medication will cause liver necrosis." C) "The medication may cause kidney damage." D) "The medication will cause pancreatitis."

C) "The medication may cause kidney damage." Explanation: The main concern with the administration of amphotericin B is the risk of nephrotoxicity. Thus, the statement that the medication may cause kidney damage is the most appropriate. The medication is not known to cause diabetes, liver necrosis, or pancreatitis

Which statement by the parents of a child diagnosed with cystic fibrosis (CF) demonstrates an understand of the goal of the prescribed treatment? A) "He requires periodic surfactant installations to prevent atelectasis from occurring." B) "Putting her on a ventilator for a few days give her lungs a chance to recover from the infection." C) "The secretions in her lungs can't be allowed to get too thick and block her airways." D) "We have to be sure that he isn't exposed to allergens that irritate his lungs."

C) "The secretions in her lungs can't be allowed to get too thick and block her airways." Explanation: Cystic fibrosis (CF) is a hereditary disease that results in the accumulation of copious amounts of very thick secretions in the lungs. Treatment is aimed at keeping the secretions fluid and moving and maintaining airway patency as much as possible. CF treatment goals are not focused on atelectasis prevention, infection control, or the avoidance of allergens.

A client, experiencing respiratory distress related bronchi constriction, will benefit from what therapeutic action provided by anticholinergic medication therapy? A) Reduction of the inflammatory response B) Decrease in the production of leukotrienes D4 and E4 C) Relaxation of smooth muscle D) Enhancement of alveolar expansion

C) Relaxation of smooth muscle Explanation: Anticholinergics can be used as bronchodilators because of their effect on the vagus nerve, resulting in relaxation of smooth muscle in the bronchi, which leads to bronchodilation. None of the other options describe the bronchial dilation affected of the anticholinergic classification of medications.

Which neurological patient is most likely to have abnormalities in breathing regulation? A) A 23-year-old male who has an injury to his frontal lobe following a sports injury B) A 45-year-old female with a spinal cord injury at C4 following a motor vehicle accident C) A 34-year-old male with damage to his pons and medulla oblongata following a blow to the back of the head D) A 66-year-old male with temporal lobe infarcts secondary to a stroke

C) A 34-year-old male with damage to his pons and medulla oblongata following a blow to the back of the head Explanation: The respiratory center is located in the pons and medulla, and damage to the temporal lobe, frontal lobe, or spinal cord at C4 is less likely to affect respiration.

Acetylcysteine may be used as a mucolytic agent. What is an additional indication for the drug? A) Conversion of cardiac arrhythmias B) Treatment of peptic ulcer disease C) Antidote for acetaminophen poisoning D) Treatment of bronchospasm

C) Antidote for acetaminophen poisoning Explanation: Oral acetylcysteine is widely used in the treatment of acetaminophen overdosage.

An adult client has been treated for strep throat with ampicillin by mouth. The client visits the occupational health nurse and reports vaginal itching. What organism is the cause of the vaginal itching? A) Klebsiella B) Enterobacter C) Candida D) Proteus

C) Candida Explanation: The yeast Candida is a normal resident of the vagina and the intestinal tract. An antibacterial drug may destroy the normal bacterial flora without affecting the fungal organism. Klebsiella, Enterobacter, and Proteus will not contribute to the development of a yeast infection.

With bronchitis, proteins leak into the area due to: A) Swelling B) Increased blood flow C) Changes in capillary permeability D) Inflammatory reaction

C) Changes in capillary permeability Explanation: Although an inflammatory reaction occurs leading to swelling and increased blood flow with bronchitis, it is the change in the capillary permeability that allows proteins to leak into the area.

The nurse is monitoring a premature infant diagnosed with respiratory distress syndrome (RDS). Which assessment finding will serve to demonstrate that the instillation of surfactant as a treatment has been effective? A) Return of cough reflex B) Body temperature within normal range C) Clear breath sounds D) Infant is interested in surroundings

C) Clear breath sounds Explanation: Treatment is aimed at instilling surfactant to prevent atelectasis and to allow the lungs to expand. The absence of any abnormal breath sounds would indicate proper expansion of the alveoli. None of the other options are directly associated with the resolution of the collapsed alveoli.

An infant has been brought to the emergency department by the parents, and initial assessment is highly suggestive of bacterial meningitis. Consequently, the infant has been admitted, and empiric antibiotic therapy has been prescribed. The nurse should understand what characteristic of this infant's current treatment plan? A) Success or failure of treatment will not be apparent for several weeks. B) The infant will receive aggressive treatment with a narrow-spectrum antibiotic. C) Culture and sensitivity results of the infant's cerebrospinal fluid are still pending. D) The infant is suspected of having an antibiotic-resistant infection.

C) Culture and sensitivity results of the infant's cerebrospinal fluid are still pending. Explanation: Empiric therapy is based on an informed estimate of the most likely pathogen(s) given the client's signs and symptoms and the site of infection, as well as knowledge of communicable diseases currently infecting other people in the community. Because laboratory tests used to definitively identify causative organisms and to determine susceptibility to antibiotics usually require 48 to 72 hours, the prescriber usually initiates treatment with an antimicrobial drug that is likely to be effective. The other listed statements are not true of empiric therapy.

The nurse is educating an 82-year-old client regarding amphotericin B (Fungizone). The nurse knows the client understand when the client states that he could develop which adverse effect? A) Diabetes B) Liver necrosis C) Damage to his kidneys D) Pancreatitis

C) Damage to his kidneys Explanation: Amphotericin B is nephrotoxic. Amphotericin B does not cause diabetes, liver necrosis, or pancreatitis.

A female client is admitted to the critical care unit with sepsis related to a contaminated central line. The health care provider orders intravenous beta-lactam antimicrobials. The client's current laboratory report reflects renal impairment. What would the nurse expect the provider to do? A) Maintain the drug dose. B) Increase the drug dose. C) Decrease the drug dose. D) Administer the drug via an intramuscular route.

C) Decrease the drug dose. Explanation: Beta-lactam antimicrobials are commonly used in critical care units to treat pneumonia, bloodstream infections, wound infections, and other infections. Renal, hepatic, and other organ functions should be monitored in critically ill clients, and drug dosages should be reduced when indicated.

An adult client has reported experiencing a dry mouth and urinary retention after several nights of taking an over-the-counter (OTC) sleep aid. The nurse should suspect that this medication contains what antihistamine? A) Loratadine B) Promethazine C) Diphenhydramine D) Olopatadine

C) Diphenhydramine Explanation: The active ingredient in OTC sleep aids is a sedating antihistamine, usually diphenhydramine. This medication may create substantial anticholinergic effects (e.g., dry mouth, urinary retention, constipation, blurred vision). None of the other medications have such an anticholinergic effect.

A female client is prescribed a first-generation antihistamine for her allergies. The nurse would expect her to experience what adverse effect? A) Diarrhea B) Incontinence C) Dry mouth D) Slurred speech

C) Dry mouth Explanation: First-generation antihistamines have substantial anticholinergic effects; therefore, they may cause dry mouth, urinary retention, constipation, and blurred vision.

A male client is diagnosed with chronic bronchitis. What would the nurse expect to be one of his physical reports? A) Rhinitis B) Rhinorrhea C) Retention of secretions D) Chronic nasal swelling

C) Retention of secretions Explanation: Retention of secretions commonly occurs with influenza, pneumonia, upper respiratory infections, acute and chronic bronchitis, emphysema, and acute attacks of asthma.

A client is allergic to penicillin and has been diagnosed with a genitourinary infection caused by Chlamydia trachomatis. Which medication will most likely be administered? A) Acamprosate calcium B) Atazanavir C) Erythromycin D) Flumazenil

C) Erythromycin Explanation: A client who is diagnosed with a genitourinary infection that is caused by trachomatis and who is allergic to penicillin should be administered erythromycin since there is no known cross-reaction. Acamprosate calcium is administered as a substance abuse deterrent, not in place of penicillin. Atazanavir is an antiviral agent that is used to treat HIV infection, not Chlamydia trachomatis. Flumazenil is a benzodiazepine antagonist and not used for Chlamydia trachomatis.

The pharmacology instructor is discussing the adverse effects of diphenhydramine. Which is not one of these effects? A) Dry mouth B) Blurred vision C) Excessive salivation D) Drowsiness

C) Excessive salivation Explanation: Adverse effects of first-generation antihistamines include urinary retention, drowsiness or sedation, dry mouth, blurred vision, and gastrointestinal distress.

A school-age child has just returned home from the family's tropical vacation and is now reporting worsening tenderness in the right ear. When the child is diagnosed with otitis externa, what causative factor of this infection should the nurse focus upon? A) Immunocompromised state resulting from sleep deprivation while on vacation B) The potential for foodborne pathogens ingested while on vacation C) Frequent swimming and water sports while the child was on vacation D) Ear trauma related to pressurization and depressurization while flying

C) Frequent swimming and water sports while the child was on vacation Explanation: People whose ears are frequently exposed to moisture are more prone to the development of otitis externa. Swimming is a more likely cause of infection than pressure changes, foodborne pathogens, or impaired immunity.

The tetracyclines are effective against a wide range of which type of organisms? A) Gram-positive organisms B) Gram-negative organisms C) Gram-positive and gram-negative organisms D) Beta-lactamase-positive organisms

C) Gram-positive and gram-negative organisms Explanation: The tetracyclines are effective against a wide range of gram-positive and gram-negative organisms, although they are usually not drugs of choice

An 80-year-old client is diagnosed with latent tuberculosis infection. What is a risk for the elderly population when being treated with INH? A) Hypovolemia B) Hypoxemia C) Hepatotoxicity D) Renal failure

C) Hepatotoxicity Explanation: Although INH is the drug of choice for treatment of LTBI, its use is controversial in older adults. Because risks of drug-induced hepatotoxicity are higher in this population, some clinicians believe that those clients with positive skin tests should have additional risk factors (e.g., recent skin-test conversion, immunosuppression, previous gastrectomy) before receiving INH.

Sympathetic nervous system stimulation of the respiratory tract would result in: A) Diaphragmatic contraction B) Bronchoconstriction C) Increased respiratory rate D) Inspiratory movement

C) Increased respiratory rate Explanation: Sympathetic stimulation leads to an increased rate and depth of respiration and bronchodilation. Parasympathetic stimulation including the vagus nerve would lead to stimulation of diaphragmatic contraction, bronchoconstriction, and inspiratory movement.

Mark, 8 years old, is prescribed flunisolide. The physician advises Mark and his parents to use a spacer when administering this medication. What is the benefit of such use? A) It helps decrease the intrapulmonary delivery of the drug. B) It reduces the risk of tachycardia. C) It helps decrease systemic absorption. D) It reduces the risk of sinusitis.

C) It helps decrease systemic absorption. Explanation: Spacers help decrease systemic absorption, because less flunisolide is swallowed. Spacers may also help alleviate dysphonia by filtering larger aerosol particles that ordinarily deposit in the oropharynx and extrathoracic airways (this precaution also reduces the risk for oropharyngeal candidiasis). The use of a spacer does not prevent intrapulmonary delivery of the drug; nor does it reduce the risk of tachycardia and sinusitis.

A client is prescribed oxymetazoline, a decongestant, for the treatment of hay fever. What should the nurse evaluate to assess the efficacy of therapy? A) Maintenance of effective hydration of the skin B) Maintenance of an effective heart rate C) Maintenance of effective airway clearance D) Maintenance of an effective urine output

C) Maintenance of effective airway clearance Explanation: Evaluation of the maintenance of effective airway clearance helps in assessing the efficacy of oxymetazoline, which is a decongestant drug. The hydration of the skin, the heart rate and the urine output are usually not changed in decongestant therapy.

The health care provider has ordered promethazine (Phenergan) for a client who is having a severe allergic reaction. The nurse is aware that this medication is also used to treat what condition? A) Central nervous system depression B) Hypotension C) Nausea and vomiting D) Joint pain

C) Nausea and vomiting Explanation: Promethazine is most commonly used in the treatment of nausea and vomiting.

The nurse should monitor the client for which common side effects of erythromycin therapy? A) Headache and fever B) Urticaria and ophthalmic drainage C) Nausea, vomiting, and diarrhea D) Shortness of breath and sore throat

C) Nausea, vomiting, and diarrhea Explanation: Gastrointestinal problems (e.g., nausea, vomiting, and diarrhea) are common side effects of erythromycin and other macrolides. Headache, fever, ophthalmic drainage, urticaria, shortness of breath and sore throat are no common side effects.

Within what patient population does respiratory distress syndrome (RDS) tend to occur most frequently? A) Elderly B) Young adults C) Premature infants D) Adolescents

C) Premature infants Explanation: RDS causes obstruction at the alveolar level. It is frequently seen in premature infants who are born before their lungs have fully developed and while surfactant levels are still very low.

A female client presents to the emergency department with acutely deteriorating asthma. Her husband tells the nurse that his wife takes salmeterol. He then tells the nurse that he gave her three extra puffs when she became ill. What statement is correct in this situation? A) The husband made the correct decision in giving the extra doses. B) The extra doses facilitated bronchodilation and probably saved her life. C) Salmeterol is contraindicated based on his wife's condition. D) The health care provider will most likely order continuation of the salmeterol with increased dosage.

C) Salmeterol is contraindicated based on his wife's condition. Explanation: The FDA has issued a black box warning that initiating salmeterol in people with significantly worsening or acutely deteriorating asthma may be life threatening.

Bacitracin (Baciguent) interferes with the cell wall synthesis of which type of bacteria? A) Proteus B) Streptococcal C) Staphylococcal D) Pseudomonas

C) Staphylococcal Explanation: Bacitracin is an antibiotic that interferes with the cell wall synthesis of susceptible staphylococcal bacteria.

The nurse understands that an infection caused by a secondary pathogen, which can occur during prolonged antibiotic therapy, is known as which of the following? A) Acquired resistance B) Cross-sensitivity C) Superinfection D) Primary infection

C) Superinfection Explanation: A superinfection is an infection caused by a secondary pathogen that can occur during prolonged antibiotic therapy.

A young adult client's acne has responded well to treatment with tetracycline. However, the client has now returned to the clinic 6 weeks later with signs and symptoms of oral candidiasis. The nurse should recognize that this client's current health problem is likely attributable to which occurrence? A) A delayed (type IV) hypersensitivity reaction B) The fact that the client may have chewed the capsules prior to swallowing them C) Superinfection following the eradication of normal oral flora D) A type I hypersensitivity reaction

C) Superinfection following the eradication of normal oral flora Explanation: Candida superinfections may result from the use of tetracycline. The superinfection results from the elimination of normal flora and is not a hypersensitivity reaction. This is not the result of physical contact between the drug and the oral mucosa. Type IV hypersensitivity is often called delayed-type hypersensitivity as the reaction takes 2 to 3 days to develop. A type I hypersensitivity reaction is triggered by an innocuous foreign substance (like dust, pollen, or animal dander) that would cause no problems in the majority of people.

Which is used in the treatment of respiratory distress syndrome (RDS) to prevent atelectasis? A) Lasix B) Prednisone C) Surfactant D) Benadryl

C) Surfactant Explanation: Treatment of RDS aims at instilling surfactant to prevent atelectasis and to allow the lung to expand

A neonate develops respiratory distress syndrome. Which would be used? A) Anti-infective agents B) Bronchodilators C) Surfactant replacements D) Antihistamines

C) Surfactant replacements Explanation: With respiratory distress syndrome, there is a surfactant deficiency necessitating the use of surfactant replacement. Anti-infectives would be used to treat infections; bronchodilators would be used to relieve bronchospasm; and antihistamines may be used to address allergic reactions.

A 62-year-old client has been prescribed an antihistamine to alleviate vasomotor rhinitis. The client reports gastric irritation after taking the tablet. Which instructions should the nurse provide to help alleviate the client's condition? A) Drink ample water before taking the tablet. B) Take an antacid before the tablet. C) Take the tablet with food. D) Take the tablet one hour before food.

C) Take the tablet with food. Explanation: If the client has gastric irritation following ingestion of the antihistamine, the nurse should instruct the client to take the tablet with food and not before food. Taking an antacid before the tablet may reduce the absorption of the antihistamine. Drinking ample water before taking the tablet will not help to alleviate the patient's condition.

When describing an anti-infective agent with a narrow spectrum of activity, what would the nurse include? A) The drug is effective against many different organisms. B) The drug is highly aggressive in killing the pathogen. C) The drug is selective in its action on organisms. D) The drug is effective in interfering with the cell's reproduction.

C) The drug is selective in its action on organisms. Explanation: An anti-infective with a narrow spectrum of activity is selective in its action; thus, it is effective against only a few microorganisms with a very specific metabolic pathway or enzyme. Broad-spectrum activity refers to effectiveness against a wide variety of pathogens. Bactericidal refers to a highly aggressive drug that causes cell death. Bacteriostatic refers to a drug's effectiveness in interfering with a cell's ability to reproduce or divide.

Aminoglycoside antibiotics tend to collect in the eighth cranial nerve. The nurse would anticipate that which clinical manifestation may occur from the cranial nerve involvement? A) Inability to swallow B) Blindness C) Vertigo D) Slurred speech

C) Vertigo Explanation: The aminoglycosides antibiotics collect in the eighth cranial nerve and can cause dizziness, vertigo, and loss of hearing.

Which is a function of the bronchial circulation? A) Carries oxygenated blood in the veins B) Carries deoxygenated blood in the arteries C) Warms and humidifies incoming air D) Arises from the pulmonary artery

C) Warms and humidifies incoming air Explanation: The bronchial circulation functions to warm and humidify incoming air, while the pulmonary circulation functions to carry oxygenated blood in the veins and deoxygenated blood in the arteries.

In the rare instance in which penicillin is considered essential, hypersensitivity can be assessed by administering: A) the medication in a controlled environment. B) a loading dose of the medication. C) a skin test. D) the medication by the intravenous route only.

C) a skin test. Explanation: In the rare instance in which penicillin is considered essential, a skin test may be helpful in assessing hypersensitivity.

A nurse is caring for a 52-year-old client who has been diagnosed with a latent tuberculosis infection. The health care provider is considering ordering isoniazid (INH). The preexistence of what condition would require cautious use of INH in this client? A) hypertension B) folic acid deficiency C) cirrhosis of the liver D) glaucoma

C) cirrhosis of the liver Explanation: Because hepatotoxicity is a potentially serious adverse effect of INH, the drug should be used cautiously in clients with liver disease.

Cystic fibrosis is a hereditary disease that affects the respiratory system. In people with the disease, breathing performance is affected by: A) bronchospasm. B) infection. C) excessive secretions. D) chemical irritation of the respiratory tract.

C) excessive secretions. Explanation: Cystic fibrosis involves the exocrine glands of the respiratory, gastrointestinal, and reproductive tracts. It results in the accumulation of copious amounts of very thick secretions in the lungs. Eventually, the secretions obstruct the airways, leading to destruction of the lung tissue. Treatment involves keeping the secretions fluid and moving and maintaining airway patency.

Which condition would contraindicate the use of oxymetazoline? A) nasal congestion B) cough C) hypertension D) skin rash

C) hypertension Explanation: Oxymetazoline is contraindicated in clients with severe hypertension or coronary artery disease because of its cardiac stimulating and vasoconstricting effects. Oxymetazoline is prescribed for nasal congestion. Oxymetazoline is not contraindicated in cases that involve cough or skin rash.

A client has been placed on amphotericin B. The nurse caring for this client should administer this medication: A) intramuscularly. B) orally. C) intravenously. D) subcutaneously.

C) intravenously. Explanation: Amphotericin B is no longer administered orally; the most common route of administration is IV though intrathecal administration is used when fungus exists in the CSF.

The squad transported the victim of an MVI to a trauma center, intubated and unresponsive. The patient is connected to a vent and transferred to ICU. The patient's head and neck injuries could be affecting the respiratory rate and depth, suggesting possible injury to the patient's: A) cerebellum. B) frontal lobe. C) medulla oblongata. D) occipital lobe.

C) medulla oblongata. Explanation: The nervous system regulates the rate and depth of respiration by the respiratory center in the medulla oblongata, the pneumotaxic center in the pons, and the apneustic center in the reticular formation. The cerebellum deals with muscle coordination; the frontal lobe controls and directs the cognitive functions of the brain; the occipital lobe bears the visual areas.

A major component of fungal cell walls is β-(1,3)-D-glucan. Which antifungal drug combats infection by inhibiting synthesis of β-(1,3)-D-glucan? A) fluconazole B) amphotericin B C) micafungin D) nystatin

C) micafungin Explanation: Echinocandins inhibit synthesis of β-(1,3)-D-glucan. Examples of echinocandins include micafungin, anidulafungin, and caspofungin.

What intervention is most important for the nurse to perform before antibiotic therapy is initiated? A) increasing the client's fluid intake B) educating the client about adverse effects C) obtaining a specimen for culture and sensitivity D) administer an antidiarrheal to prevent gastrointestinal (GI) upset

C) obtaining a specimen for culture and sensitivity Explanation: It is good practice to collect specimens (e.g., sputum, urine) for culture and Gram's stain before giving the first dose of an antibiotic. Fluid intake and medication education may take place after the therapy is initiated, while antidiarrheal medication is administered if the client demonstrates signs/symptoms of GI distress.

What would a nurse least expect as an adverse reaction to anti-infective agents? A) kidney damage B) hypersensitivity C) respiratory toxicity D) neurotoxicity

C) respiratory toxicity Explanation: The least commonly encountered adverse effect associated with the use of anti-infective agents is respiratory toxicity. The most commonly encountered adverse effects are direct toxic effects on the kidney, gastrointestinal tract, and nervous system along with hypersensitivity and superinfections.

What occurs when the normal flora is destroyed by the use of anti-infectives? A) neurotoxicity B) hypersensitivity C) superinfection D) resistance

C) superinfection Explanation: Destruction of the normal flora by anti-infectives commonly leads to superinfection, an infection that occurs when opportunistic pathogens that were kept in check by the normal bacteria have the opportunity to invade the tissues. Neurotoxicity involves damage or interference with the function of nerve tissue, usually in areas where drugs tend to accumulate in high concentrations. Hypersensitivity or allergic reactions result from antibody formation. Resistance refers to the ability over time to adapt to an antibiotic and produce cells that are no longer affected by a particular drug.

An immunocompromised patient in a critical care setting has developed a respiratory infection that has been attributed to methicillin-resistant Staphylococcus aureus (MRSA). The nurse should anticipate that the patient will require treatment with A) ciprofloxacin. B) clindamycin. C) vancomycin. D) an antistaphylococcal penicillin.

C) vancomycin. Explanation: Vancomycin is the drug of choice to manage infections caused by MRSA. MRSA is resistant to all of the antistaphylococcal penicillins, as well as to ciprofloxacin and clindamycin.

The anatomy and physiology instructor is discussing bone development in the human body. According to the instructor, what hormone inhibits bone resorption? -Parathyroid -Calcitonin -Both of the above -Neither of the above

Calcitonin

A patient has hypocalcemia secondary to hypoparathyroidism. Which would the nurse expect to be ordered? -Levothyroxine -Calcitriol -Methimazole -Propylthiouracil

Calcitriol

The nurse should review which lab result before advising a client about taking the first dose of ibandronate (Boniva)? -Glucose -Calcium -Magnesium -Potassium

Calcium

Filariasis refers to infection of the blood and tissues of healthy people by worm embryos. Which of the following characteristics do not describe filariasis? Select all that apply.

Can cause an inflammatory reaction in cardiac muscle and in the brain Can cause fatal encephalitis

Liothyronine is a thyroid hormone used in replacement therapy for hypothyroidism. The drug must be used with extreme caution in patients with: -cardiac problems. -benign prostatic hyperplasia. -mental depression. -glaucoma.

Cardiac problems

Whipworms are a type of helminthic infection. Which are characteristics of whipworms? Select all that apply.

Cause colic and bloody diarrhea Transmitted when eggs found in the soil are ingested Attach to the wall of the colon

A patient is prescribed a cardiotonic medication. Which of the following pre-administration assessments should the nurse perform on this patient?

Check for jugular vein distention.

The nurse is preparing to administer a mixture of 12 units regular insulin and 45 units NPH insulin to a client with a blood sugar of 378 mg/dL. After the nurse draws the medication into the syringe, what is the nurse's next action? -Administer the insulin to the client. -Check the dosage with another nurse. -Check the client's blood sugar again. -Ensure a meal tray is available.

Check the dosage with another nurse

Blood lipids are a category of fatty acids, which are substances used within the body to perform essential functions. Which category of blood lipids is involved in the formation of atherosclerotic plaques?

Cholesterol Cholesterol is the portion of blood lipids involved in the formation of atherosclerotic plaques.

Which of the following would be classified as a bile acid sequestrant?

Cholestyramine Cholestyramine is classified as a bile acid sequestrant. Lovastatin is a HMG-CoA reductase inhibitor. Ezetimibe is a cholesterol absorption inhibitor. Gemfibrozil is classified as a fibrate.

A patient who has been taking lovastatin (Mevacor) has seen an improvement in his cholesterol laboratory values; however, the low-density lipoprotein remains elevated. What medication will be added to the medical regime?

Cholestyramine (Questran) Cholestyramine is administered to patients to reduce LDL cholesterol in patients who are already taking a statin drug. Digoxin is not given to lower LDL cholesterol. Vitamin D is not given to lower LDL cholesterol. Calcium carbonate is not given to lower LDL cholesterol.

Which agent would a nurse identify as the prototype histamine-2 receptor antagonist?

Cimetidine Rationale: Cimetidine is considered the prototype histamine-2 receptor antagonist.

A female client visits the health care provider's office after routine labs are drawn. The nurse notes that her A1C is 9. How does the nurse interpret this finding? -Client is in good glycemic control. -Client's average blood glucose is above normal. -Client's blood glucose demonstrates longstanding hypoglycemia. -Client's blood glucose levels are not consistent.

Client's average blood glucose is above normal

A nurse is preparing to administer an antiplatelet drug. Which medication would the nurse most likely administer?

Clopidogrel

A nurse is caring for an elderly patient who has been administered cimetidine. Which intervention should the nurse perform?

Closely monitor the patient for confusion and dizziness. Rationale: The nurse should closely monitor the elderly patient who has been administered cimetidine for confusion and dizziness. When the patient is receiving an antiemetic, the nurse monitors the patient frequently for continued reports of pain, sour taste, spitting blood, or coffee-ground-colored emesis. When antacids are given to the patient, the nurse should observe the patient for concentrated urine and restlessness. When the patient is administered prolonged doses of metoclopramide, the nurse reports any sign of tardive dyskinesia or extrapyramidal symptoms to the PHCP.

The nurse is conducting a pre-colonoscopy class and knows that polyethylene glycol- electrolyte solution will be contraindicated for the client with which condition?

Colitis

The nurse should warn a patient taking aluminum and calcium containing antacids about which of the following adverse effects? (Choose one)

Constipation Rationale: The nurse should warn a patient taking aluminum and calcium containing antacids about constipation that can be associated with taking products containing either medication.

A nurse is caring for a client receiving cholestyramine to improve his blood lipid profile at a home care setting. What adverse reactions to cholestyramine should the nurse monitor in the client?

Constipation The nurse should monitor for constipation in the client receiving cholestyramine. Rash, vertigo, and cholelithiasis should be monitored by the nurse when caring for a client receiving gemfibrozil.

A clinic nurse is planning care for a 68-year-old man who has been on omeprazole (Prilosec) therapy for heartburn for some time. Regarding the patient's safety, which of the following would be a priority nursing action?

Coordinate bone density testing for the patient. Rationale: All of the options would be appropriate for this patient. However, with regard to the patient's safety and the fact that he is a 68-year-old man and has been on omeprazole therapy for some time, the nurse should be concerned about possible hip fractures. Therefore, the priority action by the nurse should be to coordinate bone density testing. Omeprazole decreases the dissolution and absorption of calcium, which may increase the risk of hip fracture. Men are more likely to have PPI-associated hip fractures than women.

A patient is receiving fosinopril. Which adverse effect would the nurse caution the patient about to help to promote compliance?

Cough Rationale: Fosinopril is associated with an unrelenting cough that can lead patients to discontinue the drug. Constipation is an adverse effect of ACE inhibitors, but would not necessarily lead a patient to discontinue the drug. GI irritation is an adverse effect of ACE inhibitors, but would not necessarily lead a patient to discontinue the drug. Photosensitivity is an adverse effect of ACE inhibitors, but would not necessarily lead a patient to discontinue the drug.

Which statement by the client would lead the nurse to believe that he has understood the teaching provided regarding angina? a. "I will not exercise because it precipitates angina. b. "As long as I take the medicine, I can maintain my lifestyle. C. "There is no correlation between my hypertension and angina. d. 'Heavy meals and cigarette smoking can precipitate an angina attack.

D

The nurse is assessing a new client's lungs who was admitted with atelectasis. Which is the best response from the nurse when asked by the client what this means? A) "The outer layer of your lungs is swollen." B) "Your throat is swollen and not allowing enough air to enter your lungs." C) :Fluid is collecting in your lungs." D) "/Some of the air sacs in your lungs have collapsed."

D) "/Some of the air sacs in your lungs have collapsed." Explanation: Atelectasis is the collapse of once-expanded alveoli, tht can occur as a result of outside pressure against the alveoli. The other choices can be contributory factors to causing the alveoli to collapse. .

The nurse is teaching a client about the potential for allergic drug reaction. What statement made by a client suggests a need for further teaching? A) "All drugs have the potential to cause an allergic reaction." B) "The additives that are in drugs can also cause allergic reactions." C) "Virtually any drug may induce an immunologic response in a susceptible person." D) "If you didn't have allergies as a child, you won't develop allergies later in life."

D) "If you didn't have allergies as a child, you won't develop allergies later in life." Explanation: Drug allergies can develop later in life, though many appear at a young age. All the other options contain accurate statements

The client has been taking a fluoroquinolone and now reports that he has a white patch in his mouth. What is the best response of the nurse? A) "You are allergic to the medication. Stop taking it immediately." B) "That is the medication working. Continue to take as directed." C) "You are experiencing an adverse affect of the medication. Those symptoms will decrease over time." D) "You may be experiencing an additional infection. I will discuss this with your health care provider."

D) "You may be experiencing an additional infection. I will discuss this with your health care provider." Explanation: A burning sensation of the mouth or throat may be an indication of a superinfection. It is important the nurse notify the provider. The nurse should not minimize the client's concerns or cause panic.

A client is receiving flucytosine. The nurse is reviewing the client's serum drug level. Which serum drug level would lead the nurse to suspect that the client is developing toxicity? A) 22 mcg/mL B) 45 mcg/mL C) 88 mcg/mL D) 110 mcg/mL

D) 110 mcg/mL Explanation: Serum flucytosine levels greater than 100 mcg/mL are associated with toxicity.

A client asks the nurse about using dextromethorphan to relieve a cough. What type of cough would the nurse explain is best treated with the drug? A) An occasional, productive cough B) A cough that is associated with an allergy to ragweed C) A cough that occurs when the client is exposed to airborne irritants D) A dry, nonproductive cough

D) A dry, nonproductive cough Explanation: The major clinical indication for use of dextromethorphan is a dry, hacking, nonproductive cough that interferes with rest and sleep. It is not desirable to suppress a productive cough because the secretions need to be removed. The character, not cause, of the cough is relevant to its treatment with dextromethorphan.

A client has developed atelectasis. What is the best explanation of the cause of atelectasis? A) Inflammation of the pleura B) Inflammation of the trachea C) Air or fluid accumulation blocked in the trachea D) Air or fluid accumulation in the pleural space

D) Air or fluid accumulation in the pleural space Explanation: Atelectasis, the collapse of once-expanded alveoli, can occur as a result of outside pressure against the alveoli—for example, from a pulmonary tumor, pneumothorax (air in the pleural space exerting high pressure against the alveoli), or pleural effusion.

A client, scheduled to receive one unit of packed red blood cells, has a history of an allergic reaction to a transfusion in the past. What class of medication will assist in preventing a reaction to the packed red blood cell transfusion? A) Antipyretics B) Antimicrobials C) Antianginals D) Antihistamines

D) Antihistamines Explanation: Premedication with an antihistamine may be used to prevent allergic reactions with the administration of a transfusion of packed red blood cells. The administration of antimicrobial or antianginal is not recommended for a transfusion reaction. An antipyretic is used to treat the symptom of fever.

A client exhibiting signs and symptoms of the common cold asks the nurse if taking an antihistamine would be helpful. What is the nurse's best response? A) Yes, but be sure to use a brand that mentions 'cold remedy' on the packaging. B) Yes, but it's important to monitor for adverse effects. C) Antihistamines can be helpful when they're combined with an antibiotic. D) Antihistamines are not recommended for treating a cold.

D) Antihistamines are not recommended for treating a cold. Explanation: Although antihistamines are popular over-the-counter drugs, studies have demonstrated that they are not successful or recommended for the treatment of the common cold. An antibiotic should not be prescribed for the treatment of the common cold, which is a viral disorder.

A client is experiencing an acute asthmatic attack. Which agent would be most effective? A) Inhaled steroid B) Leukotriene receptor antagonist C) Mast cell stabilizer D) Beta-2 selective adrenergic agonist

D) Beta-2 selective adrenergic agonist Explanation: A beta-2 selective adrenergic agonist or sympathomimetic would be most appropriate because these agents are rapidly distributed after injection and rapidly absorbed after inhalation. An inhaled steroid would require 2 to 3 weeks to reach effective levels. Leukotriene receptor antagonists and mast cell stabilizers do not have immediate effects

An instructor is describing the respiratory membrane at the alveolar level. Which would the instructor include as a component? A) Cilia B) Goblet cells C) Mast cells D) Capillary endothelium

D) Capillary endothelium Explanation: The respiratory membrane is made up of the capillary endothelium, capillary basement membrane, interstitial space, alveolar basement membrane, alveolar epithelium, and surfactant layer. Cilia, goblet cells, and mast cells are found along the upper respiratory tract.

Which type of testing should be completed prior to an anti-infective being prescribed? A) H & H B) CBC C) EKG D) Culture

D) Culture Explanation: Culture and sensitivity testing should be performed before an anti-infective agent is prescribed.

A nurse is required to educate a patient prescribed albuterol on the adverse effects associated with the drug. Which of the following symptoms, if experienced, should the nurse instruct the patient to report to the health care provider? A) Fall in blood pressure B) Increased nighttime urination C) Hearing impairment or deficit D) Headache and flushing

D) Headache and flushing Explanation: The nurse should instruct the patient to contact the health care provider if palpitations, tachycardia, chest pain, muscle tremors, dizziness, headache, flushing, or difficulty with urination or breathing occur. Fall in blood pressure, increased nighttime urination, or hearing impairment are not adverse effects associated with a sympathomimetic bronchodilator.

What is the chemical mediator released in immune and inflammatory response to allergic reactions? A) Norepinephrine B) Mast cells C) Epinephrine D) Histamine

D) Histamine Explanation: Histamine is the chemical mediator released in immune and inflammatory responses. Histamine is discharged from mast cells and basophils in response to certain stimuli (e.g., allergic reactions, cellular injury, extreme cold). This histamine is discharged by mast cells. Epinephrine is the drug of choice for treating severe anaphylaxis. Norepinephrine is a chemical released from the sympathetic nervous system in response to stress.

A client with a diagnosis of active TB has begun second-line therapy that includes the use of pyrazinamide. When monitoring this client, the nurse should suspect that adverse effects of this drug may account for which laboratory value? A) Low hematocrit and mean corpuscular volume (MCV) B) Increased INR and aPTT C) Increased blood urea nitrogen and creatinine D) Increased AST, ALT, and GGT

D) Increased AST, ALT, and GGT Explanation: The most severe adverse effect of pyrazinamide is hepatotoxicity. Consequently, liver enzymes such as GGT, ALT, and AST should be monitored. Changes in erythrocytes, coagulation, and renal function are less likely to result from pyrazinamide therapy.

The triage nurse in the emergency department has a 42-year-old client with asthma present for treatment. The client's respiratory rate is 40 breaths per minute. Based on this objective data, what is the correct nursing diagnosis for this client? A) Activity intolerance B) Impaired gas exchange C) Ineffective airway clearance D) Ineffective breathing pattern

D) Ineffective breathing pattern Explanation: The correct nursing diagnosis is Ineffective Breathing Pattern related to impaired airway as manifested by tachypnea. The other options may be applicable, but they do not take priority over the ineffective breathing pattern.

A client is brought to the emergency department after taking an overdose of levothyroxine. When assessing this client, what adverse effects would the nurse expect to find? -Skin rash and itching -Nervousness and tachycardia -Slow speech and mental dullness -Drowsiness and bradycardia

Nervousness and tachycardia

The nurse is providing education to a client with asthma on the therapeutic action of inhaled corticosteroid agents. How will the nurse describe the action? A) Inhaled corticosteroid agents will depress the central nervous system. B) Inhaled corticosteroid agents reduce respiratory rate. C) Inhaled corticosteroid agents reduce bronchodilation. D) Inhaled corticosteroid agents reduce airway inflammation.

D) Inhaled corticosteroid agents reduce airway inflammation. Explanation: Inhaled corticosteroid agents suppress the release of inflammatory mediators, block the generations of cytokines, and decrease the recruitment of airway eosinophils. Inhaled corticosteroid agents do not depress the central nervous system or reduce bronchodilation or respiratory rate

Which antifungal agent is available for systemic and topical use? A) Butoconazole B) Clotrimazole C) Voriconazole D) Ketoconazole

D) Ketoconazole Explanation: Ketoconazole can be used systemically and topically to treat fungal infections. Butoconazole and clotrimazole are for topical use only. Voriconazole is for systemic use only.

The nurse is treating a client for a systemic fungal infection with an oral fungicide. What would be an important nursing action for the nurse to perform? A) Put up foot rails. B) Provide for use of restraints. C) Offer food in three meals daily if GI upset occurs. D) Order dietary consultation as needed to ensure nutritional status.

D) Order dietary consultation as needed to ensure nutritional status. Explanation: Monitor nutritional status and arrange a dietary consultation as needed to ensure nutritional status. Putting foot rails up is not important, restraints are inappropriate, and six small meals should be offered daily if GI upset occurs.

A group of nursing students are learning about the factors that underlie recent increases in the incidence and prevalence of antibiotic-resistant microorganisms. What factor is known to contribute to antibiotic resistance? A) Increased survival rates from acute infections B) Increased population density C) Use of antibiotics that are ineffective against the infectious microorganism D) Overuse of antibiotics

D) Overuse of antibiotics Explanation: Antibiotic overuse can contribute to antibiotic resistance. Resistance is not typically attributable to increased age of clients, increased population density, or the use of ineffective antibiotics.

A nurse is instructing a colleague on how an antimicrobial produces a therapeutic effect. What should be included in the nurse's teaching? A) Fluoroquinolones interfere with the growth and development of the bacteria cell wall. B) The first drugs used to treat infections date back to the 17th century. C) Selective toxicity determines the appropriate drug dosage needed. D) Penicillin interferes with synthesis of the bacteria cell wall.

D) Penicillin interferes with synthesis of the bacteria cell wall. Explanation: The action of antimicrobials is to interfere with the normal function of the invading organism to prevent it from reproducing and to cause cell death without affecting host cells. Penicillin interferes with biosynthesis of the bacterial cell wall. Because bacteria cells have a slightly different composition than human cells, the bacteria are destroyed without interfering with the host. The first drugs used to treat systemic infections were developed in the early 20th century. Fluoroquinolones act by interfering with DNA synthesis. Sulfonamides interfere with growth and development of bacterial cells. The term selective toxicity refers to the ability to affect certain proteins or enzyme systems that are used by infecting organisms but not by human cells.

After teaching a group of students about resistance, the instructor determines that the students need additional teaching when they identify what as a way that microorganisms develop resistance? A) Production of an enzyme that deactivates the drug B) Change in cellular permeability preventing drug entrance C) Altered binding sites that no longer accept the drug D) Production of a chemical to act as an agonist

D) Production of a chemical to act as an agonist Explanation: Microorganisms develop resistance by producing a chemical that acts as an antagonist not agonstic to the drug. In addition, the microorganism can produce an enzyme that deactivates the drug, change cellular permeability so that the drug cannot enter the cell, and alter binding sites to no longer accept the drug.

Due to complications, a male postoperative patient has been unable to mobilize for several days following surgery and has developed atelectasis. Which process would his care team anticipate with relation to his health problem? A) Vasodilation in the alveolar vessels in the affected region of his lung B) Increased workload for the left side of the patient's heart C) Increased blood flow to the area of atelectasis D) Redirection of blood flow into the lung regions that are hypoxic

D) Redirection of blood flow into the lung regions that are hypoxic Explanation: Regional hypoxia, such as with a diagnosis of atelectasis, is associated with vasoconstriction and redirection of blood away from, not toward, the affected area of the lung. This also contributes to an increased workload for the right side of the heart.

A middle-aged patient has been prescribed tetracycline as part of his Prevpac for the treatment of H. pylori. The patient has a history of atrial fibrillation which is being treated with digitalis drugs. Given his history and current medications, the patient is at risk for which of the following conditions? A) Respiratory depression B) Increase in serum levels C) Increase in serum theophylline level D) Risk of toxicity

D) Risk of toxicity Explanation: When digitalis drugs interact with tetracyclines, the patient is at a risk for toxicity. Respiratory depression is an effect observed when neuromuscular-blocking drugs interact with lincosamides. Increase in serum levels is observed when digoxin interacts with macrolides. Increase in serum theophylline level occurs when theophylline interacts with macrolides.

A medical nurse on a night shift is reviewing a client's medication administration record for the following day. The nurse notes that a combination antimicrobial drug is prescribed. What is implied by the fact that the client has been prescribed a combination drug? A) The client's infection likely has a fungal or protozoal etiology. B) The client likely has a history of recurrent, multisystemic infections. C) The client may be unable to tolerate treatment with a single antimicrobial. D) The client may have an infection caused by multiple microorganisms.

D) The client may have an infection caused by multiple microorganisms. Explanation: Indications for combination therapy may include infections caused by multiple microorganisms. A fungal or protozoal infection is not implied, and the client may or may not have a history of recurrent infections. Intolerance of single antibiotics is not normally an indication for combination therapy.

Chest radiography and sputum sample microscopy have confirmed a diagnosis of TB in a 40-year-old man who has a history of type 1 diabetes and who currently has a diabetic foot ulcer. How should the nurse account for this patient's health status when planning care during INH treatment? A) The patient will require intramuscular, rather than oral, administration of INH. B) The patient will require oral antihyperglycemics because exogenous insulin will be contraindicated. C) The patient will require positive-pressure isolation because he will be immunocompromised. D) The patient will require vigilant monitoring of blood glucose levels.

D) The patient will require vigilant monitoring of blood glucose levels. Explanation: INH treatment constitutes a risk for hyperglycemia and close monitoring of blood glucose levels is thus indicated. This patient's history will not likely impact the chosen route for the drug and insulin therapy can be safely continued. The patient will be vulnerable to other infections for the foreseeable future, but he will not likely be immunocompromised to the degree that positive-pressure isolation is required.

A nurse has questioned why a client's health care provider has prescribed a narrow-spectrum antibiotic rather than a broad-spectrum drug in the treatment of an infection. Which facts provide the best rationale for the use of narrow-spectrum antibiotics whenever possible? A) Broad-spectrum antibiotics confound the results of subsequent culture and sensitivity testing. B) Narrow-spectrum antibiotics normally require a shorter duration of treatment. C) The efficacy of most narrow-spectrum antibiotics has not been proven. D) The use of broad-spectrum antibiotics can create a risk for a superinfection.

D) The use of broad-spectrum antibiotics can create a risk for a superinfection. Explanation: The benefit of a narrow-spectrum antimicrobial agent is that it limits the potential for adverse effects, such as superinfection. In a superinfection, an antibiotic suppresses all susceptible microbes, including the body's natural flora, which may keep other microbes in check. In the absence of these bacteria, nonsusceptible microbes can proliferate. Narrow-spectrum antibiotics do not lack demonstrated efficacy and they do not necessarily require a shorter duration of treatment. All antimicrobial drugs have the potential to impact subsequent C&S testing.

A male client presents to the health care provider's office for his annual visit. When questioned about over-the-counter medication use, he states that he uses echinacea to prevent colds. What statement is true about echinacea? A) He is healthier because he uses the echinacea. B) Echinacea is the OTC drug of choice for prevention of viral infections. C) To be effective, echinacea must be taken daily regardless of symptoms. D) There is limited or no support for the use of echinacea to prevent or treat symptoms of the common cold.

D) There is limited or no support for the use of echinacea to prevent or treat symptoms of the common cold. Explanation: There is limited or no support for the use of dietary or herbal supplements to prevent or treat symptoms of the common cold.

Like many other classes of drugs, anti-infective drugs can have a variety of adverse effects. What is the most common, potentially serious, adverse effect of antimicrobial drugs? A) Skin rash B) Pain C) Constipation D) Toxic effects on the kidney

D) Toxic effects on the kidney Explanation: The most commonly encountered adverse effects associated with the use of anti-infective agents are direct toxic effects on the kidney, gastrointestinal (GI) tract, and nervous system. Pain, constipation, and hypopnea are not common adverse effects of antimicrobial drugs.

A group of students are reviewing information about anti-infective agents. The students demonstrate a need for additional review when they identify what as an anti-infective agent? A) antibiotic B) anthelmintic C) antiprotozoal D) anticoagulant

D) anticoagulant Explanation: An anticoagulant interferes with blood clotting and is not an anti-infective agent. Antibiotics, anthelmintics, antiprotozoals, antivirals, and antifungals are all anti-infective agents.

A nurse is providing education to a 56-year-old man who is admitted to the emergency department with an acute asthma attack. The nurse's initial assessment reveals that the patient has a history of pneumonia, drinks large quantities of coffee, and eats a high-calorie diet. Albuterol is prescribed for him. The important consideration when the nurse is preparing a teaching plan will be that the patient A) is a 56-year-old man. B) has a history of pneumonia. C) prefers a high-calorie diet. D) has a high coffee intake.

D) has a high coffee intake. Explanation: Caffeine has sympathomimetic effects that may increase the risk for adverse effects with albuterol. The nurse should assess the patient's intake of caffeine through coffee, tea, soda, cocoa, candy, and chocolate. The patient's age, history of pneumonia, and preference for high-calorie food would not have important implications for his albuterol drug therapy.

A nurse is providing health teaching to a newly diagnosed asthmatic client. The client has been prescribed theophylline. The nurse should encourage this client to stop: A) using insulin. B) taking anti-inflammatory drugs. C) exercising. D) smoking cigarettes.

D) smoking cigarettes. Explanation: Nicotine increases the metabolism of xanthines in the liver and the xanthine dosage must be increased in clients who continue to smoke while using xanthines. In addition, extreme caution must be used if the client decides to decrease or discontinue smoking because severe xanthine toxicity can occur. Bronchoconstriction is not caused by using insulin and anti-inflammatory drugs do not cause bronchoconstriction.

A nurse is teaching a newly diagnosed client with asthma how to manage the disease. What should the nurse encourage this client to do? A) begin using insulin B) taking anti-inflammatory drugs C) decrease exercising D) stop smoking cigarettes

D) stop smoking cigarettes xplanation: Since bronchoconstriction can be triggered by smoke, clients with asthma should be encouraged to quit smoking and to avoid secondhand smoke. Instead of instructing a client to abandon exercise, healthcare providers should consider prescribing bronchodilators as pre-exercise prophylaxis. nonsteroidal anti-inflammatory drugs can cause trigger an asthmatic attack in some clients; however, not all anti-inflammatory drugs cause this reaction.

Tuberculosis typically affects the lungs but can also involve other parts of the body. Which of the following can be affected by the disease? A) Kidneys B) Lymph nodes C) GI/GU tract D)All the above

D)All the above Explanation: Tuberculosis is an infectious disease that usually affects the lungs but may involve most parts of the body, including lymph nodes, meninges, bones, joints, kidneys, and the gastrointestinal tract.

The mother of a 4-year-old child tells the nurse that her child has been sleeping very poorly. The child has also been scratching in her rectal or perineal area. The nurse suspects that this child has what parasitic infection?

Pinworms

A nurse is caring for a client experiencing a hypertensive emergency. If blood pressure is not lowered immediately which of the following can occur? Select all that apply:

Damage to the eyes Damage to the kidneys Damage to the heart Rationale: A hypertensive emergency if not recognized and treated quickly can result in damage to target organs including the heart, kidneys, and eyes.

Ms. Catalin has been started on labetalol. Which of the following effects can occur as a result of the beta blocking?

Decrease in the plasma renin level Rationale: The beta-blocking effect of labetalol results in a decrease in the plasma renin level. The alpha-blocking action causes orthostatic hypotension. The beta-blocking action prevents reflex tachycardia and elevations in blood pressure.

A nurse is caring for a patient admitted to the intensive care unit because of heart failure. The patient is prescribed digoxin. Which nursing diagnosis would be appropriate for this patient?

Decreased Cardiac Output related to altered cardiac function

A nurse is caring for a patient admitted to the intensive care unit because of heart failure. The patient is prescribed digoxin. Which of the following nursing diagnoses would be appropriate for this patient?

Decreased Cardiac Output related to altered cardiac function

A nurse is caring for a patient with hypothyroidism. The nurse would know that the effects of hypothyroidism include: -Fever -Decreased cardiac output -Increased blood pressure -Nervousness and restlessness

Decreased cardiac output

When circulatory shock occurs, there is massive vasodilation causing pooling of the blood in the periphery of the body. As an ICU nurse caring for a patient in circulatory shock, you know that the pooling of blood in the periphery leads to what?

Decreased venous return Rationale: Pooling of blood in the periphery results in decreased venous return. Decreased venous return results in decreased stroke volume and decreased cardiac output. Decreased cardiac output, in turn, causes decreased blood pressure and, ultimately, decreased tissue perfusion. Option C is incorrect; heart rate increases in an attempt to meet the demands of the body.

Your 71-year-old male patient has recently been diagnosed with hypertension. Which of the following measurements is a partial indication of effective treatment and management?

Diastolic blood pressure below 90 mm Hg Rationale: Successful treatment involves reducing blood pressure below hypertensive levels. In adults, this is typically defined as a systolic pressure below 140 mm Hg and a diastolic pressure below 90 mm Hg.

Key behavioral determinants of blood pressure are related to what factor?

Dietary consumption of calories and salt Rationale: Key behavioral determinants of blood pressure are related to dietary consumption of calories and salt; the prevalence of hypertension rises proportionally to average body mass index.

What conclusion should the nurse draw when a client's digoxin level is reported to be 2.2 ng/mL?

Digitalis toxicity is a possibility.

Your client's digoxin level is reported to be 2.2 ng/mL. You know that this result indicates:

Digitalis toxicity.

Which are the most frequent cause of helminth infections in the United States?

Pinworms

A 94-year-old client is to begin taking psyllium hydrophilic mucilloid daily. What instructions should the nurse include in the discharge teaching?

Drink at least 8 ounces of fluid with the medication

A 4-year-old is tested and found to have deficient growth hormone (GH). What does this condition cause? a.Gigantism b.Acromegaly c.Syndrome of inappropriate antidiuretic hormone (SIADH) d.Dwarfism

Dwarfism Feedback: GH deficiency in children results in short stature (dwarfism). Excess production of GH results in gigantism and acromegaly. SIADH is caused by excessive production of antidiuretic hormone.

As part of a routine physical examination, a 60-year-old patient's primary care provider has ordered blood work that includes cholesterol levels. What result would most strongly suggest the need for treatment with an antihyperlipidemic drug?

Elevated LDL levels Elevated LDL levels are considered a risk factor for coronary artery disease. Elevated HDL levels, low VLDL levels, and a high ratio of HDL to LDL are associated with a decreased risk of heart disease.

A client is admitted with thrombophlebitis and sent home on enoxaparin therapy. Which statement indicates a good understanding of why enoxaparin is being administered?

Enoxaparin inhibits the formation of additional clots.

The nurse is conducting a follow up assessment of a client who had a pinworm infection and who completed a course of mebendazole one week ago. The client states, "I followed the directions for taking the pills exactly, but the itching seems as bad as ever." What is the nurse's best action?

Ensure that the client follows up with the care provider promptly

Insulin binds with and activates receptors on cell membranes. Once insulin-receptor binding occurs, the membranes become highly permeable to glucose. Which action does this enable? -Release of glucagon from the cells -Storage of glucagon in the cells -Entry of glucose into the cells -Interruption of glucose movement across the membrane

Entry of glucose into the cells

True or False: A positive Chvostek's sign suggests hypercalcemia.

False -Hypocalcemia is indicated by a positive Chvostek's sign.

True or False: Clients with type 2 diabetes have nonfunctioning beta pancreatic cells.

False -Type 2 diabetes reflects an inability to produce enough insulin as needed or a change in insulin receptor sensitivity

Saliva substitute is absorbed systemically.

False Rationale: Saliva substitute is not generally absorbed systemically.

If a patient is receiving sucralfate and omeprazole, the doses can be taken together.

False Rationale: Sucralfate is not absorbed well in the presence of proton pump inhibitors and doses should be spaced at least 30 minutes apart if both drugs are prescribed.

After reviewing information about lipoproteins, a group of students demonstrate understanding of the information when they identify which of the following as being loosely packed?

HDLs HDLs are loosely packed lipids that are used for energy and to pick up remnants of fats and cholesterol left in the periphery by the breakdown of LDLs. LDLs are tightly packed cholesterol, triglycerides, and lipids that are carried by proteins that enter the circulation to be broken down for energy or stored for future use as energy.

A client who has had no previous contact with the healthcare system presents with elephantiasis. of the lower legs. What helminthic infection should the nurse suspect?

Filariasis

Which would a nurse expect to assess in a client experiencing hyperthyroidism? -Intolerance to cold -Flushed, warm skin -Slow and deep tendon reflexes -Bradycardia

Flushed, warm skin

A nurse is caring for a patient prescribed nicotinic acid for hyperlipidemia. Which of the following are experienced by patients taking nicotinic acid?

Flushing of the skin The nurse should inform the patient prescribed nicotinic acid that flushing of the skin is generally experienced by patients during this treatment. The nurse should not state weakness, tachycardia, or dyspnea are adverse reactions experienced by patients taking nicotinic acid. These are toxic reactions to flax.

The health care provider orders an antiparasitic drug for the client. The drug is prescribed for treatment of local effects in the skin and which other area?

GI tract

After teaching a group of students about fats and biotransformation, the instructor determines that the teaching was successful when the students identify which of the following as the storage location of bile acids?

Gallbladder The presence of fatty acids, lipids, and cholesterol in the duodenum stimulates contraction of the gallbladder and the release of bile, which contains bile acids. Once their action is completed, they are reabsorbed and recycled to the gallbladder, where they remain until the gallbladder is stimulated again.

Helminthic infections are in which part of the body?

Gastric tract

The three major classes of drugs used to control blood lipids are statins, bile acid resins (or sequestrants), and fibrates. Which of the following is a fibrate?

Gemfibrozil Gemfibrozil is a fibrate. Colestipol, colesevelam, and cholestyramine are bile acid resins.

After reviewing the various agents used to lower lipid levels, the students demonstrate understanding of the information when they identify which agents as stimulating the breakdown of lipoproteins from the tissues and their removal from the plasma? Select all that apply.

Gemfibrozil Fenofibrate Fibrates stimulate the breakdown of lipoproteins from the tissues and their removal from the plasma. Examples include fenofibrate and gemfibrozil.

Which of the following medications is classified as a fibric acid derivative (fibrate)? (Choose one)

Gemfibrozil (Lopid) Lopid is a fibric acid derivative. Mevacor is a HMG-CoA reductase inhibitor. WelChol is a bile acid resin. Niaspan is a miscellaneous antihyperlipidemic. Zetia is a miscellaneous antihyperlipidemic.

The nurse is teaching a female patient about newly prescribed digoxin. The patient tells the nurse that she occasionally uses herbal therapies. The nurse should caution the patient against using therapies that involve which herbs?

Ginseng

You are teaching a patient about her newly prescribed digoxin. The patient tells you that she occasionally uses herbal therapies. You should caution the patient against using therapies that involve which of the following herbs?

Ginseng

A nurse is caring for a patient who has developed a hypoglycemic reaction. Which intervention should the nurse perform if swallowing and gag reflexes are present in the patient? -Administer glucagon by the parenteral route. -Administer the insulin via insulin pump. -Give oral fluids or candy. -Administer oral antidiabetics to the patient.

Give oral fluids or candy.

Mebendazole (Vermox) interferes with a worm's ability to use what, which leads to the inability to reproduce and cell death?

Glucose

The nurse is educating new parents on treatment of infants with constipation. What is the most effective and safest way to treat constipation in infants prior to bowel training?

Glycerin suppositories

The nurse instructs a client to take the prescribed pravastatin at bedtime based on the understanding about which of the following?

Greater drug effectiveness is achieved at this time. The drug is administered at bedtime because the highest rates of cholesterol synthesis occur between 12 and 5 AM, and the drug should be taken when it will be most effective.

A male patient is taking cimetidine (Tagamet). Which adverse effect is more likely to occur with cimetidine than with other histamine2 receptor antagonists?

Gynecomastia Rationale: Gynecomastia is an adverse effect that is more likely to occur with cimetidine than with other histamine2 antagonists. Hypoxia, hypertension, and seizures are not adverse effects of cimetidine. (less)

The nurse is evaluating the effectiveness of drug therapy in a client with hyperlipidemia. Effective therapy is best demonstrated by which laboratory values?

HDL 58 LDL 96 Cholesterol 178 Normal HDL should be less than 40 mg/dl, LDL should be less than 100 mg/dl, and total cholesterol should be less than 200 mg/dl.

Which drug will be most effective in reducing the major types of dyslipidemia?

HMG-CoA reductase inhibitors For single-drug therapy, a statin is preferred. Statins are used for types IIa, IIb, IV, and V. Bile acid sequestrants primarily lower LDL. Fibrates decrease VLDL and increase HDL. Niacin decreases LDL cholesterol.

The pharmacology instructor is discussing medications used in the treatment of dyslipidemia. Which of the following drug classes would the instructor identify as the most widely used dyslipidemia drugs?

HMG-CoA reductase inhibitors HMG-CoA reductase inhibitors (or statins) are the most widely used dyslipidemia drugs. They are useful in treating most of the major types of dyslipidemia.

The nurse is educating a client with hypertension about dietary changes that help to decrease blood pressure. Which of the following menu selections indicates the need for further client education?

Ham sandwich with mustard, carrots and cheesy dip Rationale: A ham sandwich with mustard and a cheesy dip has increased fat and sodium. The DASH diet includes whole grains, poultry, fish, and nuts and has reduced amounts of fats, red meats, sweets, and sugared beverages.

Before administering a prescribed 20 units of NPH insulin to a client, the nurse should implement which intervention? -Have a colleague confirm the dosage. -Assess the client's urine for the presence of glucose. -Assess the client's understanding of diabetes. -Massage the chosen injection site.

Have a colleague confirm the dosage.

A client is prescribed calcitriol. Which instruction would be most important for the nurse to include in the teaching plan? -"Have your calcium levels checked periodically." -"Limit your intake of dairy products." -"Take the drug with food if gastrointestinal issues occur." -"Take the drug with a magnesium antacid."

Have your calcium levels checked periodically

The physician has prescribed ranitidine for a patient in your unit. Prior to administering the drug for the first time, which of the following adverse effects should you mention to the patient?

Headache Rationale: Adverse effects with H2RAs such as ranitidine are typically uncommon and mild with the usual doses and duration of treatment. However, effects can include diarrhea or constipation, headache, dizziness, muscle aches, and fatigue.

What assessment should the nurse perform prior to administration of a scheduled dose of levothyroxine? -oxygen saturation level -level of consciousness and orientation -respiratory rate -heart rate

Heart Rate

What is the benefit of biotransformation?

Helps to prevent medications from causing adverse effects on the body (First pass effect)

A client with a history of clot formation is scheduled for bowel resection due to colorectal cancer. What anticoagulant agent will be administered prophylactically?

Heparin

A postsurgical client possesses numerous risk factors for venous thromboembolism, including a previous deep vein thrombosis. What drug would the nurse anticipate administering while this client recovers in the hospital?

Heparin

The nurse is preparing to administer protamine emergently to a client per instructions from the health care provider. The nurse concludes this is necessary due to an adverse reaction to which drug?

Heparin

A female client is started on warfarin therapy. The client asks the nurse why she is no longer on heparin therapy. What differentiates heparin from warfarin?

Heparin is administered parenterally, while warfarin is administered orally.

A patient is taking cimetidine (Tagamet) for increased gastric pain and hypersecretion. Nurses should use caution when administering cimetidine to patients with which of the following conditions?

Hepatic disease Rationale: No contraindications exist, but the use of cimetidine should be used cautiously in patients with impaired hepatic and renal function. The patient with cancer, hypertension, and diabetes mellitus can take cimetidine safely.

What is the best position for the client after the administration of a bisphosphonate medication? -Prone -High Fowler's -Lateral recumbent -Supine

High Fowler's

When administering anthelmintic drugs, which meals should be avoided to prevent interference with drug effectiveness?

High fat

What is the most common reason for an elevated cholesterol level in a client who does not have a genetic disorder of lipid metabolism?

His dietary intake of saturated fat Unless a person has a genetic disorder of lipid metabolism, the amount of cholesterol in the blood is strongly related to dietary intake of saturated fat.

A nurse is caring for a client who has been diagnosed with hypothyroidism. Levothyroxine (Synthroid) has been prescribed. Before the drug therapy is started, the nurse will assess for: -allergy to seafood. -hirsutism. -history of taking anticoagulant drugs. -the client's age.

History of taking anticoagulant drugs

The health care provider has ordered a change of prescription from rapid-acting insulin to an intermediate-acting type. Which adverse effect must the nurse closely monitor for in the client? -Bradycardia -Lipodystrophy -Hypoglycemia -Hypotension

Hypoglycemia

A patient is receiving a diuretic for the treatment of hypertension. For which of the following conditions should the nurse monitor for in patients taking diuretics?

Hyponatremia Rationale: The nurse should assess for hyponatremia in patients receiving diuretics. Diuretic usage causes electrolyte disturbances such as hyponatremia and hypokalemia. The nurse should inform the primary healthcare provider if signs and symptoms of electrolyte imbalance occur. Hyperkalemia, hypomagnesemia, and hypocalcemia do not occur in patients taking diuretics.

You are caring for a patient at risk of shock. What physiologic response would you know to look for while assessing for shock?

Hypoperfusion of tissues Rationale: Regardless of the initial cause of shock, certain physiologic responses are common to all types of shock. These physiologic responses include hypoperfusion of tissues, hypermetabolism, and activation of the inflammatory response.

A client is taking an antacid that contains aluminum salts. The nurse would monitor the client for which of the following?

Hypophosphatemia Rationale: Aluminum binds dietary phosphates and causes hypophosphatemia, but they do not cause acid rebound like other antacids. Magnesium antacids cause diarrhea; calcium salts cause hypercalcemia.

A patient, aged 78 years, is receiving nitroprusside for hypertensive emergency. For which of the following conditions should the patient be assessed?

Hypotensive crisis Rationale: Hypotensive crisis may be seen in older patients on nitroprusside. To prevent this, the dosage should be reduced during the initial period of therapy. Thromboembolism, stroke, and blindness will not occur during nitroprusside therapy.

Carter, age 2, is diagnosed with cystic fibrosis; the physician places him on pancrelipase. What would be the most appropriate diagnosis for Carter related to his medication regimen?

Imbalanced nutrition: less than body requirements, related to impaired digestion secondary to insufficient pancreatic enzymes Rationale: Pancrelipase is administered as enzymatic replacement therapy for patients with deficient exocrine pancreatic secretions; cystic fibrosis; chronic pancreatitis; ductal obstructions caused by cancer of the pancreas or common bile duct; pancreatic insufficiency; or steatorrhea from malabsorption syndrome and after pancreatectomy, gastrectomy, or post-GI surgery, such as Billroth II gastroenterostomy. Caution must be used with large doses because they may cause nausea, abdominal cramps, and diarrhea. Hyperuricosuria and hyperuricemia have occurred with extremely high doses. Less often, allergic reactions have occurred.

Milrinone (Primacor) is a miscellaneous inotropic drug used in the short-term management of heart failure. What is the only way this drug is approved to be administered?

Intravenously

You are caring for a 45-year-old client who has been prescribed sucralfate for treatment of peptic ulcer disease. Which of the following nursing diagnoses may relate specifically to administration of this medication?

Impaired Bowel Elimination: Constipation Rationale: Since sucralfate can cause constipation, Impaired Bowel Elimination: Constipation may be relevant to administration of the drug. Although the other diagnoses may be appropriate for the underlying disease, they do not relate to the use of sucralfate.

Which adverse reaction of proton pump inhibitors is specific to menopausal women with osteoporosis?

Increased fracture risk Rationale: An increase in fractures of the hip, wrist, and spine have been seen in clients taking high doses of PPIs and undergoing treatment of osteoporosis with bisphosphonates.

The nursing instructor explains to students that positive inotropic action affects the heart in which way?

Increased myocardial contraction

Ms. Law has been taking omeprazole for the last 10 years. What is the risk to the patient from long-term use of omeprazole?

Increased risk of gastric cancer Rationale: Long-term PPI administration in rats has demonstrated an increased rate of gastric cancer. This finding has not been replicated in humans. PPIs do cause hypergastrinemia, which in turn causes diffuse, linear, or micronodular hyperplasia and atrophic gastritis. These changes occur most frequently in patients with Helicobacter pylori infection and markedly increased gastrin levels. Gastric hyperplasia and atrophic gastritis in combination with H. pylori infection increase the risk of gastric cancer, but there are no studies that show a direct correlation between PPI administration and gastric cancer.

A client with diabetes has started thyroid hormone replacement therapy. The nurse should monitor closely for which effects? -Decreased appetite -Oliguria -Increased thirst -Decreased oral fluid intake

Increased thirst

When describing the effects of incretins on blood glucose control to a group of students, which would an instructor include? -Increases GI emptying -Increases glucagon release -Increases protein building -Increases insulin release

Increases insulin release

The nurse is preparing to administer dexpanthenol to a client based on the understanding that this drug acts in which manner?

Increasing acetylcholine levels

In pharmacology class the students are learning about the mechanism of action of different laxatives. What would the students learn about the mechanism of action of psyllium hydrophilic mucilloid?

Increasing mass and water content of stool, promoting evacuation

Primary medications used in the management of shock are adrenergic drugs. Medications with alpha-adrenergic activity raise blood pressure via:

Increasing peripheral vascular resistance. Rationale: Drugs with alpha-adrenergic activity increase peripheral vascular resistance and raise blood pressure.

When interviewing a client with a suspected worm infection, what would lead the nurse to suspect trichinosis?

Ingestion of undercooked pork

The nurse should explain that metoclopramide (Reglan) decreases nausea and vomiting by which of the following actions?

Inhibiting stimulation of the vomiting center in the brain Rationale: Reglan inhibits stimulation of the vomiting center in the brain. This medication also increases the amplitude of gastric contractions and gastric motility while decreasing gastric emptying time.

The pharmacology instructor is providing education regarding propylthiouracil to the nursing students. What would the instructor identify as the primary mode of action for this medication? -Sedation of the central nervous system and suppression of cardiac function -Suppression of the anterior pituitary gland's hormonal secretions -Destruction of part of the thyroid gland -Inhibition of production of thyroid hormone

Inhibition of production of thyroid hormone

A nurse at a health care facility is assigned to administer insulin to the patient. Which intervention should the nurse perform before administering each insulin dose? -Inspect the previous injection site for inflammation. -Keep prefilled syringes horizontally. -Check for symptoms of myalgia or malaise. -Do not administer insulin kept at room temperature.

Inspect the previous injection site for inflammation

A nurse is preparing to administer a rapid-acting insulin. Which medication would the nurse most likely administer? -insulin detemir -isophane insulin suspension -insulin lispro -insulin glargine

Insulin lispro

A 59-year-old client with vascular disorders is prescribed cilostazol. The nurse instructs the client that it is for the treatment of what condition?

Intermittent claudication

A client is to receive eptifibatide. The nurse would expect to administer this drug by which route?

Intravenous

A nurse is preparing to administer inamrinone. The nurse would administer this drug most likely by which route?

Intravenous

Milrinone (Primacor) is a miscellaneous inotropic drug used in the short term management of heart failure. What is the only way this drug is approved to be administered? (Choose one)

Intravenously

The patient has been placed on a nitroprusside drip for the treatment of a hypertensive crisis. Which mechanism of action does the nurse know is true for nitroprusside?

It directly relaxes vascular smooth muscle, allowing dilation of peripheral arteries and veins. Rationale: Nitroprusside, which is used in hypertensive crisis, directly relaxes vascular smooth muscle, allowing the dilation of peripheral arteries and veins. Angiotensin II receptor blockers block the action of angiotensin II from all the different pathways where it is formed, not just the single substrate altered by ACE inhibitors. Selective aldosterone blockers bind selectively to the mineralocorticoid receptors, thereby blocking aldosterone from binding to these receptors, while calcium channel blockers inhibit the movement of calcium ions across cell membranes.

What is the expected action of sitagliptin on type 2 diabetes? -It slows the rate of inactivation of the incretin hormones. -It inhibits hydrogen, potassium, and ATPase. -It is a synthetically prepared monosodium salt. -It blocks the S phase of the cell cycle.

It slows the rate of inactivation of the incretin hormones

Which of the following lipid levels would the nurse interpret as being high?

LDL cholesterol of 190 mg/dL LDL level of 190 mg/dL would be considered high. A total cholesterol level of 200 mg/dL would be considered borderline high. A triglyceride level of 160 would be borderline high. HDL level of 48 would be considered low to optimal. Levels about 60 mg/dL would be considered high.

Which of the following would be the proton pump inhibitor of choice for a child with GERD?

Lansoprazole Rationale: Lansoprazole is the only proton pump inhibitor approved for use in children and would be the drug of choice.

A client has undergone diagnostic testing for a prolonged history of abdominal pain and diarrhea and been diagnosed with schistosomiasis. The nurse is teaching the client about this condition before treatment begins. Place the stages of the schistosome life cycle in the correct order, beginning after the eggs infect snails in bodies of water.

Larvae develop within snails Larvae are shed into the water Larvae burrow into the skin of humans Larvae mature in the lungs and liver Adults migrate to the intestines and bladder Eggs are shed in human feces

An older patient is experiencing diarrhea. Which is a likely cause of diarrhea?

Laxative abuse

Which is a common cause of diarrhea in older adults?

Laxative overuse

Which of the following is the correct rationale for why the nurse would administer a laxative at a separate time from the client's other medications?

Laxatives may reduce absorption of other drugs present in the GI tract.

After teaching a group of students about the cardiovascular system and pressures, the students demonstrate understanding of the information when they identify which area as having the highest pressure?

Left ventricle Rationale: The area of highest pressure in the system is always the left ventricle during systole. This pressure propels the blood out of the aorta and into the system.

An older adult has been prescribed a thyroid hormone replacement medication. The nurse should first clarify which prescription? -Levothyroxine (Synthroid) 150 mcg orally once per day -Liothyronine (Cytomel) 25 mcg orally once per day -Armour 60 mg orally once per day -Liotrix (Thyrolar) ½ grain tablet orally once per day

Levothyroxine (Synthroid) 150 mcg orally once per day

An older adult has been prescribed a thyroid hormone replacement medication. The nurse should first clarify which prescription? -Liothyronine (Cytomel) 25 mcg orally once per day -Liotrix (Thyrolar) ½ grain tablet orally once per day -Levothyroxine (Synthroid) 150 mcg orally once per day -Armour 60 mg orally once per day

Levothyroxine (Synthroid) 150 mcg orally once per day

Nonpharmacologic management should be tried alone or with drug therapy. What methods of nonpharmacologic management are used in the treatment of hypertension? (Select all that apply.)

Limited alcohol intake Weight reduction Moderate sodium restriction Rationale: Nonpharmacologic management of hypertension should be tried alone or with drug therapy. For example, weight reduction, limited alcohol intake, moderate sodium restriction, and smoking cessation may be the initial treatment of choice if the client is hypertensive and overweight. The hot water in a hot tub may actually increase blood pressure.

In what state does a drug not have to go through the pharamaceutic phase?

Liquid state

Which of the following to lower blood pressure primarily via suppression of the rennin-angiotensin-aldosterone system? (Choose one)

Lisinopril (Prinivil) Rationale: Angiotensin-converting enzyme inhibitors (ACEIs) act primarily for suppress the rennin-angiotensin-aldosterone system and lisinopril is an ACEI. Verapamil and diltiazem are calcium channel blockers and furosemide is a diuretic.

Rosiglitazone is being considered for the treatment of diabetes in an adult male client. Before the initiation of rosiglitazone therapy, the nurse should review what laboratory work recently drawn? -creatinine -liver enzymes -D-dimer -platelet count

Liver enzymes

A group of students are reviewing the various antihypertensive agents. The students demonstrate understanding of the information when they identify which of the following as an example of an angiotensin II receptor blocker?

Losartan Rationale: Losartan is an example of an angiotensin II receptor blocker. Moexipril is an ACE inhibitor. Minoxidil is a vasodilator. Amlodipine is a calcium channel blocker.

What types of worms are nematodes, or roundworms? Select all that apply.

Pinworms Whipworms Threadworms Ascaris

A nurse is performing patient education for a woman who has just been prescribed a bisphosphonate. Which diagnostic and history findings would have prompted the woman's care provider to prescribe a bisphosphonate? -Low bone density and a family history of osteoporosis -Labile moods and short-term memory deficits -Height in the lowest quartile of the population and a history of joint pain -Reports of cold intolerance, recurrent constipation, and evidence of diverticular disease

Low bone density and a family history of osteoporosis

Which would be least appropriate when administering insulin by subcutaneous injection? -Injecting the insulin slowly -Massaging the site after removing the needle -Inserting the needle at a 45-degree angle -Using a 25 gauge 1/2-inch needle

Massaging the site after removing the needle

A client, who experienced hypoglycemia twice in the past week, eats one meal per day and snacks the rest of the day. What client education will the nurse provide for a client who reports regularly experiencing hypoglycemic symptoms? -Daily snacks should increase protein intake. -Meals should be eaten at regular times. -Daily caloric intake should be increased by 200 calories. -Alcohol should be limited and taken only with meals.

Meals should be eaten at regular times.

A nurse is preparing to administer a scheduled dose of levothyroxine to an elderly client who is being treated in the hospital for a respiratory infection. Prior to administering the drug, the nurse should perform what assessment? -assessment of pupillary response -temperature measurement -measurement of blood pressure -chest auscultation for rales

Measurement of blood pressure

The nurse is caring for a patient receiving cardiotonic drugs. The patient has edema. Which intervention should be taken to alleviate edema?

Measurement of intake and output

The nurse is caring for a patient receiving cardiotonic drugs. The patient has edema. Which of the following interventions should be taken to alleviate edema?

Measurement of intake and output

A nurse is reviewing the various anthelmintics. The nurse correctly identifies which drug as killing the parasites by blocking uptake of glucose by the helminth?

Mebendazole

After reviewing class information about anthelminthic agents, a group of students demonstrate understanding of the material when they identify which drug as the prototype anthelmintic?

Mebendazole

A nurse is speaking to a group of parents about pinworm infections. What priority teaching point should the nurse stress?

Meticulous hand washing

The nurse educating a client on the prescription of metronidazole should base the information shared on what statement? Select all that apply.

Metronidazole is contraindicated during the first 3 months of pregnancy. Metronidazole should not be taken by clients with blood disorders. Clients should avoid alcohol when taking metronidazole. Bell's palsy is a nervous system disorder that contraindicates the use of metronidazole.

The nurse is preparing to administer digoxin to a 9-month-old infant. What must the nurse do prior to administration of this medication?

Monitor the infant's apical pulse

The ICU nurse is caring for a patient in shock. What is one of the most important functions of the nursing role in caring for this patient?

Monitoring for complications and side effects of treatment Rationale: General nursing measures include ensuring safe administration of prescribed fluids and medications and documenting their administration and effects. An important function of the nursing role is monitoring for complications and side effects of treatment and reporting them promptly. Options A, C, and D are all correct answers; however, they are not more important functions of nursing care than monitoring for complications and side effects of treatment.

A patient has recently been diagnosed with hypertension. Which of the following outcomes is most important in this patient?

Multiple diastolic blood pressure readings <90 mm Hg Rationale: The diastolic blood pressure below 90 mm Hg is the most important outcome in this patient. Verbalization of understanding of the medical regime is a patient-related outcome that is important but not the most important outcome. Compliance with antihypertensive medications is important but does not confirm a decreased blood pressure. Compliance with a low-sodium diet will assist in decreasing blood pressure but will not confirm that the blood pressure is decreased.

A nurse at a long-term care facility is conducting a medication reconciliation for a client who has just moved into the facility. The client is currently taking clopidogrel. The nurse is most justified suspecting that this client has a history of what condition?

Myocardial infarction

After receiving the results of his echocardiogram, your patient has many questions regarding results and their impact on his lifestyle. He is a 57-year-old Hispanic man with a 10-year history of hypertension. Disturbed by all the dietary changes proposed by the cardiologist, he questions what effect hypertension has on his heart. What is your reply?

Myocardium hypertrophy Rationale: As a result of increased cardiac workload from cardiac wall sclerosis, the myocardium hypertrophies as a compensatory mechanism and heart failure eventually occurs.

A patient is taking lovastatin (Mevacor). Which of the following is noted as the most common adverse effects?

Nausea, flatulence, and constipation The most common adverse effects of statins are nausea, constipation, diarrhea, abdominal cramps or pain, headache, and skin rash. The patient will not experience increased appetite and blood pressure as adverse effects of statins. The patient will not experience fatigue and mental disorientation as adverse effects of statins. The patient will not experience hiccups, nasal congestion, and dizziness as adverse effects of statins.

After teaching a group of students about H-2 receptor antagonists, the instructor determines that the teaching was successful when the students identify which drug as most appropriate for a patient with hepatic dysfunction?

Nizatidine Rationale: Nizatidine is the drug of choice for patients with liver dysfunction because it does not undergo first pass metabolism in the liver like the other histamine-2 receptor antagonists.

A 79-year-old woman is admitted to the hospital with black, tarry stools and several large, scattered ecchymosis. The client has a history of TIAs and has taken clopidogrel for several years. What would the nurse expect to be prescribed to control her bleeding?

None of these

A client admitted to the hospital with hyperthyroidism treated with propylthiouracil suddenly develops a skin rash. Which action would the nurse implement first? -Avoid using soap to cleanse affected areas. -Notify the primary health care provider. -Record weight and report weight gain or loss. -Provide soothing cream to affected areas.

Notify the primary health care provider

A patient is diagnosed with peptic ulcer due to H. pylori. The nurse would anticipate administering which agent in conjunction with an antibiotic?

Omeprazole Rationale: Omeprazole is a proton pump inhibitor which is used as part of combination therapy to treat H. pylori infections. Magaldrate is an antacid that is used to relieve GI hyperacidity. Sucralfate is a GI protectant that is used as short-term treatment of duodenal ulcers. Cimetidine is a H-2 receptor antagonist used to treat duodenal and benign gastric ulcers.

Which of the following would a nurse expect as most likely to be used in combination with antibiotics for treatment of Helicobacter pylori infection?

Omeprazole Rationale: Proton pump inhibitors such as omeprazole are used as part of combination therapy with antibiotics for treatment of Helicobacter pylori infection.

You are caring for a 66-year-old female client who is receiving digoxin. When preparing to administer a dose, you observe that the client's apical pulse rate is 55 bpm. What is the appropriate action to take?

Omit the dose and contact her physician.

A nurse is caring for a 66-year-old female client who is receiving digoxin. When preparing to administer a dose, the nurse observes that the client's apical pulse rate is 55 bpm. What is the appropriate action to take?

Omit the dose and contact the health care provider.

What are the 3 stages of pharmacodynamics?

Onset: 1st initial time you see a response Peak: highest amount of meds in the body Duration: the entire time medication is in the body

A client is to receive clopidogrel. The nurse would expect to administer this agent by which route?

Oral

A client diagnosed with excessive parathyroid production is prone to develop: -osteopenia. -Alzheimer's disease. -heart disease. -hypertension.

Osteopenia

The nurse is discussing the use of corticosteroids with a group of nursing students and tells that students that both men and women who take corticosteroids are at risk for what side effect? -Infertility -Osteoporosis -Hypertension -Paget's disease

Osteoporosis

What values are used to monitor the effectiveness of warfarin therapy?

PT and INR

A client is receiving heparin. Which would the nurse use to monitor the effects of the drug?

Partial thromboplastin time

When speaking of pharmacokinetics, lipid soluble drugs are considered to be?

Passive

What are the three major processes for drug absorption through the gastrointestinal membrane?

Passive and active absorption, and pinocytosis

A patient is taking nonsteroidal anti-inflammatory agents for arthritis of the knees and hips. Which of the following diseases is a result of cellular destruction of the gastrointestinal tract from this medication?

Peptic ulcer disease Rationale: Cell destruction will occur from the ingestion of NSAIDs, which can lead to the development of peptic ulcer disease. Nonsteroidal anti-inflammatory agents do not cause esophageal cancer, bowel obstruction, or liver cancer.

What should be the nurse's initial response when a client diagnosed with type 1 diabetes suddenly reports feeling weak, shaky, and dizzy? -Administer 1 ampule of 50% dextrose intravenously (IV). -Perform a blood sugar analysis. -Administer 10 units of regular insulin subcutaneously (sub-Q). -Have the client drink a 4-ounce glass of orange juice.

Perform blood sugar analysis

After reviewing the signs and symptoms of heart failure, a student demonstrates understanding when the student identifies which finding as associated with right sided heart failure?

Peripheral edema

Which of the following would a nurse expect to assess if a client is experiencing right-sided heart failure?

Peripheral edema

Which would a nurse expect to assess if a client is experiencing right-sided heart failure?

Peripheral edema

A patient is prescribed trandolapril (Mavik). What adverse effect should the patient be instructed on that can occur with angiotensin-converting enzyme inhibitor (ACE inhibitor)?

Persistent cough Rationale: A persistent cough can develop with the use of ACE inhibitors. Sedation, tachycardia, and rash are not noted with ACE Inhibitors.

A client is started on captopril, an ACE inhibitor. The client should be informed of the possibility of what adverse effect?

Persistent cough Rationale: A persistent, nonproductive cough develops in approximately 10% to 20% of clients using ACE inhibitors and may lead to stopping the drug. Hyperkalemia can occur in some patients, such as those who have diabetes mellitus or renal impairment or who are taking nonsteroidal anti-inflammatory drugs, potassium supplements, or potassium-sparing diuretics.

A hospital client's current medication administration record specifies oral administration of propylthiouracil (PTU) every 8 hours. What sign or symptom may have originally prompted the care provider to prescribe this drug? -visual disturbances -persistent tachycardia -orthostatic hypotension -tinnitus

Persistent tachycardia

All of the following statements are true EXCEPT:

Pin-Rid can be used for infants and children of any age.

The nurse walks into the room of a client with type 1 diabetes and finds the client pale and diaphoretic. The client reports a headache and being hungry. Immediately, the client is unable to talk. What is the nurse's immediate intervention for this client? -Give 8 oz orange juice. -Place glucose gel between the gums and cheek. -Administer regular insulin subcutaneously. -Raise the head of the bed.

Place glucose gel between the gums an cheek

After teaching a group of students about laxatives, the instructor determines that the teaching has been successful when the students identify which agent as an example of a bulk laxative?

Polycarbophil

A patient who has been experiencing liquid stools is prescribed polycarbophil (FiberCon). What is the reason for administering a bulk-forming laxative?

Polycarbophil absorbs large amounts of water and decreases fluidity of stools

A patient needing to evacuate the colon for endoscopy would likely take:

Polyethylene glycol electrolyte solution (NuLYTELY)

A client is scheduled for a colonoscopy in the morning. What laxatA client is scheduled for a colonoscopy in the morning. What laxative would the nurse expect to be prescribed the evening before the procedure?ive would the nurse expect to be prescribed the evening before the procedure?

Polyethylene glycol-electrolyte solution

A patient is scheduled for a colonoscopy in the morning. Which laxative is most likely prescribed the evening before the colonoscopy?

Polyethylene glycol-electrolyte solution (NuLYTELY)

A patient is scheduled for a colonoscopy. The nurse knows that the preferred drug for bowel cleansing before this procedure is which of the following?

Polyethylene glycol-electrolyte solution (NuLYTELY)

The pharmacology instructor is describing medications that increase the contractile force of the heart. Which of the following terms describes this effect?

Positive inotropic

A patient with a history of congestive heart failure is being treated with digoxin (Lanoxin). This medication increases the force of contractions of the heart. What effect improves the contractility of the heart?

Positive inotropic effect

A nurse is caring for a patient who is taking digoxin and a loop diuretic. Which of the following would be most important for the nurse to monitor?

Potassium levels

Which of the following agents would be used to treat hypertension by blocking the postsynaptic alpha-1 receptor sites?

Prazosin Rationale: Prazosin is an alpha-1 blocker that is used to treat hypertension. Labetalol and guanabenz are alpha- and beta blockers used to treat hypertension. Nadolol is a beta blocker used to treat hypertension.

HMG-CoA reductase inhibitors (statins) are in which pregnancy category? (Choose one)

Pregnancy Category X HMG-CoA reductase inhibitors (statins) are in pregnancy category X and contraindicated during any stage of pregnancy.

Your patient, a 37-year-old woman, presents with a blood pressure of 118/86 mm Hg. She exercises regularly, is weight-appropriate for her height, and indicates no use of tobacco or alcohol. According to the 2003 guidelines, what would her blood pressure classify?

Prehypertension Rationale: Prehypertension = systolic 120 to 139 mm Hg or diastolic 80 to 89 mm Hg.

A nurse checks a client's blood pressure and finds it to be 130/82. This client should be classified as having which stage of hypertension?

Prehypertension Rationale: Prehypertension is defined as a systolic pressure between 120 and 130 mmHg or a diastolic pressure between 80 and 89 mmHg.

A male client is receiving heparin by continuous intravenous infusion. The nurse will instruct the client and family members to report what should it occur?

Presence of blood in urine or stools

The health care provider discovers a clot in the client's left lower leg. Anticoagulant drugs are prescribed to prevent formation of new clots and to achieve which other effect?

Prevent extension of clots already present

The pharmacology instructor is discussing heparin with the students. How would the instructor explain the action of heparin?

Promotes the inactivation of clotting factors

How can the home care nurse assist the client or the client's family members who have hypertension? (Select all that apply.)

Promoting compliance with the prescribed pharmacologic modifications Promoting compliance with the prescribed lifestyle modifications Monitoring for drug effects Rationale: Whether the client or another member of the household is taking antihypertensive medications, the home care nurse may be helpful in teaching about the drugs, monitoring for drug effects, and promoting compliance with the prescribed regimen (pharmacologic and lifestyle modifications). The nurse would not provide financial assistant with daily expenses though the nurse would help the client and family discover resources that might provide what is needed.

A client exhibits signs and symptoms of heparin overdose. The nurse would anticipate administering:

Protamine sulfate

Which of the following best reflects the rationale for using histamine-2 receptor antagonists for stress ulcer prophylaxis?

Protects the stomach lining via acid blockage Rationale: Histamine-2 receptor antagonists are used for stress ulcer prophylaxis because the drugs block the production of acid thereby protecting the stomach lining, which is at risk because of decreased mucus production. Reducing the overall acid level is the rationale for use as short-term treatment of active duodenal ulcer. Blocking the overproduction of hydrochloric acid is the rationale for treatment of pathologic hypersecretory conditions. Decreasing the acid being regurgitated into the esophagus is the rationale for treatment of erosive gastroesophageal reflux.

A nurse is caring for a 22-year-old woman who has just been diagnosed with an helminthic infection. The client is terrified and states she feels "grossed out and embarrassed." The nurse makes a diagnosis of Disturbed Personal Identity. Which interventions would be important to include? Select all that apply.

Provide education and reassurances regarding diagnosis and treatments, including medications. Provide reassurance that her medical condition and record are confidential.

The nurse is caring for a client who will need to use a medication to assist in the prevention of constipation for an extended period of time. The nurse is aware that which drug is the most desirable for long-term use?

Psyllium

When considering the management of diabetic ketoacidosis (DKA), what type of insulin can be administered intravenously? -regular -isophane insulin (NPH) -insulin glargine -lispro

Regular

A group of students are reviewing information about the indications for laxatives. The students demonstrate understanding of the information when they identify which of the following as an indication?

Remove ingested poisons from the lower GI tract

The nurse would be alert to signs and symptoms of which of the following in a patient receiving albendazole therapy and dexamethasone?

Renal failure

Which condition would least likely contribute to the development of heart failure?

Renal failure

A provider chooses to treat an African American client suffering from hypertension with a beta blocker and diuretic instead of an ACE inhibitor. The nurse knows that the provider did this for what reason?

Research has shown beta blockers, in combination with diuretics, treat hypertension in African Americans better than ACE inhibitors. Rationale: Cost, dark skin, and prescribing multiple medications are not the reasons a beta blocker and a diuretic are used to treat hypertension in African American clients. Research has shown that the combination of a beta blocker and a diuretic treat hypertension in African Americans more effectively than other common hypertension medications.

A student asks the instructor what the goal of drug therapy is in hypotension and shock. What would the instructor respond?

Restore and maintain adequate tissue perfusion. Rationale: The goal of adrenergic drug therapy in hypotension and shock is to restore and maintain adequate tissue perfusion, especially to vital organs.

What is the most serious side effect of lovastatin?

Rhabdomyolysis One adverse effect with potentially serious consequences is muscle damage, the exact cause of which is not known. The most serious skeletal muscle effect that may result from lovastatin is rhabdomyolysis, although it is very rare (occurring in 0.1% of patients on statins as monotherapy). Rhabdomyolysis is an acute, sometimes fatal disease, in which direct injury to the plasma membrane of the skeletal muscle occurs (manifested by increased levels of CK, also known as creatine phosphokinase [CPK]). The damage to the muscle causes leakage of the skeletal muscle components (myoglobin) into the blood or the urine; brown urine usually results.

A nurse is planning care for a 59-year-old woman who is on ranitidine therapy. The nurse is concerned for the patient's safety. Which of the following would be an appropriate nursing diagnosis?

Risk for Injury related to drug-induced somnolence, dizziness, confusion, or hallucinations Rationale: The appropriate nursing diagnosis related to safety would be Risk for Injury related to drug-induced somnolence, dizziness, confusion, or hallucinations. Diarrhea related to adverse effects of drug therapy and Acute Pain related to adverse drug effects, headache are appropriate nursing diagnosis for a patient taking ranitidine, but are not related to safety. Potential Complication: Electrolyte Imbalance related to hypophosphatemia, secondary to drug therapy would be appropriate for a patient taking an aluminum hydroxide with magnesium hydroxide antacid (Maalox, Mylanta).

The nurse has been caring for a child who has been receiving growth hormone therapy for several years. When the child returns for evaluation following a sudden growth spurt, what nursing diagnosis should the nurse most likely add to the plan of care? a.Disturbed body image related to change in height b.Deficient knowledge regarding drug therapy c.Risk for imbalanced nutrition: less than body requirements related to metabolic changes d.Decreased cardiac output related to increased metabolic needs

Risk for imbalanced nutrition: less than body requirements related to metabolic changes Feedback: A child who is taking growth hormone may experience sudden growth, which will require increased nutritional intake, so it is important to include nutritional needs in the plan of care. More than likely an increase in caloric intake and nutrients will be necessary. Most children who are small for their age see growth as a positive thing and not a disturbed body image. After taking the drug for several years, the patient should have received adequate teaching from the nurse ot make deficient knowledge unlikely. The child should not be at risk for, but have a diagnosis of, disproportionate growth as the reason for taking the medication.

A patient is experiencing orthostatic hypotension that is due to his antihypertensive therapy. Which nursing diagnosis would be most appropriate?

Risk for injury Rationale: Risk for injury is appropriate because the changes in blood pressure with position changes increases the patient's risk for falls. Ineffective airway clearance would be appropriate if the patient has copious respiratory secretions or the unrelenting cough of ACE inhibitors. Impaired peripheral tissue perfusion would be appropriate if the patient was experiencing changes in the color or circulation to his extremities. Fluid volume deficit would be appropriate if the patient was dehydrated.

The nurse is developing a plan of care for a patient who has been admitted to the hospital after a myocardial infarction. The patient begins lovastatin to lower his LDL levels and increase his HDL levels. Based on the adverse reaction of this medication, what would be the most appropriate nursing diagnosis?

Risk for injury to skeletal muscles related to adverse effects of drug therapy The only diagnosis that focuses on the risk factors associated with taking the medication is risk for injury to the skeletal muscles. The other diagnoses focus on the patient's risk factors prior to the medication or the problem associated with having a heart attack.

Prior to administering propylthiouracil, the nurse has reviewed the relevant black box warning and should teach the client about the need for what form of follow-up? -chest radiographs every 3 months -serial complete blood counts -routine liver function testing -daily nonfasting blood glucose testing

Routine liver function testing

During a routine check up of a 45-year-old patient with renal disease, the nurse observes an increase in the patient's blood pressure. Which of the following is the most likely consequence of renal impairment?

Secondary hypertension Rationale: Secondary hypertension results as a consequence of renal impairment. In secondary hypertension there is usually a known cause for the development of hypertension. Renal disease is one of the causes of secondary hypertension. When there is no known cause of hypertension it is called essential hypertension. Rebound hypertension occurs when a patient abruptly stops taking antihypertensive medication. Hypertensive emergency is a high blood pressure state which has to be lowered immediately.

The nurse is caring for a client receiving tolvaptan and digoxin. When assessing for drug-drug interactions, the nurse should prioritize what laboratory result? a.Creatinine level b.Digoxin level c.Serum potassium level d.Hematocrit

Serum potassium level Feedback: Tolvaptan should be used with care with digoxin, which could cause hyperkalemia, so the nurse must carefully monitor serum potassium levels. The combination of drugs would not cause reduced digoxin levels or tolvaptan toxicity. The indication for administering tolvaptan would be to treat hyponatremia, so an elevation in serum sodium levels to normal range would be an indication the drug was working but would not be a result of a drug-drug interaction.

A nurse is caring for a patient who is diabetic and has been diagnosed with hypertension. An angiotensin-converting enzyme inhibitor, captopril, has been prescribed for her. Which of the following should the nurse assess before beginning drug therapy?

Serum potassium levels Rationale: Captopril inhibits the angiotensin-converting enzyme that is needed to change the inactive angiotensin I to the active form, angiotensin II. This reduction of angiotensin II decreases the secretion of aldosterone, preventing sodium and water retention. This action decreases peripheral vascular resistance and lowers blood pressure. Serum potassium may increase as a result of decreased aldosterone levels. It would be helpful to have a baseline level at the beginning of the drug therapy to monitor the possible effect of hyperkalemia. There is no need to monitor calcium and magnesium levels. While it is important to always know a diabetic's blood glucose level, captopril does not alter the level during therapy.

When providing client teaching to the family of a 12-year-old child receiving somatropin, the nurse stresses the need to notify prescriber if what manifestation occurs? a.Severe hip or knee pain b.Upper respiratory infection lasting more than 6 days c.Nosebleeds d.Dry skin accompanied by pruritus

Severe hip or knee pain Feedback: The adverse effects that most often occur when using a growth hormone (GH) include the development of antibodies to GH and subsequent signs of inflammation and autoimmune-type reactions, such as swelling and joint pain, and the endocrine reactions of hypothyroidism and insulin resistance. It would not be necessary to notify the physician for development of a bruise. The health care provider should always be notified if a patient develops severe hypertension or tachycardia but this would not be related to administration of somatropin so it would not be included in drug teaching.

A patient is brought to the emergency department in hypertensive crisis. Nitroprusside is administered intravenously. The patient experiences diaphoresis and dizziness. Which of the following is the appropriate action by the nurse?

Slow the rate of the infusion Rationale: Administering nitroprusside too quickly can result in abdominal pain, apprehension, diaphoresis, dizziness, headache, muscle twitching, nausea, palpitations, restlessness, retching, and retrosternal discomfort. The nurse should quickly inform the prescriber and slow the infusion. Symptoms quickly subside when the nitroprusside infusion is slowed or stopped, and usually do not return when the infusion resumes at a slower rate. Consulting the prescriber for a dose change would not help; the rate of drug administration is the issue. Continuing to give the nitroprusside at the ordered rate could be fatal.

Which of the following activities increases high-density lipoproteins?

Smoking cessation Weight loss Regular aerobic exercise Weight loss, regular aerobic exercise, and smoking cessation (for those who smoke) all increase HDL cholesterol.

A patient reports chronic abdominal pain. The patient has a 40-pack year history but reports a decrease in smoking to approximately ½ pack/day. The physician, after thorough assessment and evaluation and labs, prescribes a treatment regimen for duodenal ulcer. What important teaching would the nurse include in relation to treatment?

Smoking's effects on healing Rationale: Duodenal ulcers are associated with cigarette smoking. The ulcers of smokers heal more slowly and recur more rapidly than do those of non-smokers.

Which of the following antacids is contraindicated in patients with congestive heart failure? (Choose one)

Sodium bicarbonate (Bellans) Rationale: Sodium bicarbonate is the antacid that is contraindicated in patients with cardiovascular problems, such as hypertension and congestive heart failure, and those on sodium-restricted diets, as sodium bicarbonate can alter the body's sodium-water balance and cause fluid retention.

A patient has an elevated total serum cholesterol of 260. Which of the following aspects of patient teaching of lifestyle changes is most important for the patient?

Stop smoking Therapeutic lifestyle changes to lower serum cholesterol, including exercise, smoking cessation, change in diet, and drug therapy, are recommended to lower serum cholesterol. The patient should increase dietary vegetables in a heart-healthy diet. The patient with an elevated serum cholesterol should increase exercise and not increase rest periods. The patient should consume low-fat dairy products and avoid whole milk.

The nurse is caring for a client who has developed severe diarrhea a few days after beginning an antibiotic prescribed for a severe respiratory infection. The nurse is prepared to initiate what initial treatment?

Stopping the causative drug

A nurse is preparing an in-service presentation for a group of staff members on diabetes. Which would the nurse include as the primary delivery system for insulin? -Subcutaneous injection -Insulin pen -External pump -Jet injector

Subcutaneous Injection

A nurse is preparing an in-service presentation for a group of staff members on diabetes. Which would the nurse include as the primary delivery system for insulin? -External pump -Jet injector -Insulin pen -Subcutaneous injection

Subcutaneous injection

A nurse is caring for a patient undergoing thyroid hormone replacement therapy. What should the nurse inform this patient regarding administration of the drug? -Take the drug before bedtime. -Take the drug just before dinner. -Take the drug before breakfast. -Take the drug after lunch.

Take the drug before breakfast

A patient with congestive heart failure has been digitalized. The patient requires long-term digoxin therapy. Which of the following instructions should the nurse provide the patient on discharge?

Take the drug regularly without skipping a dose.

A client was diagnosed with type 2 diabetes several months ago and has presented for a scheduled follow-up appointment. Which stated behavior most clearly indicates that the client has established effective health maintenance? -The client can describe the differences between type 1 and type 2 diabetes. -The client exercises two to three times per week. -The client tells the nurse that he/she has gone on a diet. -The client frequently checks his/her blood glucose levels.

The client frequently checks his/her blood glucose levels

Which condition must be met in order for glyburide treatment to be effective? -The client must have hemoglobin A1C of ≤7%. -The client must have functioning pancreatic beta cells. -The client must not have hyperglycemia. -The client must be able to self-administer the medication.

The client must have functioning pancreatic beta cells.

A client with filariasis asks the nurse how the infection was transmitted. The nurse would incorporate an understanding of what information when responding to the client?

The client was bitten by an insect that contained worm embryos.

A 21-year-old client is currently breastfeeding her 5 month-old infant. The client has been diagnosed with pinworms. What affect should the nurse anticipate when the client is prescribed anthelmintic medication for pinworms?

The client will be encouraged to stop breastfeeding while being treated

A patient who has an elevated triglyceride level and reduced high-density lipoprotein cholesterol is seen by her primary care physician. What do these laboratory tests indicate in this patient?

The development of metabolic syndrome Metabolic syndrome is noted when the patient has elevated waist circumference, elevated triglycerides, reduced high-density lipoprotein cholesterol, elevated blood pressure, and elevated fasting glucose. Elevated triglyceride level and reduced high-density lipoprotein cholesterol are not indicative of arthritic syndrome. Reye's syndrome is marked by acute encephalopathy and seen in children under the age of 15 years after an acute viral infection. Tay-Sachs is a genetic disease characterized by neurological deterioration in the first year of life.

A client has an elevated BUN. The client has been prescribed digoxin for heart failure. What aspect of care is the priority regarding this client?

The dose should be decreased in this client. The dose must be reduced in the presence of renal failure because most of the digoxin is excreted unchanged by the kidneys, leading to drug accumulation and toxicity. The client should be taught to limit sodium intake in the diet. The client's heart rate should remain above 60. If the heart rate falls below 60, the digoxin should be held. The dose of 1.0 mg is too large for a client with altered renal function.

A patient has an elevated BUN. The patient has been prescribed digoxin (Lanoxin) for heart failure. What factor is important regarding this patient?

The dose should be decreased in this patient.

A patient who is malnourished and has hypertension is being treated with losartan (Cozaar). How should the dose of the medication be adjusted in this patient?

The dose should be lower than normal. Rationale: Both losartan and the metabolite are highly bound to plasma albumin, and losartan has a shorter duration of action. Due to malnutrition, a low-dose of losartan should be prescribed. The dose should not be higher than normal due to possible toxicity. The dose should not be the same as normal due to possible toxicity. The medication will not be combined with a diuretic unless the blood pressure is not controlled.

Pharmacokinetics is the study of?

The movement of medication. It involves the study of absorption, distribution, metabolism (biotransformation), and excretion of drugs

Following an assessment by her primary care provider, a 70-year-old resident of an assisted living facility has begun taking daily oral doses of levothyroxine. Which assessment finding should prompt the nurse to withhold a scheduled dose of levothyroxine? -The resident's apical heart rate is 112 beats/minute with a regular rhythm. -The resident received her annual influenza vaccination the previous day. -The resident had a fall during the night while transferring from her bed to her bathroom. -The resident has not eaten breakfast because of a recent loss of appetite.

The resident's apical heart rate is 112 beats/minute with a regular rhythm.

Beta-blockers reduce both morbidity and mortality associated with heart failure. By what mechanism do they work in heart failure?

They block the sympathetic response.

What information would be included in a lipoprotein profile? Select all that apply:

Total cholesterol LDL Triglycerides A lipoprotein profile is a laboratory test which reports total cholesterol, LDL, HDL, and triglycerides. AST and ALT are values that would be found reported from liver function test.

Cholesterol is the base unit for the formation of steroid hormones.

True

Heart failure was once called dropsy.

True

Nesiritide is a human B-type natriuretic peptide.

True

Pancrelipase would most likely be used to treat a patient with cystic fibrosis.

True

Schistosomes invade tissue.

True

The use of calcium carbonate is associated with the development of acid rebound.

True

True or False: Regular insulin may be administered intravenously or intramuscularly in an emergency situation.

True

True or false: liquids have a quicker effect that solid medication

True

Vasodilators used to treat heart failure include nitrates and ACE inhibitors.

True

A patient with metabolic syndrome usually has increased plasminogen activator levels.

True Metabolic syndrome involves increase plasminogen activator levels.

In treating Helicobacter pylori infection, a physician prescribes two antimicrobials. Why two?

Two are needed to prevent the emergence of drug-resistant H. pylori organisms. Rationale: Effective combinations include two antimicrobials. For the antimicrobial component, two of the following drugs—amoxicillin, clarithromycin, metronidazole, or tetracycline—are used. A single antimicrobial agent is not used because of concern about emergence of drug-resistant H. pylori organisms.

A 30-year-old nonpregnant patient is prescribed misoprostol. Which of the following should the nurse instruct the patient as part of the teaching plan?

Use a reliable contraceptive. Rationale: The nurse should instruct the patient to use a reliable contraceptive to avoid pregnancy during the course of treatment as it can cause birth defects. The nurse should instruct the patient taking proton pump inhibitors not to chew, open, or crush the tablet but to swallow the tablet whole at least one hour before eating. The nurse needs to inform the patient to take the drug for motion sickness about one hour before travel.

A 23-year-old female patient is prescribed misoprostol (Cytotec). Which of the following patient teaching interventions is most important?

Use effective contraceptive measures Rationale: Misoprostol (Cytotec) is used to prevent NSAID gastric ulcer development. The medication can cause spontaneous abortion; thus, the medication should be administered with a good form of birth control. It should not be administered with magnesium antacids. It is not necessary for a patient aged 23 years to report postmenopausal bleeding. The medication should be administered two to four times per day.

A young adult client has been diagnosed with roundworms following the results of stool testing. The client states, "I'm absolutely mortified. I can't even imagine anything more disgusting." What should the nurse do when responding?

Validate the client's feelings without characterizing the diagnosis in the same way

A group of nursing students are reviewing cardiotonic drugs. The students demonstrate understanding of the information when they identify which adverse reaction associated with cardiotonic medications?

Visual disturbances

Which of the following is an adverse reaction associated with cardiotonic medications? (Choose one)

Visual disturbances

The nurse is assisting a client who has just begun medication therapy for hypothyroidism. Which nursing assessment is most important in this client? -Pain -Vital signs -Skin -Vision

Vital Signs

The nurse is caring for a client taking insulin. The nurse realizes the client is experiencing symptoms of hypoglycemia when the client displays: -increased pulse rate and fruity smelling breath. -decreased respiratory rate and hot, dry skin. -increased thirst and increased urine output. -weakness, sweating, and decreased mentation.

Weakness, sweating, and decreased mentation

The nurse should advise a client who is taking levothyroxine for the first time that resolution of hypothyroid symptoms may not occur for how long after therapy is started? -days -weeks -months -hours

Weeks

What factors influence drug effects?

Weight Age Gender Physiological Factors Pathological Factors Genetic Factors Immunological Factors Psychological Factors Environmental Factors Drug Tolerance Cumulative Effect Interactions

Which statement reflects the relationship between calcium and phosphate? -When there is an increase in calcium, the phosphate is elevated. -There is no relationship with calcium and phosphate. -When there is a decrease in calcium, the phosphate is decreased. -When there is an increase in calcium, the phosphate is decreased.

When there is an increase in calcium, the phosphate is decreased

What statement should be the basis for a nurse's description of the role of the parathyroid on the development of osteoporosis? -When there is too much thyroid hormone, the parathyroid releases calcium into the blood at a rate that is too high, resulting in bones that have too little calcium. -When there is too much parathyroid hormone, the bones release their calcium into the blood at a rate that is too high, resulting in bones which have too little calcium. -When there is too much parathyroid hormone, the bones retain calcium at a rate that is too high, resulting in bones that have too little calcium. -When there is too much parathyroid hormone, the thyroid releases calcium into the blood at a rate that is too high, resulting in bones that have too little calcium.

When there is too much parathyroid hormone, the bones release their calcium into the blood at a rate that is too high, resulting in bones which have too little calcium.

A nurse assessing a patient on digoxin observes features of toxicity. Which of the following visual disturbances should the nurse assess for in such patients?

Yellow or green vision

A 26-year-old white male patient has been prescribed captopril for hypertension. A nurse has been assigned to the patient to provide education regarding the use of this drug. The nurse will advise the patient that

a persistent, dry cough may occur; however, it is not serious. Rationale: The nurse will advise the patient that the captopril may produce a persistent, dry cough that is not serious. Patients may want to discontinue therapy because of the cough, but the nurse should encourage the patient to continue therapy and help the patient minimize the cough. However, if the cough becomes intolerable, the patient should contact the prescriber. The patient should be instructed to take the first dose at bedtime to minimize the possibilities of injury from first-dose hypotension. The patient should be advised to notify the prescriber promptly if sore throat; fever; swollen hands or feet; irregular heartbeat; chest pain; swollen face, eyes, lips and tongue; difficulty breathing; or hoarseness occur. These effects could indicate angioedema, which can be life threatening. If the patient normally uses a salt substitute containing potassium or a potassium supplement, these substances may need to be discontinued to avoid possible hyperkalemia. Hypokalemia is not a concern.

The nurse is caring for a client who has been receiving long-term growth hormone treatment to stimulate growth. What diagnostic testing would the nurse expect to see ordered as a standard part of the treatment plan? Select all that apply. a.Blood sugar level b.Serum electrolytes c.X-ray of the long bones d.Liver enzyme levels e.Computed tomography of the head

a.Blood sugar level b.Serum electrolytes c.X-ray of the long bones Feedback: Periodic radiography of the long bones, as well as monitoring of blood sugar levels and electrolytes, should be a standard part of the treatment plan for children who receive any of the hypothalamic or pituitary agents. There would be no indication for nasal assessment because growth hormone is not administered nasally. Bone density would not be impacted by these drugs.

While the nurse is talking with a client, the client informs the nurse that the client has been under treatment for weight loss management. The client indicates taking antithyroid medication to increase weight loss. Included in client education about antithyroid medication is that taking: -antithyroid medication will not contribute to weight loss; it only results in fat redistribution. -antithyroid medication will increase weight loss; however, the client needs to take additional dietary supplements of iodine as well. -antithyroid medication will increase weight loss; however, the client needs to take a calcium supplement as well. -excessive or unnecessary antithyroid medication may produce serious or life-threatening manifestations of toxicity.

excessive or unnecessary antithyroid medication may produce serious or life-threatening manifestations of toxicity.

The nurse is caring for a trauma client who presents with total body edema and scant urine output. The nurse should recognize the effects of what endocrine disruption? a.Excessive secretion of antidiuretic hormone b.Insufficient secretion of antidiuretic hormone c.Excessive secretion of androgens d.Acute growth hormone deficiency

excessive secretion of antidiuretic hormone Explanation: This client's presentation is suggestive of syndrome of inappropriate antidiuretic hormone. SIADH occurs with excessive secretion of ADH. Insufficient secretion of ADH causes diabetes insipidus, which is the opposite problem. ADH is not related to androgenic actions or a decrease in growth hormone levels.

A nurse is caring for a patient who has been prescribed lovastatin to control blood lipid levels. While teaching the patient about the medication, the nurse should caution the patient against consuming large amounts of:

grapefruit juice. Consumption of large amounts of grapefruit juice during lovastatin therapy can increase serum drug levels.

The most common adverse effects of lovastatin include:

headache and flatulence. The most common adverse effects of statins include GI symptoms (including nausea, constipation, flatulence, and abdominal pain), headache, and muscle aches. These effects are usually mild and transient.

The nurse is caring for a 76-year-old client who is receiving IV heparin 5000 units q4h. At the initiation of therapy, the client's control activated thromboplastin time (aPTT) was found to be 35 seconds. One hour prior to the next scheduled dose, aPTT is determined to be 92 seconds. Based on this result, the nurse will:

hold the dose and contact the provider.

A nurse has poured a hospital client's scheduled dose of hydrochlorothiazide (HCTZ). The nurse should know that this drug reduces preload by which means?

increasing renal excretion of sodium and water Hydrochlorothiazide inhibits the reabsorption of sodium and chloride in the distal renal tubule, increasing the excretion of sodium and water by the kidneys. Thiazide diuretics do not relax smooth muscle, increase contractility, or stimulate the SA node.

superinfection

infections that occur when opportunistic pathogens that were kept in check by the "normal" bacteria have the opportunity to invade tissues and cause infections because the normal flora bacteria have been destroyed by antibiotic therapy

Ranitidine and cimetidine are both H2RAs used in the treatment of peptic ulcer disease. Of the two, ranitidine is preferred in critically ill clients because it is less likely to

interfere with the metabolism of other drugs. Rationale: Unlike cimetidine, ranitidine does not affect the cytochrome P450 drug-metabolizing system in the liver and therefore does not interfere with the metabolism of other drugs by that system.

What plays the most important role in excretion of medication?

kidneys

The nurse is interviewing the latest admission to the GI floor. The female patient requests a magnesium citrate "cocktail" every night to "keep her regular." Her request is evidence of potential:

laxative abuse

A client presents to the ED with wheezing and blood-tinged sputum. The nurse suspects the client is experiencing pulmonary edema. The nurse should suspect the cause of the pulmonary edema is most likely:

left ventricular failure Pulmonary edema occurs when left ventricular failure (or dysfunction) results in accumulation of blood and fluid in pulmonary veins and tissues.

A 52-year-old man is suffering from a deficiency of exocrine pancreatic secretions and is prescribed pancrelipase (Pancrease MT). Before the medication therapy begins, the nurse will assess for allergies related to

pork. Rationale: Pancrelipase is contraindicated in patients who are hypersensitive to pork protein or enzymes because the drug is derived from pork. Allergies to ragweed, pollen, and shellfish are not associated with the use of the drug.

A client with a history of heart failure is being treated with digoxin. The nurse knows that this medication increases the force of contractions of the heart. What effect improves the contractility of the heart?

positive inotropic effect

An older adult resident of a long-term care facility has been prescribed calcium citrate to address decreasing bone density. The nurse should review the resident's medication administration record knowing that what medication may decrease the effects of calcium? -diltiazem -prednisone -ibuprofen -hydrochlorothiazide

prednisone

A client is being treated with heparin therapy for a pulmonary embolism. The nurse caring for this client knows that heparin:

prevents the extension of the clot that has formed in lungs.

What medication will be a likely component of therapy for a client admitted with a diagnosis of thyroid storm? Select all that apply. -levothyroxine -propylthiouracil -potassium iodide (SSKI) -digoxin -propranolol

propranolol

A client accidentally ingested extra doses of warfarin at home and now presents with an elevated INR and bleeding. Which medication will the nurse prepare as prescribed to administer along with vitamin K?

prothrombin complex concentrate

spectrum

range of bacteria against which an antibiotic is effective (e.g., broad-spectrum antibiotics are effective against a wide range of bacteria; narrow-spectrum antibiotics are effective only against very selective bacteria)

A client with diabetes insipidus has been administered desmopressin and is now reporting drowsiness, light-headedness, and headache. What intervention will best address this client's symptoms? a.Temporary bed rest b.STAT administration of epinephrine c.Administration of a loop diuretic d.Reduction in the client's dose of desmopressin

reduction in the client's dose of desmopressin Explanation: The adverse effects associated with the use of desmopressin include water intoxication (drowsiness, light-headedness, headache, coma, convulsions) related to the shift to water retention and resulting electrolyte imbalance. These symptoms warrant a reduction in dose. The use of diuretics would not resolve the client's symptoms. Epinephrine would resolve anaphylaxis, not water intoxication. Best rest would have no therapeutic effect beyond ensuring physical safety.

The client has been taking rosuvastatin for hyperlipidemia. The client now presents with severe weakness. The client states barely being able to move the extremities because of weakness. The nurse suspects the client has:

rhabdomyolysis. An adverse reaction of rosuvastatin is muscle toxicity, which may cause rhabdomyolysis. Rhabdomyolysis is the breakdown of muscle that can cause pain, weakness, nausea, and even kidney damage. Gallbladder disorders are related to the use of fibric acid derivatives.

An older adult client has been admitted to the emergency department with severe chest pain. Onset of symptoms is within the last 60 minutes. What medication would the nurse expect the health care provider to prescribe for this acute disorder?

thrombolytic drugs

A client, newly diagnosed with heart failure, questions why the therapy with digoxin will begin with four doses of digoxin rather than the usual one dose, in a 24-hour period. What is the rationale for the nurse's response?

to rapidly bring the client's serum digoxin levels up to therapeutic levels

A female client comes to the clinic with reports of burning pain in the stomach. She states that she thinks she had a virus 48 hours ago and she vomited many times. She hasn't vomited in 24 hours. She asks the physician to give her a prescription for omeprazole because she saw an ad on TV that said it would heal stomach pain. The nurse assists the client in understanding that PPIs are considered drugs of choice for treatment of what conditions? (Select all that apply.)

• Duodenal ulcers • Zollinger-Ellison syndrome • Gastric ulcers Rationale: PPIs are considered drugs of choice for treatment of heartburn, gastric and duodenal ulcers, GERD, esophagitis, and hypersecretory syndromes such as Zollinger-Ellison syndrome. It is not used in the treatment of anaphylaxis.

Which of the following reduce the secretion of gastric acid by inhibition of the hydrogen-potassium adenosine triphosphate (ATPase) enzyme system of the gastric parietal cells? (Select all that apply)

• Esomeprazole (Nexium) • Lansoprazole (Prevacid) Rationale: The proton pump inhibitors, like esomeprazole (Nexium) and lansoprazole (Prevacid) reduce the secretion of gastric acid by inhibition of the hydrogen-potassium adenosine triphosphate (ATPase) enzyme system of the gastric parietal cells.

When caring for an older adult client taking an antacid, the nurse should institute which safety precautions? Select all that apply.

• Fall risk • Bed alarm • Strict intake and output Rationale: The older adult should be monitored closely for fluid and electrolyte disturbances while taking antacids. Signs of dehydration include decreased urine output, restlessness, dizziness, and confusion. These symptoms increase the risk of falls. The client should be assisted when ambulating. A bed alarm can alert the nurse to restlessness and attempts to get out of bed alone. Contact precautions are transmission-based precautions and are not necessary. Fluid and electrolyte imbalance are not transmitted from one client to another. Strict intake and output measurements are necessary to assess for decreased urinary output.

Which of the following reduce the secretion of gastric acid by inhibiting the action of histamine at H2 receptors in the stomach? (Select all that apply)

• Famotidine (Pepcid) • Ranitidine (Zantac) Rationale: Ranitidine and famotidine are drugs that reduce the secretion of gastric acid by inhibiting the action of histamine at H2 receptors in the stomach.

In preparing a teaching plan for a patient who is to receive misoprostol, which of the following would the nurse include as possible adverse effects? Select all that apply.

• Flatulence • Dyspepsia • Excessive vaginal bleeding • Dysmenorrhea Rationale: Flatulence is a possible adverse effect. Excessive vaginal bleeding is a possible adverse effect of misoprostol. Dyspepsia is a possible adverse effect of misoprostol. Dysmenorrhea is a possible adverse effect of misoprostol. Dizziness is not associated with misoprostol. Vertigo is not associated with misoprostol.

A client is receiving sodium bicarbonate orally. Which of the following would lead the nurse to suspect that the client is developing systemic alkalosis? (Select all that apply.)

• Headache • Confusion • Irritability • Tetany Rationale: Indications of systemic alkalosis include headache, confusion, irritability, tetany, nausea, and weakness.

Which of the following drugs are commonly used in combination with certain antibiotics in the treatment of H. pylori? Select all that apply:

• Omeprazole (Prilosec) • Lansoprazole (Prevacid) Rationale: The proton pump inhibitors, like omeprazole (Prilosec) and lansoprazole (Prevacid), are commonly used in combination with certain antibiotics in the treatment of H. pylori.

When dosing digoxin (Lanoxin), which factors are considered? (Select all that apply.)

• Patient lean body weight • Patient age • Renal function • Pharmacokinetics • Pharmacodynamics

When dosing digoxin (Lanoxin), which of the following factors are considered? Select all that apply:

• Patient lean body weight • Patient age • Renal function • Pharmacokinetics • Pharmacodynamics

A nurse should be aware that use of statin drugs is contraindicated in clients with which of the following conditions? Select all that apply:

• Pregnancy • Serious hepatic disease • Lactation Statin drugs are contraindicated in individuals with known hypersensitivity to the drugs, serious liver disease, and during pregnancy (Category X), and lactation.

What should the nurse review about an antihyperlipidemic medication with the client when completing discharge counseling? Select all that apply:

• Reason for taking the prescribed drug • Dosage form • Correct dose • Frequency of administration • Drug name The nurse should review the reasons for the drug and prescribed therapy, including drug name, form and method of preparation, correct dose, and frequency of administration as part of a client's discharge counseling.

The administration of a calcium channel blocker results in which of the following? Select all that apply:

• Relaxation of blood vessels. • Increase oxygen supply to the heart. • Reduce the workload on the heart. • Decrease blood pressure. Rationale: The use of calcium channel blockers results in relaxation of blood vessels, increase oxygen supply to the heart, reduce the workload on the heart, and decrease blood pressure.

During ongoing assessment of a client receiving an antihyperlipidemic drug, the nurse should collect the following? Select all that apply:

• Vital signs • Assessment of bowel function Clients on antihyperlipidemic medications should have vital signs checked and bowel function assessed.


Conjuntos de estudio relacionados

Psych/Mental Health Nursing Final Exam Review

View Set

TOEIC | Đọc hiểu 5 - Đề 14

View Set

Unit 1 "The Living World: Ecosystems" - AP Environmental Science Topic Questions

View Set

Evidence Based Practice- MIDTERM

View Set

Measures of Development Ch 17 APHG

View Set

Chapter 21 Law for Business and Personal Use

View Set

Pharmacological and parenteral therapies quiz

View Set